Arrondo Teoria De Numeros

  • Uploaded by: Anonymous iH6noeaX7
  • 0
  • 0
  • March 2021
  • PDF

This document was uploaded by user and they confirmed that they have the permission to share it. If you are author or own the copyright of this book, please report to us by using this DMCA report form. Report DMCA


Overview

Download & View Arrondo Teoria De Numeros as PDF for free.

More details

  • Words: 38,892
  • Pages: 82
Loading documents preview...
Apuntes de Teor´ıa Elemental de N´ umeros por Enrique Arrondo(*)

Versi´ on del 19 de Enero de 2009 Aunque en orden diverso, estas notas est´ an basadas en el libro “Elementary Number Theory”, de David M. Burton, donde el lector puede profundizar en los detalles.

0. Preliminares 1. Divisibilidad y factorizaci´on en enteros 2. Teor´ıa de congruencias 3. Funciones aritm´eticas ´ 4. Ordenes, ra´ıces primitivas e ´ındices 5. Congruencias cuadr´ aticas 6. Ecuaciones diof´ anticas 7. Fracciones continuas

´ (*) Departamento de Algebra, Facultad de Ciencias Matem´aticas, Universidad Complutense de Madrid, 28040 Madrid, [email protected] 1

0. Preliminares Definici´ on. Denotaremos por Z al conjunto de n´ umeros enteros. Dado a ∈ Z, denotaremos por Z≥a al conjunto de los n´ umeros enteros mayores o iguales que a. De esta forma evitaremos pol´emicas est´eriles sobre si los n´ umeros naturales empiezan con el 0 o con el 1, y escribiremos simplemente Z≥0 o Z≥1 seg´ un nos haga falta. Teorema 0.1 (Principio del buen orden). Sea a un n´ umero entero y sea S ⊂ Z≥a un subconjunto no vac´ıo de Z≥a . Entonces S tiene un elemento m´ınimo. Demostraci´ on: Como S es no vac´ıo, existe b ∈ S. Como S ⊂ Z≥a , entonces a ≤ b. Considerando el conjunto finito {a, a + a, . . . , b − 1, b}, tomamos c el menor elemento de tal conjunto que est´e en S (sabemos que, en el peor de los casos, b lo est´a). Es claro entonces que c es un elemento m´ınimo de S. Teorema 0.2 (Principio de inducci´on). Sea P una propiedad relativa a los n´ umeros de Z≥a . Supongamos que se verifica alguna de las dos siguientes hip´otesis: (i) (Inducci´ on d´ebil): P es cierta para el n´ umero a y cada vez que P es cierta para cada n ∈ Z≥a entonces P es cierta para n + 1. (ii) (Inducci´ on fuerte): P es cierta para el n´ umero a y cada vez que P es cierta para todos los n´ umeros de a a n entonces P es cierta para n + 1. Entonces P es cierta para todo n ∈ Z≥a . Demostraci´ on: Sea S el conjunto de enteros mayores o iguales que a que no verifica la propiedad P . Como estamos suponiendo que a verifica P , entonces a 6∈ S. El teorema estar´a demostrado si demostramos que S es el conjunto vac´ıo. Supongamos entonces, por reducci´on al absurdo, que S es no vac´ıo. Por el principio del buen orden, el conjunto S tendr´a un m´ınimo m. Esto quiere decir que los n´ umeros a, a + 1, . . . , m − 1 no est´an en S, es decir, verifican la propiedad P . Pero tanto (i) como (ii) aplicadas a n = m − 1 implican que entonces m tambi´en verifica P , lo que contradice que m est´e en S.

Ejercicio 0.3. Se considera la sucesi´ on de Fibonacci {un } de n´ umeros enteros 1, 1, 2, 3, 5, 8, 13, 21, . . . definida de forma√recursiva por u1 = u2 = 1, un = un−1 + un−2 . Probar por inducci´ on √    1+ 5 n n 1 1− 5 que un = √5 − . Comprobar la f´ormula para n = 4. 2 2

2

1. Divisibilidad y factorizaci´on en enteros Definici´ on. Se dice que un n´ umero entero a divide a otro n´ umero entero b y se escribe a|b si existe c ∈ Z tal que b = ac. En tal caso, diremos que a es un divisor de b. Demostramos a continuaci´ on las propiedades b´asicas de divisibilidad que necesitaremos. Las demostraciones son extremadamente simples, pero vale la pena hacerlas con rigor al menos una vez, ya que veremos enseguida que hay propiedades aparentemente inmediatas (en el sentido de que son propiedades de sobra conocidas) y que, sin embargo, no pueden hacerse a partir de la definici´on y requieren t´ecnicas m´as sofisticadas. Lema 1.1. Dados enteros a, b, c, se tiene: (i) Si a|b y b 6= 0, entonces |a| ≤ |b|. (ii) c|a y c|b, entonces c|ax + by para todo x, y ∈ Z. (iii) a|b si y s´ olo si a|ac + b. (iv) Si c 6= 0, entonces a|b si y s´ olo si ac|bc. Demostraci´ on: Demostremos primero (i). Si a|b, entonces existe un entero c tal que b = ac. Tomando valores absolutos, se tiene |b| = |a||c|. Como b 6= 0, se tiene c 6= 0, luego |c| ≥ 1. Por tanto, |b| ≥ |a|. Para demostrar (ii), vemos que c|a y c|b implica que existen enteros a0 , b0 tales que a = a0 c y b = b0 c. Por tanto, ax + by = a0 cx + b0 cy = (a0 x + b0 y)c, por lo que c|ax + by. La parte (iii) es consecuencia de (ii). En efecto, si a|b, como obviamente a|a, se tiene a|ac + b · 1 = ac + b. Rec´ıprocamente, si a|ac + b, de nuevo por (ii) se tiene a|(ac + b) · 1 + a(−c) = b. Finalmente, para ver (iv), por definici´on tenemos que a|b si y s´olo si existe un entero d tal que b = ad. Como c 6= 0, esto es equivalente a bc = acd, que de nuevo equivale, por definici´on, a ac|bc. Definici´ on. Un n´ umero primo es un entero p > 1 cuyos u ´nicos divisores positivos son 1 y p. Por el contrario, un n´ umero compuesto es un n´ umero entero n > 1 con alg´ un divisor positivo n1 6= 1, n, y por tanto n = n1 n2 , con n1 , n2 > 1. Empezamos con una aplicaci´ on del principio de inducci´on fuerte: Teorema 1.2 (Fundamental de la aritm´etica: existencia). Todo n´ umero entero n ≥ 2 se puede escribir como producto finito de n´ umeros primos. Demostraci´ on: Lo demostraremos por inducci´on sobre n ∈ Z≥2 . Claramente 2 es primo, luego el teorema se verifica trivialmente para n = 2. Supongamos ahora que tenemos 3

demostrado el teorema para todos los n´ umeros 2, 3, . . . , n y veamos que tambi´en es cierto para n + 1. Si n + 1 fuera un n´ umero primo, de nuevo el resultado ser´ıa trivial. Queda entonces demostrarlo en el caso en que n + 1 no es primo. En tal caso, n + 1 tendr´ıa un divisor positivo a distinto de 1 y n + 1. Es decir, existe b ∈ Z tal que n + 1 = ab. Como n + 1 y a son positivos, tambi´en lo es b. Adem´as, como a 6= 1, n + 1, tambi´en se tiene b 6= 1, n + 1. De aqu´ı se deduce, junto al Lema 1.1(i), que a y b est´an en el conjunto {2, 3, . . . , n} en que sabemos que es cierto el teorema. Por tanto, a y b se escriben como producto finito de n´ umeros primos, de lo que se sigue que tambi´en n + 1 = ab es producto finito de n´ umeros primos, como quer´ıamos demostrar. Observaci´ on 1.3. Obs´ervese que de momento no se puede demostrar que la descomposici´on en primos sea u ´nica. En efecto, si tenemos que n = p1 . . . pr = p01 . . . p0s , no podemos demostrar con la mera definici´on de n´ umero primo que las dos factorizaciones son iguales (salvo el orden de los factores). La estrategia ser´ıa decir que, por ejemplo, p01 divide a n, es decir al producto p1 . . . pr . Pero, aunque nos hayan ense˜ nado que si 0 un n´ umero primo p1 divide a un producto de n´ umeros p1 . . . pr entonces divide a alguno de los factores, no podemos usarlo porque de momento s´olo podemos usar la definici´ on de n´ umero primo, que no implica nada de esto de forma inmediata. S´olo m´as adelante, cuando demostremos el Lema de Euclides (Teorema 1.13) podremos demostrar la unicidad. De momento, usemos la existencia para demostrar otro resultado importante debido a Euclides: Teorema 1.4 (Euclides). Existen infinitos n´ umeros primos. Demostraci´ on: Supongamos, por reducci´on al absurdo, que el conjunto de n´ umeros primos es un conjunto finito {p1 , p2 , . . . , pr }. Sea el n´ umero n = p1 p2 . . . pr + 1. Claramente n ≥ 2 (ya que alg´ un pi es 2), por lo que el Teorema 1.2 implica que n se escribe como producto de n´ umeros primos. En particular, n tiene alg´ un factor primo, que necesariamente ser´ a de la forma pi , para alg´ un i = 1, 2, . . . , r. Es decir, pi |n = p1 p2 . . . pr + 1. Por el Lema 1.1(iii), pi |1, lo que es absurdo. Definici´ on. Se llama m´ aximo com´ un divisor de dos n´ umeros a, b ∈ Z (y lo denotaremos por mcd(a, b)) al mayor divisor positivo de a y b. Dado que esta definici´on no tiene sentido si a = b = 0 (ya que cualquier n´ umero positivo divide a 0), se puede convenir en escribir mcd(0, 0) = 0, puesto que mcd(a, 0) = a para cualquier a 6= 0. Si (a, b) 6= (0, 0), entonces es claro que mcd(a, b) ≥ 1 (por tanto, siempre que demos por descontado que el m´aximo com´ un divisor de dos n´ umeros es distinto de cero es porque excluimos el caso trivial en que ambos n´ umeros son cero). Diremos que dos n´ umeros a y b son primos entre s´ı o coprimos si mcd(a, b) = 1. 4

Como ya hemos observado, con la mera definici´on de primos y divisibilidad no se puede andar muy lejos. Para el caso del m´aximo com´ un divisor, sin saber que la descomposici´ on en producto de n´ umeros primos es u ´nica, no habr´ıa siquiera en principio un modo de calcularlo. Enunciamos en primer lugar los resultados que s´ı pueden hacerse sin m´as que la propia definici´ on. Proposici´ on 1.5. El m´ aximo com´ un divisor verifica las siguientes propiedades: (i) mcd(a, an + b) = mcd(a, b) para cualquier n ∈ Z. (ii) Si d = mcd(a, b), entonces mcd( ad , db ) = 1. (iii) Dado un n´ umero primo p y un entero cualquiera a, entonces o bien p|a o bien mcd(a, p) = 1. Demostraci´ on: Para demostrar la parte (i) bastar´a que los divisores comunes de a y an + b son lo divisores comunes de a y b. Tomemos entonces un divisor d|a y veamos que d|an + b si y s´olo si d|b. En efecto, como d|a, podremos escribir a = da0 . El Lema 1.1(iii) nos dice ahora que d divide a an + b = d(a0 n) + b si y s´olo si d divide a b. Para la parte (ii), si fuera c = mcd( ad , db ) > 1, entonces, como c| ad y c| db , se seguir´ıa cd|a y cd|b. Como cd > d, se llega a contradicci´on. La parte (iii) es inmediata de la definici´on de n´ umero primo, ya que, por ser mcd(a, p) un divisor de p, se tiene que o bien mcd(a, p) = 1 o bien mcd(a, p) = p (en cuyo caso p|a).

El primer resultado importante es el siguiente: Teorema 1.6 (Algoritmo de divisi´on). Dados a, b ∈ Z con b > 0, existen q, r ∈ Z u ´nicos tales que a = qb + r y 0 ≤ r < b. Demostraci´ on: Consideremos el conjunto S = {a − xb | x ∈ Z, a − xb ≥ 0}. Como para x = −|a| se tiene a − xb = a + |a|b ≥ a + |a| ≥ 0, el conjunto S es no vac´ıo. Por el Principio del Buen Orden (usando S ⊂ Z≥0 ) se deduce que S tiene un elemento m´ınimo r = a − qb. Por definici´ on, r ≥ 0. Veamos que tambi´en r < b. En efecto, si fuera r ≥ b, entonces tendr´ıamos 0 ≤ r − b = a − (q + 1)b, luego r − b ser´ıa un elemento de S menor que r, lo que es absurdo. Esto demuestra la existencia de la divisi´on. Para ver la unicidad, supongamos que tenemos dos divisiones distintas qb + r = a = q b + r0 con 0 ≤ r, r0 < b. Entonces se tendr´ıa (q − q 0 )b = r0 − r, es decir, b|r0 − r. Ahora bien, r0 − r < b − 0 = b y r − r0 < b − 0 = b, por lo que |r0 − r| < b = |b|. El Lema 1.1(i) implica entonces r0 − r = 0, de donde se deduce r = r0 y q = q 0 . 0

5

Observaci´ on 1.7. El resultado anterior nos permite clasificar los n´ umeros seg´ un el resto que den al dividir por un n´ umero dado. Por ejemplo, dividiendo por 4, es claro que todo entero es de alguna de las siguientes formas: 4k, 4k + 1, 4k + 2, 4k + 3. Claramente, los n´ umeros de la forma 4k nunca ser´an primos, y el u ´nico n´ umero primo de la forma 4k + 2 es p = 2. Por tanto, seg´ un el Teorema 1.4, habr´a infinitos primos de la forma 4k + 1 o de la forma 4k + 3 (veremos m´as adelante que hay infinitos de cualquiera de las dos formas). Veamos de momento que existen infinitos n´ umeros primos de la forma 4k + 3 (m´as en general, un teorema de Dirichlet asegura que, dados enteros a, b con mcd(a, b) = 1, existen infinitos n´ umeros primos de la forma ak + b; la demostraci´on de este teorema requiere t´ecnicas mucho m´ as avanzadas que las presentes en estas notas, por lo que s´ olo demostraremos casos muy particulares). El lector reconocer´a aqu´ı (y en demostraciones de resultados parecidos) la t´ecnica de la demostraci´on de Euclides para la existencia de infinitos n´ umeros primos (Teorema 1.4). Si suponemos que p1 , p2 , . . . , pr son todos los n´ umeros primos de la forma 4k + 3, consideramos el n´ umero n = 4p1 p2 . . . pr − 1. Como al menos alg´ un pi es 3, se tiene n ≥ 2. Por el Teorema Fundamental de la Aritm´etica (Teorema 1.2), n ser´ a producto finito de n´ umeros primos (necesariamente distintos de 2, porque n es impar). No pueden ser todos de la forma 4k + 1, porque entonces su producto n ser´ıa tambi´en de la forma 4k + 1, mientras que n es de la forma 4k + 3. Por tanto, n es divisible por alg´ un n´ umero primo de la forma 4k + 3, es decir, es divisible por alg´ un p i . Por el Lema 1.1(i), pi | − 1, lo que es absurdo. Teorema 1.8 (B´ezout). Dados a, b ∈ Z, entonces el conjunto {ax + by ∈ Z | x, y ∈ Z} es el conjunto de los m´ ultiplos de d = mcd(a, b). En particular, mcd(a, b) = 1 si y s´ olo si existen x, y ∈ Z tales que ax + by = 1. Demostraci´ on: Podemos suponer que a y b son distintos de cero, ya que en caso contrario el resultado es inmediato. Obs´ervese que, como d|a, b, entonces el Lema 1.1(ii) implica d|ax + by para todo x, y ∈ Z. Basta entonces demostrar que cualquier m´ ultiplo de d se escribe de la forma ax + by, para lo cual es suficiente verlo para d. Consideramos el conjunto S = {ax + by | x, y ∈ Z, ax + by ≥ 1}. Como a2 + b2 > 0, se tiene que S es no vac´ıo, ya que contiene al menos a a2 + b2 . Por el Principio del Buen Orden (viendo S como subconjunto de Z≥1 ), se sigue que S tiene un elemento m´ınimo, que ser´a de la forma d0 = ax + by. Queremos ver que d0 = d, lo que terminar´ıa la demostraci´ on. Veamos primero que d0 |a. Para ello efectuamos la divisi´on eucl´ıdea y tendremos a = qd0 + r, con 0 ≤ r < d0 . Escribimos entonces r = a − qd0 = a − q(ax + by) = a(1 − q) + b(−qy). Como r < d0 y d0 es m´inimo en S, no puede ser que r est´e en S, por lo que la igualdad anterior implica r ≤ 0, luego r = 0 y por tanto d0 |a. De forma an´aloga se demuestra d0 |b. 6

Basta ver, por la definici´ on de m´aximo com´ un m´ ultiplo, que cualquier divisor com´ un positivo de a y b es menor o igual que d0 . Sea entonces c > 0 tal que c|a y c|b. Se tiene entonces, por el Lema 1.1(ii) que c|ax + by = d0 , luego por el Lema 1.1(i) c ≤ d0 , como quer´ıamos. El resultado anterior nos permite obtener un gran n´ umero de propiedades de la divisibilidad que ser´ıan obvias usando factorizaci´on en primos pero que no son obvias usando s´olo la definici´ on de divisibilidad. Proposici´ on 1.9. Dados n´ umeros enteros no nulos a1 , a2 , b, se tiene que mcd(a1 a2 , b) = 1 si y s´olo si mcd(a1 , b) = 1 y mcd(a2 , b) = 1. Demostraci´ on: Es claro, usando s´olo divisibilidad, que si mcd(a1 a2 , b) = 1, entonces a1 y b no pueden tener un divisor com´ un mayor de uno (porque ser´ıa un divisor com´ un de a1 a2 y b), ni tampoco a2 y B pueden tenerlo, por lo que mcd(a1 , b) = 1 y mcd(a2 , b) = 1. Rec´ıprocamente, si mcd(a1 , b) = 1 y mcd(a2 , b) = 1, por el Teorema 1.8 se sigue que existen x1 , y1 , x2 , y2 ∈ Z tales que a1 x1 + by1 = 1 a2 x2 + by2 = 1. Multiplicando ambas expresiones, se obtiene a1 a2 x1 x2 + b(a1 x1 y2 + a2 x2 y1 + by1 y2 ) = 1, lo que implica, de nuevo por el Teorema 1.8, mcd(a1 a2 , b) = 1. Muchas veces que tengamos un resultado como el anterior, v´alido para dos elementos a1 , a2 , ser´ a v´ alido tambi´en para un n´ umero finito de elementos, demostr´andose este hecho por recurrencia (que en esencia es lo mismo que la inducci´on d´ebil). Para ilustrar este hecho, demostramos a continuaci´ on la generalizaci´on de la Proposici´on 1.9 a r factores con el lenguaje de la inducci´ on, mientras que en el Corolario 1.12 haremos lo mismo a partir de la Proposici´ on 1.11, usando en ese caso el m´etodo de inducci´on. Queda al lector el convencerse de que en realidad ambos m´etodos son equivalentes, y escoger cu´al de los dos le resulta m´ as convincente para aplicarlo en casos sucesivos (ya que a partir de ahora enunciaremos sin demostrar resultados de este tipo). Corolario 1.10. mcd(a1 a2 . . . ar , b) = 1 si y s´olo si mcd(ai , b) = 1 para todo i = 1, 2, . . . , r. Demostraci´ on: Si agrupamos a1 a2 . . . ar = (a1 a2 . . . ar−1 )ar , por la Proposici´on 1.9 tendremos que mcd(a1 a2 . . . ar , b) = 1 si y s´olo si mcd(a1 a2 . . . ar−1 , b) = 1 y mcd(ar , b) = 1. 7

Repetimos lo mismo para a1 a2 . . . ar−1 y tendremos ahora que mcd(a1 a2 . . . ar−1 , b) = 1 si y s´olo si mcd(a1 a2 . . . ar−2 , b) = 1 y mcd(ar−1 , b) = 1. Por tanto, mcd(a1 a2 . . . ar , b) = 1 si y s´olo si mcd(a1 a2 . . . ar−2 , b) = 1, mcd(ar−1 , b) = 1 y mcd(ar , b) = 1. Est´a claro que, reiterando el procedimiento, se llega al enunciado. Proposici´ on 1.11. Dados a, b, n ∈ Z tales que mcd(a, b) = 1 y a|n, b|n, se tiene que entonces ab|n. Demostraci´ on: De la hip´ otesis a|n, b|n podemos escribir n = ar = bs, con r, s ∈ Z. Por otra parte, como mcd(a, b) = 1, el Teorema 1.8 implica que existen x, y ∈ Z tales que ax + by = 1. Tenemos entonces n = n · 1 = n(ax + by) = nax + nby = (bs)ax + (ar)by = ab(sx + ry), por lo que ab|n. Corolario 1.12. Sean a1 , a2 , . . . , ar n´ umeros enteros tales que mcd(ai , aj ) = 1 si i 6= j. Entonces, dado un n´ umero entero n tal que ai |n para cada i = 1, 2, . . . , r, se tiene que a1 a2 . . . ar |n. Demostraci´ on: Lo demostramos por inducci´on sobre el n´ umero de factores r. Si r = 2, entonces es la Proposici´ on 1.11. Supongamos entonces, por hip´otesis de inducci´on, que sabemos que, si r n´ umeros coprimos dos a dos dividen a un entero, entonces el producto tambi´en divide a ese n´ umero. Ve´ amoslo ahora para r + 1 n´ umeros coprimos entre s´ı. Suponemos entonces que tenemos a1 , a2 , . . . , ar+1 tales que mcd(ai , aj ) = 1 si i 6= j y supongamos ai |n para cada i = 1, 2, . . . , r + 1. Por hip´otesis de inducci´on, como en particular ai |n para cada i = 1, 2, . . . , r, se tiene a1 a2 . . . ar |n. Adem´as, por el Corolario 1.10, se tiene mcd(a1 a2 . . . ar , ar+1 ) = 1, luego aplicando la Proposici´on 1.11 con a = a1 a2 . . . ar y b = ar+1 , se tiene a1 a2 . . . ar ar+1 |n, como quer´ıamos. Teorema 1.13 (Lema de Euclides). Dados a, b, c ∈ Z tales que a|bc y mcd(a, b) = 1, se tiene que entonces a|c. Demostraci´ on: Como mcd(a, b) = 1, por el Teorema 1.8 existen x, y ∈ Z tales que ax+by = 1. Por tanto c = c · 1 = c(ax + by) = a(cx) + (bc)y. Como a|bc, se sigue que a divide a los dos sumandos de la derecha, luego a|c. Este resultado nos permite demostrar ya la propiedad clave que todos conocemos de los n´ umeros primos. 8

Corolario 1.14. Sea p un n´ umero primo. Si a1 , a2 , . . . , ar son n´ umeros enteros tales que p|a1 a2 . . . ar , entonces existe i = 1, 2, . . . , r tal que p|ai . Demostraci´ on: Supongamos, por reducci´on al absurdo, que, para cada i = 1, 2, . . . , r, ocurriera p 6 |ai . Por el Lema 1.5(iii), se tendr´ıa mcd(p, ai ) = 1, luego, por el Corolario 1.10, mcd(p, a1 a2 , . . . , ar ) = 1. Aplicando el Lema de Euclides con a = p, b = a1 a2 . . . ar y c = 1, se llegar´ıa a p|1, lo que es absurdo. Con esta propiedad podemos demostrar finalmente por completo el teorema fundamental de la aritm´etica (ver Observaci´on 1.3). Teorema 1.15. (Fundamental de la aritm´etica). Todo n´ umero entero n ≥ 2 se puede escribir de forma u ´nica (salvo el orden de los factores) como producto finito de n´ umeros primos. Demostraci´ on: La existencia ya la demostramos en el Teorema 1.2, as´ı que s´olo hay que demostrar la unicidad de la descomposici´on. Supongamos que tenemos dos descomposiciones n = p1 p2 . . . pr = p01 p02 . . . p0s de n en producto de n´ umeros primos. Como p0s |p01 p02 . . . p0s = p1 p2 . . . pr , el Corolario 1.14 implica que ps divide a alg´ un pi . Reordenando los primos 0 p1 , p2 , . . . , pr , podemos suponer ps |pr . Como pr es primo, se sigue pr = p0s . Por tanto, cancelando este factor en la igualdad anterior, tendremos p0s |p01 p02 . . . p0s−1 = p1 p2 . . . pr−1 . Podemos reiterar el proceso demostrando que cada p0j es un pi y cancel´andolos en la igualdad. N´otese que no puede ocurrir que se acaben primero los p0j (es decir r > s), ya que entonces llegar´ıamos a una igualdad p1 p2 . . . pr−s = 1, lo que es imposible. De la misma forma, no se pueden acabar primero los pi (es decir r < s) ya que en tal caso se llegar´ıa al umero de factores es el mismo, y ´estos coinciden absurdo p01 p02 . . . p0s−r = 1. Por tanto, el n´ salvo el orden (adem´ as cada factor primo aparece el mismo n´ umero de veces). Notaci´ on. En general, agruparemos los n´ umeros primos que se repitan en la descoma1 a2 posici´on y escribiremos n = p1 p2 . . . par r , con p1 , p2 , . . . , pr todos distintos entre s´ı. De hecho, mientras no especifiquemos lo contrario, siempre que digamos que n = pa1 1 pa2 2 . . . par r es la factorizaci´ on en primos de n, se entender´a que p1 , p2 , . . . , pr todos distintos entre s´ı. Con los resultados vistos hasta ahora podemos recuperar los m´etodos ya conocidos de calcular el m´ aximo com´ un divisor. Proposici´ on 1.16. Sea a = pa1 1 pa2 2 . . . par r la descomposici´on de un entero a ≥ 2 en producto de n´ umeros primos. Entonces los divisores positivos de a son los n´ umeros de c1 c2 cr la forma p1 p2 . . . pr , con 0 ≤ ci ≤ ai para cada i = 1, 2, . . . , r. Por tanto, si b = pb11 pb22 . . . pbrr , se tendr´ a mcd(a, b) = pc11 pc22 . . . pcrr , con ci = min{ai , bi } para cada i = 1, 2, . . . , r. 9

Demostraci´ on: Es claro que los n´ umeros de la forma pc11 pc22 . . . pcrr , con 0 ≤ ci ≤ ai , son divisores positivos de a. Rec´ıprocamente, sea d un divisor positivo de a. Existir´ a por tanto otro entero q, tambi´en positivo, tal que a = qd. Si q = 1 o d = 1 el resultado es trivial, as´ı que supondremos q, d ≥ 2. Por el Teorema Fundamental de la a0 a0 a0 a00 a00 a00 Aritm´etica podremos escribir q = p01 1 p02 2 . . . p0s s y d = p001 1 p002 2 . . . p00t t . Por tanto, a0 a0 a0 a00 a00 a00 a = p01 1 p02 2 . . . p0s s p001 1 p002 2 . . . p00t t es una descomposici´on de a en factores primos (en que puede haber primos repetidos). Por la unicidad demostrada en el Teorema 1.15, necea00 a00 a00 sariamente los factores de p001 1 p002 2 . . . p00t t son parte de los factores de pa1 1 pa2 2 . . . par r . Esto a00 a00 a00 quiere decir que p001 1 p002 2 . . . p00t t = pc11 pc22 . . . pcrr , con 0 ≤ ci ≤ ai , lo que demuestra el resultado. Corolario 1.17. Sean a, b enteros positivos tales que mcd(a, b) = 1. Entonces: (i) Los divisores positivos de ab son los productos de la forma dd0 , con d|a y d0 |b y d, d0 > 0. (ii) Si ab = cn para alg´ un c ∈ Z≥1 , entonces a = un y b = v n para ciertos u, v ∈ Z. Demostraci´ on: Como mcd(a, b) = 1, en las factorizaciones en factores primos a = pk11 pk22 . . . pkr r b = q1l1 q2l2 . . . qsls se tiene pi 6= qj para todo i = 1, 2, . . . , r y j, 1, 2, . . . , s. Por tanto, la factorizaci´ on en factores primos de ab es ab = pk11 pk22 . . . pkr r q1l1 q2l2 . . . qsls , luego los divisores positivos de ab son, por la Proposici´on 1.16, los n´ umeros de la forma k10 k20 kr0 l10 l20 ls0 p1 p2 . . . pr q1 q2 . . . qs , con 0 ≤ ki0 ≤ ki y 0 ≤ lj0 ≤ lj , es decir, el producto de un divisor positivo de a y un otro de b. Esto demuestra (i). Para la parte (ii), sea p un primo que divide a c. En particular, p|cn = ab = pk11 pk22 . . . pkr r q1l1 q2l2 . . . qsls . Por el Corolario 1.14, se sigue que p es un pi o un qj , luego la k0

k0

k 0 l0 l0

l0

factorizaci´ on en primos de c se escribe de la forma c = p11 p22 . . . pr r q11 q22 . . . qss . Por tanto, nk0 nl0 nl0 nl0 nk0 nk0 pk11 pk22 . . . pkr r q1l1 q2l2 . . . qsls = ab = cn = p1 1 p2 2 . . . pr r q1 1 q2 2 . . . qs s . Por la unicidad del Teorema 1.15, se sigue que ki = nki0 para i = 1, 2, . . . , r y lj = nlj0 para j = 1, 2, . . . , s. Por k0

k0

k0

l0 l0

l0

tanto a = (p11 p22 . . . pr r )n y b = (q11 q22 . . . qss )n . Teorema 1.18. Sean a, b, c enteros positivos tales que an = bn c para alg´ un entero positivo n n. Entonces c = d para alg´ un d ∈ Z. En particular, si c no es la potencia n-´esima de un √ n n´ umero entero, entonces c no es un n´ umero racional. 10

Demostraci´ on: Si a = pk11 pk22 . . . pkr r es la descomposici´on de a en factores primos, entonces la descomposici´ on de an en factores primos ser´a 1 nk2 r . . . pnk an = pnk r . 1 p2

Como b|an , entonces, por la Proposici´on 1.16, la descomposici´on de b en factores primos ser´a de la forma k0 k0 k0 b = p11 p22 . . . pr r con 0 ≤ ki0 ≤ nki para i = 1, 2, . . . , r. De la misma forma, tendremos k00 k00

k00

c = p11 p22 . . . pr r con 0 ≤ ki00 ≤ nki para i = 1, 2, . . . , r. Por tanto, nk10 +k100 nk20 +k200 p2

a n = bn c = p 1

nkr0 +kr00

. . . pr

y, por la unicidad de la descomposici´on en primos, para cada i = 1, 2 . . . , r tendremos nki = nki0 + ki00 . Por tanto, ki00 = n(ki − ki0 ) ≥ 0, es decir, ki − ki00 ≥ 0. Luego el n´ umero k1 −k10 k2 −k20 kr −kr0 n d = p1 p2 . . . pr es un entero positivo que verifica d = c. √ √ Para la u ´ltima parte, si n c fuera racional, se pod´ıa escribir n c = ab , con a, b enteros positivos. Por tanto, an = bn c, y por la parte ya demostrada llegar´ıamos a la contradicci´ on que c debe ser una potencia n-´esima de un n´ umero entero. Para el c´ alculo del m´ aximo com´ un divisor no siempre es pr´actico usar la Proposici´ on 1.16 (que requiere, por ejemplo, saber factorizar los n´ umeros en cuesti´on). Usaremos en cambio muchas veces el m´etodo m´as cl´asico: Teorema 1.19 (Algoritmo de Euclides). Sean a, b n´ umeros enteros con b > 0. Definimos por recurrencia r−1 = a, r0 = b y, supuestos definidos rk−1 , rk con rk 6= 0, definimos rk+1 como el resto de la divisi´ on eucl´ıdea de de rk−1 entre rk . Entonces, existe alg´ un n tal que rn = 0, y se verifica adem´ as que rn−1 = mcd(a, b). Demostraci´ on: De la definici´ on por recurrencia se sigue que 0 ≤ rk+1 < rk para todo k ≥ 0. Por tanto r0 , r1 , . . . forma una sucesi´on estrictamente decreciente de enteros no negativos, por lo que tiene que ocurrir rn = 0 para alg´ un miembro de la sucesi´on. Por otra parte, si para cada k = 0, . . . , n escribimos la divisi´on rk−1 = qk rk + rk+1 , por la Proposici´on 1.5(i) se tendr´ a mcd(rk−1 , rk ) = mcd(qk rk + rk+1 , rk ) = mcd(rk+1 , rk ). Por tanto: mcd(a, b) = mcd(r−1 , r0 ) = mcd(r0 , r1 ) = . . . = mcd(rn−1 , rn ) = mcd(rn−1 , 0) = rn−1 . 11

Ejemplo 1.20. Veamos c´ omo el algoritmo de Euclides sirve no s´olo para calcular el m´aximo com´ un divisor de dos n´ umeros, sino tambi´en para obtenerlo como combinaci´ on lineal de ellos, como indica el Teorema 1.8. por ejemplo, supongamos que queremos calcular mcd(28, 72) y escribirlo en funci´ on de 28 y 72. Efectuamos para ellos las sucesivas divisiones eucl´ıdeas: 72 = 2 · 28 + 16 28 = 1 · 16 + 12 16 = 1 · 12 + 4 12 = 3 · 4 + 0. Hemos llegado por tanto a un resto 0, luego el m´aximo com´ un divisor de 72 y 28 es el resto anterior, que es 4 (que es, por supuesto, el m´aximo com´ un divisor obtenido a partir de la descomposici´ on de 72, 28 en factores primos). Si vamos despejando este 4 desde la pen´ ultima divisi´ on hacia arriba obtendremos: 4 = 16 − 1 · 12 = 16 − 1 · (28 − 1 · 16) = 2 · 16 − 28 = 2(72 − 2 · 28) − 28 = 2 · 72 − 5 · 28, lo que nos da la expresi´ on de 4 como combinaci´on de 72 y 28, confirmando por tanto el Teorema de B´ezout. Teorema 1.21. Dados a, b, c ∈ Z y d = mcd(a, b), la ecuaci´on ax+by = c tiene soluci´ on si y s´olo si d|c. Adem´ as, si x0 , y0 es una soluci´on de la ecuaci´on y , b no son simult´aneamente nulos, el conjunto de soluciones se puede escribir como  b   x = x0 + t d a   y = y0 − t d con t ∈ Z. Demostraci´ on: Que la ecuaci´ on tenga soluci´on si y s´olo si d|c (es decir, c es un m´ ultiplo de d) es una mera reformulaci´ on del Teorema 1.8. Por otra parte, supongamos que tenemos una soluci´on fija x0 , y0 , es decir, c = ax0 +by0 . El hecho de que a, b no son simult´aneamente cero quiere decir que d 6= 0, por lo que podemos dividir por d. Es claro entonces que los pares de la forma (x0 + db t, y0 − ad t) son soluci´on de la ecuaci´on. Rec´ıprocamente, si x, y es una soluci´on de la ecuaci´on, entonces ax + by = ax0 + by0 , que es equivalente a a(x − x0 ) = b(y0 − y) o tambi´en ad (x − x0 ) = db (y0 − y), por lo que 12

a b d | d (y0

− y). Por la Proposici´ on 1.5(ii) se tiene mcd( ad , db ) = 1, luego por el Lema de Euclides (Teorema 1.13) se tiene ad |y0 − y, luego existe t ∈ Z tal que y0 − y = ad t, es decir, y = y0 − ad t. Sustituyendo y por este valor en ad (x − x0 ) = db (y0 − y) se obtiene x = x0 + db t, luego todas las soluciones son de la forma del enunciado.

Ejemplo 1.22. Retomemos el Ejemplo 1.20 para ver c´omo resolver de forma pr´actica una ecuaci´on como la del teorema anterior. Nos planteamos resolver la ecuaci´on 72x + 28y = 8. Como mcd(72, 28) = 4 y 4|8, la ecuaci´on tiene soluci´on. Adem´as, vimos que podemos escribir 4 = 2 · 72 − 5 · 28, por lo que multiplicando por 2 obtenemos que x0 = 4, y0 = −10 es una soluci´ on. Las soluciones de la ecuaci´on ser´an, por tanto, de la forma (

x=

4 + 7t

y = − 10 − 18t con t ∈ Z. Obs´ervese que este proceso puede interpretarse como encontrar todos los puntos de la recta 72x+28y = 8 del plano af´ın que tienen sus dos coordenadas enteras. El Teorema 1.21 nos da una caracterizaci´ on de cu´ando una recta af´ın dada por una ecuaci´on impl´ıcita con coeficientes enteros tiene puntos con coordenadas enteras. Adem´as, en caso afirmativo, los puntos de coordenadas enteras se pueden obtener dando valores enteros a una ecuaci´ on param´etrica (con coeficientes enteros) de la recta.

13

2. Teor´ıa de congruencias Definici´ on. Fijado un n´ umero entero positivo n y dados n´ umeros enteros a, b, diremos que a es congruente con b m´ odulo n, y lo denotaremos con a ≡ b (mod n), si n|a − b. Proposici´ on 2.1. Fijado un entero n > 0 y dados n´ umeros enteros a, b, c, d, se tiene: (i) a ≡ a (mod n). (ii) a ≡ b (mod n) si y s´ olo si b ≡ a (mod n). (iii) Si a ≡ b (mod n) y b ≡ c (mod n), entonces a ≡ c (mod n). (iv) Si a ≡ b (mod n) y c ≡ d (mod n), entonces y a + c ≡ b + d (mod n). (v) Si a ≡ b (mod n) y c ≡ d (mod n),, entonces ac ≡ bd (mod n). (vi) Si ac ≡ bc (mod n), entonces a ≡ b (mod nd ), donde d = mcd(c, n). En particular, si mcd(c, n) = 1, entonces ac ≡ bc (mod n) si y s´olo si a ≡ b (mod n). Demostraci´ on: Es un simple ejercicio que se deja al lector. Hagamos al menos la demostraci´on de la parte (vi). Por definici´on, ac ≡ bc (mod n) quiere decir n|ac−bc = (a−b)c. Dividiendo por d tendremos nd |(a − b) dc . Como mcd( nd , dc ) = 1 (ver Proposici´on 1.5(ii)), por el Lema de Euclides (Teorema 1.13) se sigue nd |(a − b), es decir, a ≡ b (mod nd ). Observaci´ on 2.2. En lenguaje m´as matem´atico, obs´ervese que las tres primeras propiedades de la Proposici´ on 2.1 est´ an diciendo que la relaci´on ≡ (mod n) es una relaci´ on de equivalencia ((i) es la propiedad reflexiva, (ii) es la sim´etrica y (iii) es la transitiva). El ´ cociente por esta relaci´ on es lo que en Algebra B´asica se denota por Zn , y las propiedades (iv) y (v) nos dicen que tal cociente tiene estructura de anillo. N´otese que, por la propiedad (ii), el orden de los elementos en la congruencia es indistinto, por lo que se puede hablar de que a y b son congruentes entre s´ı o no. Una pregunta natural es si todos los enteros son congruentes a n´ umeros sencillos. El siguiente resultado da una respuesta satisfactoria a dicha pregunta: Proposici´ on 2.3. Dado n un n´ umero entero positivo, cada n´ umero entero a es congruente m´odulo n con uno y s´ olo uno de los elementos del conjunto {0, 1, . . . , n − 1}, precisamente al resto de la divisi´ on eucl´ıdea de a entre n. En particular, dos enteros a, b son congruentes m´odulo n si y s´ olo si a y b tienen el mismo resto al dividir entre n. Demostraci´ on: Sea a = qn + r la divisi´on eucl´ıdea de a entre n. Es claro que n|a − r, por lo que a ≡ r (mod n). Por otra parte, si a ≡ s (mod n) para alg´ un s ∈ {0, 1, . . . , n − 1}, entonces n|a − s, luego se podr´ıa escribir a = cn + s, para alg´ un c ∈ Z. Como 0 ≤ s < n, 14

la unicidad de la divisi´ on eucl´ıdea (Teorema 1.6) implica que a = cn + s es precisamente la divisi´on eucl´ıdea de a , por lo que s = r. De hecho, la proposici´ on anterior no hace sino decir en otro lenguaje un resultado que ya conoc´ıamos: que, fijado n, cada n´ umero entero se puede escribir de una sola de las formas nk + r, con r ∈ {0, 1, . . . , n − 1}. Si cambiamos el conjunto {0, 1, . . . , n − 1}, tendremos la siguiente definici´ on. Definici´ on. Se llama sistema completo de restos m´ odulo n a un conjunto R de n´ umeros enteros tales que cada a ∈ Z es congruente m´odulo n a un elemento de R y s´olo a uno. Por ejemplo, {8, −3, 6, −1} ser´ıa un sistema completo de restos m´odulo 4, ya que 8 ≡ 0 (mod 4), −3 ≡ 1 (mod 4), 6 ≡ 2 (mod 4) y −1 ≡ 3 (mod 4). En otras palabras, comparando con la Observaci´ on 1.7, estamos diciendo que cada n´ umero entero es de una y s´olo una de las formas 4k + 8, 4k − 3, 4k + 6, 4k − 1. Una partici´on as´ı puede parecer arbitraria, pero por ejemplo el lector puede darse cuenta ahora de que nuestra demostraci´ on de la existencia de infinitos n´ umeros primos de la forma 4k + 3 utilizaba en realidad los n´ umeros de la forma 4k − 1. Damos a continuaci´ on un m´etodo u ´til para caracterizar sistemas completos de restos m´odulo un n´ umero n (que da en particular, como condici´on necesaria, que deben ser conjuntos de n elementos). Lema 2.4. Sea n un n´ umero entero positivo y sea R = {r1 , r2 , . . . , rm } un conjunto de 0 respectivamente los restos de la divisi´on eucl´ıdea entre n´ umeros enteros. Sean r10 , r20 , . . . , rm n de r1 , r2 , . . . , rm . Entonces son equivalentes: (i) R es un sistema completo de restos m´odulo n. (ii) m ≥ n y para cada i 6= j se tiene ri 6≡ rj (mod n). 0 coinciden, en cierto orden, con 0, 1, . . . , n − 1. (iii) Los n´ umeros r10 , r20 , . . . , rm

Demostraci´ on: Haremos la demostraci´on de forma c´ıclica. (i)⇒(ii): Como cada uno de los n´ umeros 0, 1, . . . , n − 1 son congruentes m´odulo n a alg´ un elemento de R y dos de ellos no son congruentes entre s´ı, necesariamente R debe contener al menos n elementos, es decir, m ≥ n. Adem´ as, si para alg´ un i 6= j fuera ri ≡ rj (mod n), entonces el n´ umero ri ser´ıa simult´aneamente congruente m´ odulo n con ri y rj , contrario a la definici´on de sistema completo. (ii)⇒(iii): Si i 6= j, tenemos por hip´otesis ri 6≡ rj (mod n), luego, por la Proposici´ on 0 0 0 0 0 2.3, se tendr´ a ri 6= rj . Por tanto, los n´ umeros r1 , r2 , . . . , rm son m n´ umeros distintos entre los n n´ umeros 0, 1, . . . , n − 1. Como m ≥ n, ambos sistemas de n´ umeros deben coincidir. 15

(iii)⇒(i): Por la Proposici´ on 2.3, cada n´ umero entero es congruente con uno y s´olo uno 0 . Como, de de los n´ umeros 0, 1, . . . , n − 1, que por hip´otesis son los n´ umeros r10 , r20 , . . . , rm 0 nuevo por la Proposici´ on 2.3, un n´ umero es congruente con ri si y s´olo si es congruente con ri , se concluye que cada n´ umero entero es congruente con uno y s´olo uno de los n´ umeros r1 , r2 , . . . , rm , es decir, R es un sistema completo de restos m´odulo n. A partir del lema, podemos ya dar muchas formas de construir sistemas completos de restos m´odulo n: Proposici´ on 2.5. Dado un entero positivo n, los siguientes conjuntos son sistemas completos de restos m´ odulo n: (i) Cualquier conjunto de n enteros consecutivos. (ii) {ar1 + b, ar2 + b, . . . , arn + b}, donde {r1 , r2 , . . . , rn } es un sistema completo de restos m´odulo n, a es un entero tal que mcd(a, n) = 1 y b es un entero cualquiera. (iii) {b, a + b, 2a + b, . . . , (n − 1)a + b}, donde a es un entero tal que mcd(a, n) = 1 y b es un entero cualquiera. Demostraci´ on: Es todo consecuencia del Lema 2.4(ii). Para el conjunto (i) usamos que tiene n elementos y que claramente no hay dos elementos que sean congruentes entre s´ı. Para el conjunto (ii), usamos que, por ser {r1 , r2 , . . . , rn } es un sistema completo de restos m´odulo n, si i 6= j entonces ri 6≡ rj (mod n). Como mcd(a, n) = 1, se sigue de la Proposici´ on 2.1 que ari 6≡ arj (mod n) y ari + b 6≡ arj + b (mod n). El Lema 2.4(ii) implica entonces que {ar1 + b, ar2 + b, . . . , arn + b} es un sistema completo de restos m´odulo n. Finalmente, (iii) es el caso particular de (ii) en que tomamos {r1 , r2 , . . . , rn } = {0, 1, . . . , n − 1}.

Ejemplo 2.6. Se pueden usar las congruencias para dar criterios de divisibilidad. Recordemos que un n´ umero con expresi´ on decimal ar ar−1 . . . a1 a0 quiere decir el n´ umero n = r r−1 ar · 10 + ar−1 · 10 + . . . + a1 · 10 + a0 . Entonces, por ejemplo: –Como 10 ≡ 1 (mod 3), se sigue de las propiedades del Lema 2.1 que n ≡ ar + ar−1 + . . . + a1 + a0 (mod 3). En particular se obtiene el resultado conocido: un n´ umero es divisible por 3 si y s´ olo si la suma de sus cifras es divisible por 3. –Como tambi´en 10 ≡ 1 (mod 9), se obtiene del mismo modo que un n´ umero es divisible por 9 si y s´ olo si la suma de sus cifras es divisible por 9. M´as a´ un, de la congruencia n ≡ ar + ar−1 + . . . + a1 + a0 (mod 3) se sigue la cl´asica prueba del nueve. Por ejemplo, si queremos multiplicar 53914 por 13518, usamos que 161277 ≡ 1 + 6 + 1 + 2 + 7 + 7 = 24 ≡ 2 + 4 = 6 (mod 9) 16

140263 ≡ 1 + 4 + 0 + 2 + 6 + 3 = 16 ≡ 1 + 6 = 7 (mod 9) por lo que debe ser 161277 · 140263 ≡ 6 · 7 = 42 ≡ 4 + 2 = 6 (mod 9). Por tanto, una forma de comprobar que el producto est´a bien calculado es comprobar del mismo modo que es congruente con 6 m´odulo 9. Y, en efecto, tenemos: 22621195851 ≡ 2 + 2 + 6 + 2 + 1 + 1 + 9 + 5 + 8 + 5 + 1 = 42 ≡ 4 + 2 = 6 (mod 9). Obviamente, si la prueba no sale es que la operaci´on est´a mal hecha, pero el que salga bien la prueba no implica que la operaci´on est´e bien hecha (por ejemplo, 22531195851 hubiera dado el mismo resultado m´ odulo 9). –Si usamos ahora 10 ≡ −1 (mod 11), obtenemos el criterio que un n´ umero es divisible por 11 si y s´ olo si la suma con signo alternado de sus cifras es divisible por 11, ya que r n ≡ (−1) ar + (−1)r−1 ar−1 + . . . − a1 + a0 (mod 11). De hecho, este hecho nos permite inventarnos una “prueba del once”. Siguiendo con el ejmplo anterior, tendremos: 161277 ≡ −1 + 6 − 1 + 2 − 7 + 7 = 6 (mod 11) 140263 ≡ −1 + 4 − 0 + 2 − 6 + 3 = 2 (mod 11) por lo que debe ser 161277 · 140263 ≡ 6 · 2 = 12 ≡ −1 + 2 = 1 (mod 11). En efecto, tenemos 22621195851 ≡ 2 − 2 + 6 − 2 + 1 − 1 + 9 − 5 + 8 − 5 + 1 = 12 ≡ −1 + 2 = 1 (mod 11). Obs´ervese que esta nueva prueba servir´ıa para reforzar la prueba del nueve, ya que ahora 22531195851 ≡ 2 − 2 + 5 − 3 + 1 − 1 + 9 − 5 + 8 − 5 + 1 = 10 6≡ 1 (mod 11), lo que indica que 22531195851 no puede ser el producto. Muchas veces es m´ as sencillo reducir una congruencia m´odulo un entero n a congruencias con m´ odulos m´ as peque˜ nos que n. El siguiente resultado explica c´omo: Proposici´ on 2.7. Sean n1 , n2 , . . . , nr n´ umeros naturales tales que mcd(ni , nj ) = 1 si i 6= j. Entonces, para cada par de enteros a, b se tiene que a ≡ b (mod n1 n2 . . . nr ) si y s´ olo si a ≡ b (mod n1 ), a ≡ b (mod n2 ), . . . , a ≡ b (mod nr ). Demostraci´ on: Por definici´ on, a ≡ b (mod n1 n2 . . . nr ) si y s´olo si n1 n2 . . . nr |a − b. Como mcd(ni , nj ) = 1 si i 6= j, el Corolario 1.12 implica que n1 n2 . . . nr |a − b es equivalente a que, para cada i = 1, 2, . . . , r, se tenga ni |a − b, es decir, a ≡ b (mod ni ). Si en cambio queremos pasar de una congruencia m´odulo n a congruencias m´odulo un u ´nico divisor de n, entonces el resultado es m´as complicado: 17

Proposici´ on 2.8. Si d|n entonces a ≡ b (mod nd ) si y s´olo si a ≡ b + i nd (mod n) para alg´ un i ∈ {0, 1, . . . , d − 1}. Adem´ as, los n´ umeros b, b + nd , . . . , b + (d − 1) nd son dos a dos incongruentes m´ odulo n. Demostraci´ on: Por una parte, es claro que, si a ≡ b + i nd (mod n), entonces existe c ∈ Z tal que a − b − i nd = cn. Por tanto, a = b + nd (i + cd), luego a ≡ b (mod nd ). Rec´ıprocamente, si a ≡ b (mod divisi´on eucl´ıdea de t entre d:

n d ),

existe t ∈ Z tal que a = b + nd t. Consideramos la

t = qd + r con 0 ≤ r < d. Por tanto, podremos escribir a=b+

n n n t = b + (qd + r) = b + nq + r . d d d

De aqu´ı se deduce a ≡ b + r nd (mod n) y, como 0 ≤ r < d, es una de las d congruencias que quer´ıamos. Finalmente, los n´ umeros b, b + nd , . . . , b + (d − 1) nd son dos a dos incongruentes m´ odulo n porque son un subconjunto de los n´ umeros b, b + 1, . . . , b + (n − 1), que es un sistema completo de restos m´ odulo n (Proposici´on 2.5(i)). Con el lenguaje de congruencias, podemos reinterpretar el Teorema 1.21 del siguiente modo: Proposici´ on 2.9. Sean a, b, n numeros enteros con n > 0 y a 6= 0, y sea d = mcd(a, n). Entonces la congruencia ax ≡ b (mod n) tiene soluci´on si y s´olo si d|b. Adem´as, en este caso hay exactamente d soluciones m´odulo n. Demostraci´ on: La congruencia tiene soluci´on si y s´olo si existe x ∈ Z tal que n|b − ax, es decir, si y s´ olo si existen x, y ∈ Z tales que b − ax = ny. Como el Teorema 1.21 afirma que la ecuaci´on ax + ny = b tiene soluci´on si y s´olo si d|b, la primera parte queda demostrada. Adem´as, el mismo Teorema 1.21 dice que las soluciones de la ecuaci´on dependen de un par´ametro t y son de la forma  n  x = x0 + t d a  y = y − t. 0 d Entonces las soluciones de la congruencia son los enteros x de la forma x ≡ x0 (mod nd ) que, por la Proposici´ on 2.8, da d soluciones m´odulo n. Ejemplo 2.10. Consideremos la congruencia lineal 72x ≡ 8 (mod 28). Como indica la demostraci´ on de la Proposici´ on 2.9, para resolverla, debemos resolver primero la ecuaci´ on 18

72x + 28y = 8, que ya resolvimos en el Ejemplo 1.22, y cuya soluci´on era ( x= 4 + 7t y = − 10 − 18t. Por tanto, la soluci´ on a nuestra congruencia es x = 4+7t, es decir x ≡ 4 (mod 7) o, escrito m´odulo 28, x ≡ 4, 11, 18, 25 (mod 28). La Proposici´ on 2.9 nos dice que toda congruencia de la forma ax ≡ b (mod n) es equivalente a varias congruencias de la forma x ≡ x0 (mod n) (o en realidad a una congruencia de la forma x ≡ x0 (mod nd )). Por tanto, si queremos resolver varias congruencias lineales al mismo tiempo basta resolver un sistema de congruencias todas del tipo x ≡ x0 (mod n). El siguiente teorema nos dice c´ omo resolver tales sistemas. El inter´es del resultado no ser´ a s´olo el enunciado te´ orico (existencia de soluci´on bajo ciertas hip´otesis), sino tambi´en la demostraci´ on, que da un m´etodo efectivo de resoluci´on. Teorema 2.11 (chino del resto). Sean n1 , n2 , . . . , nr n´ umeros enteros positivos tales que mcd(ni , nj ) = 1 si i 6= j. Entonces cualquier sistema de congruencias  x ≡ a1 (mod n1 )       x ≡ a2 (mod n2 ) ..   .     x ≡ ar (mod nr ) tiene soluci´ on, que adem´ as es u ´nica m´odulo n1 n2 . . . nr . Demostraci´ on: Para cada i = 1, 2, . . . , r definimos Ni =

n1 n2 . . . nr = n1 n2 . . . ni−1 ni+1 . . . nr . ni

Por el Corolario 1.10 se tendr´ a mcd(ni , Ni ) = 1, ya que mcd(ni , nj ) = 1 para cada factor nj de Ni . Por tanto, por la Proposici´on 2.9, existe xi ∈ Z tal que Ni xi ≡ ai (mod ni ), ya que obviamente mcd(ni , Ni )|ai . Veamos que x0 = N1 x1 + N2 x2 + . . . + Nr xr es una soluci´ on del sistema de congruencias. En efecto, para cada i = 1, 2, . . . , r se tiene Nj ≡ 0 (mod ni ) si i 6= j, luego x0 ≡ Ni xi ≡ ai (mod ni ). Para la unicidad, observemos en primer lugar que, si x ≡ x0 (mod n1 n2 . . . nr ), es claro que x es tambi´en soluci´ on del sistema de congruencias. Rec´ıprocamente, sea x una 19

soluci´on del sistema. Entonces x ≡ x0 (mod ni ) para cada i = 1, 2, . . . , r, luego, por la Proposici´on 2.7, x ≡ x0 (mod n1 n2 . . . nr ).

Ejemplo 2.12. Veamos que el resultado anterior se puede usar incluso si los ni no son todos primos entre s´ı (con la diferencia de que entonces puede ocurrir que el sistema de congruencias no tenga soluci´ on). Supongamos por ejemplo que queremos resolver el sistema de congruencias  x ≡ 5 (mod 24)   x ≡ 1 (mod 28)   x ≡ − 4 (mod 15) Evidentemente, no estamos en las condiciones del Teorema Chino del Resto, ya que, por ejemplo, mcd(24, 28) = 4. Sin embargo, podemos usar la Proposici´on 2.7 para sustituir nuestro sistema por el sistema equivalente:    x ≡ 5 (mod 8)     x ≡ 5 (mod 3)      x ≡ 1 (mod 4)  x ≡ 1 (mod 7)       x ≡ − 4 (mod 3)     x ≡ − 4 (mod 5). Por supuesto, seguimos sin estar en las hip´otesis del Teorema Chino del Resto, pero ahora vamos a poder quitar las congruencias que nos sobran. En primer lugar, tenemos las congruencias x ≡ 5 (mod 3) y x ≡ −4 (mod 3), que son equivalentes, ya que 5 ≡ −4 (mod 3), as´ı que podemos eliminar una de ellas. A´ un tenemos el par de congruencias x ≡ 5 (mod 8) y x ≡ 1 (mod 4) que no nos permiten aplicar el Teorema Chino del Resto. En este caso, observamos que x ≡ 5 (mod 8) implica obviamente x ≡ 5 (mod 4), que es equivalente a x ≡ 1 (mod 4), porque 5 ≡ 1 (mod 4). As´ı pues, podemos eliminar la congruencia x ≡ 1 (mod 4) y tenemos ya el sistema equivalente de congruencias:  x ≡ 5 (mod 8)      x ≡ 5 (mod 3)  x ≡ 1 (mod 7)     x ≡ − 4 (mod 5) que ya est´ a en las hip´ otesis del Teorema Chino del Resto, luego ya sabemos que tiene soluci´on (vale la pena hacer aqu´ı una pausa para observar que podr´ıa haber ocurrido que, 20

en lugar de encontrar congruencias equivalentes m´odulo 4, hubi´eramos encontrado congruencias incompatibles; esto hubiera querido decir que el sistema de partida es incompatible y no tiene soluci´ on). Aplicamos ahora el m´etodo de resoluci´on que nos da el Teorema Chino del Resto, es decir, resolvemos primero (con el m´etodo explicado en el Ejemplo 2.10) las congruencias: • 105x ≡ 5(mod 8), que tiene como soluci´on x ≡ 5(mod 8). • 280x ≡ 5(mod 3), que tiene como soluci´on x ≡ 2 (mod 3). • 120x ≡ 1(mod 7), que tiene como soluci´on x ≡ 1 (mod 7). • 168x ≡ −4(mod 5), que tiene como soluci´on x ≡ 2 (mod 5). La soluci´on final queda, por tanto x ≡ 105 · 5 + 280 · 2 + 120 · 1 + 168 · 2 = 1541 ≡ 701 (mod 840). Como nos indica el ejemplo anterior, para estudiar congruencias, basta estudiar congruencias m´ odulo la potencia de un primo. El caso m´as simple es el de congruencias m´odulo un primo, sobre el que se pueden demostrar toda una serie de resultados. El primero de todos es el siguiente: Teorema 2.13 (Fermat). Sea p un n´ umero primo. Entonces, para cada a ∈ Z no divisible p−1 por p se tiene a ≡ 1 (mod p). Como consecuencia, ap ≡ a (mod p) para cualquier a ∈ Z. Demostraci´ on: Por la Proposici´ on 2.5(iii), sabemos que, si mcd(a, p) = 1, los n´ umeros 0, a, 2a, . . . , (p − 1)a forman un sistema completo de restos m´odulo p. Por el Lema 2.4, esto quiere decir que sus restos al dividir por p son, en cierto orden, 0, 1, 2, . . . , p − 1. Como obviamente el cero del primer conjunto se corresponde con el cero del segundo, se tiene que los n´ umeros a, 2a, . . . , (p−1)a son congruentes m´odulo p, en cierto orden, con 1, 2, . . . , p−1. Multiplicando todas las congruencias tendremos (p − 1)!ap−1 ≡ (p − 1)! (mod p). Adem´ as, como p es un n´ umero primo, es coprimo con todos los factores 1, 2, . . . , p − 1 de (p − 1)!, luego (Corolario 1.10) mcd((p − 1)!, p) = 1. Se sigue entonces de la Proposici´on 2.1(vi) que ap−1 ≡ 1 (mod p). La congruencia anterior implica autom´aticamente ap ≡ a (mod p), siempre bajo la hip´otesis mcd(a, p) = 1. Si en cambio p|a, entonces tanto ap como a son congruentes con cero m´odulo p, luego la congruencia ap ≡ a (mod p) es tambi´en v´alida en este caso. La propiedad del Teorema de Fermat no es caracter´ıstica de los n´ umeros primos, ya que hay n´ umeros compuestos que satisfacen lo mismo. 21

Definici´ on. Se llama n´ umero pseudoprimo en base a a un n´ umero compuesto n tal que an ≡ a (mod n). Se llama simplemente n´ umero pseudoprimo a un n´ umero compuesto n tal n que 2 ≡ 2 (mod n). Finalmente, se llama pseudoprimo absoluto o n´ umero de Carmichael n a un n´ umero compuesto n tal que a ≡ a (mod n) para cualquier n´ umero entero a. Obviamente son mucho m´ as interesantes (y m´as dif´ıciles de encontrar) los n´ umeros de Carmichael que los pseudoprimos (aunque los matem´aticos chinos de hace 25 siglos pensaban que no hab´ıa n´ umeros pseudoprimos). De ambos se sabe que hay infinitos de ellos. En el caso de pseudoprimos, no cuesta mucho ver que, dado un n´ umero pseudoprimo impar n n, entonces 2 − 1 es tambi´en un n´ umero pseudoprimo, lo que permite construir infinitos pseudoprimos (basta empezar por 341 = 11 · 31, que es el primer n´ umero pseudoprimo). Para n´ umeros de Carmichael, nos limitaremos a dar el siguiente resultado: Proposici´ on 2.14. Sea n = p1 p2 . . . pr un producto de r primos distintos, con r > 1. Si para cada i = 1, 2, . . . , r se tiene pi − 1|n − 1, entonces n es un n´ umero de Carmichael. Demostraci´ on: Claramente n es compuesto, porque r > 1, as´ı que hay que demostrar que n a ≡ a (mod p1 p2 . . . pr ) para todo entero a. Por la Proposici´on 2.7, hay que demostrar que, para cada i = 1, 2, . . . , r, se tiene an ≡ a(mod pi ). Obviamente esto es cierto si pi |a, as´ı que podemos suponer mcd(a, pi ) = 1. En tal caso, por el Teorema de Fermat se tendr´ a pi −1 n−1 a ≡ 1 (mod pi ) y, como por hip´otesis pi −1|n−1, se tendr´a tambi´en a ≡ 1(mod pi ), n lo que implica a ≡ a(mod pi ). Ejemplo 2.15. Con la proposici´on anterior, se demuestra que 561 = 3 · 11 · 17 es un n´ umero de Carmichael (de hecho es el primero de todos). Otros ejemplos obtenidos de este modo son los n´ umeros 1729 = 7 · 13 · 19, 6601 = 7 · 23 · 41 y 10585 = 5 · 29 · 73. Aunque no verifiquen el critero de la proposici´on anterior, tambi´en son n´ umeros de Carmichael 1105 = 5 · 13 · 17, 2821 = 7 · 13 · 31, 15841 = 7 · 31 · 73 y 16046641 = 13 · 37 · 73 · 457 (uno de los pocos conocidos con cuatro factores, descubierto por el propio Carmichael). S´ olo en 1994 se demostr´ o que existen infinitos n´ umeros de Carmichael. Son tan escasos que s´ olo hay 43 de ellos menores que un mill´on. Ejemplo 2.16. Si consideramos el conjunto de restos m´odulo 11, observamos que tenemos los pares disjuntos {2, 6}, {3, 4}, {5, 9}, {7, 8} con la propiedad de que el producto de los elementos de un par es congruente con 1 m´odulo 11. Adem´as, la uni´on de todos los pares da el conjunto {2, 3, 4, 5, 6, 7, 8, 9}, por lo que tendremos 2 · 3 · 4 · 5 · 6 · 7 · 8 · 9 = (2 · 6)(3 · 4)(5 · 9)(7 · 8) ≡ 1 (mod 11). Esto no es casualidad, y para cada n´ umero primo tenemos la misma situaci´on, lo que nos permite demostrar el siguiente teorema. 22

Teorema 2.17 (Wilson). Si p es un n´ umero primo, entonces (p − 1)! ≡ −1 (mod p). Demostraci´ on: Suponemos p > 3, ya que los casos p = 2, 3 se pueden comprobar trivialmente a mano. Para cada a = 1, 2 . . . , p − 1, consideramos la congruencia ax ≡ 1 (mod p) que, por la Proposici´ on 2.9 tendr´a exactamente una soluci´on m´odulo p, luego se podr´ a 0 representar exactamente por un n´ umero a ∈ {1, 2, . . . , p − 1} (claramente, no puede ser 0 a = 0). Por tanto, tenemos una aplicaci´on {1, 2, . . . , p − 1} → {1, 2, . . . , p − 1} que asocia a cada a el elemento a0 . Tal aplicaci´on es necesariamente biyectiva, al tener ambos conjuntos el mismo n´ umero de elementos. Obs´ervese que, si a0 = a, entonces a2 ≡ 1 (mod p), es decir p|a2 − 1 = (a + 1)(a − 1). Por el Corolario 1.14, se tiene p|a + 1 o bien p|a − 1. Como a ∈ {1, 2, . . . , p − 1}, se sigue a = 1 o bien a = p − 1. Por tanto, el conjunto {2, 3, . . . , p − 2} se descompone, como en el Ejemplo 2.16, en pares {a, a0 } con la propiedad aa0 ≡ 1 (mod p). Agrupando seg´ un estos pares el producto de todos los elementos de {2, 3, . . . , p − 2}, se llega a 2 · 3 . . . (p − 2) ≡ 1 (mod p) y, por tanto, (p − 1)! = 1 · (2 · 3 . . . (p − 2))(p − 1) ≡ 1 · 1 · (p − 1) ≡ −1 (mod p), como quer´ıamos.

Observaci´ on 2.18. N´ otese que, al contrario que pasa con el teorema de Fermat, la propiedad anterior s´ı que caracteriza a los n´ umeros primos. En efecto, dado un n´ umero compuesto n, si d 6= 1, n es un divisor suyo, se tendr´a que d es uno de los factores de (n − 1)!, por lo que (n − 1)! ≡ 0 (mod d). Por tanto, no puede ser (n − 1)! ≡ −1 (mod n). En realidad, no es dif´ıcil demostrar que, si n > 4 es un n´ umero compuesto, entonces (n − 1)! ≡ 0 (mod n). El Teorema de Wilson tiene el siguiente corolario inesperado: Teorema 2.19. Si p es primo, la congruencia x2 ≡ −1 (mod p) tiene soluci´on si y s´ olo si p = 2 o bien p ≡ 1 (mod 4). Demostraci´ on: Supongamos primero que la congruencia tiene una soluci´on, que llamaremos a. Obviamente, a no puede ser divisible por p, porque en tal caso a2 ≡ 0 (mod p). Para ver que p = 2 o bien p ≡ 1 (mod 4) basta demostrar que p 6≡ 3 (mod 4). En efecto, si fuera p ≡ 3 (mod 4), entonces p−1 ser´ıa un n´ umero impar, y como a2 ≡ −1 (mod p), 2 entonces ap−1 = (a2 )

p−1 2

≡ (−1)

p−1 2

(mod p) = −1 (mod p),

mientras que el Teorema de Fermat (Teorema 2.13), implica ap−1 ≡ 1 (mod p), lo que da una contradicci´ on. Rec´ıprocamente, supongamos p = 2 o bien p ≡ 1 (mod 4) y veamos que la congruencia tiene soluci´ on. Es claro que x2 ≡ −1 (mod 2) tiene soluci´on (de hecho, cualquier x impar 23

lo es), as´ı que podemos limitarnos al caso p ≡ 1 (mod 4). En tal caso, tenemos que es un n´ umero par. Esto quiere decir que el n´ umero de congruencias (triviales)

p−1 2

1 ≡ −(p − 1) (mod p) 2 ≡ −(p − 2) (mod p) .. . p−1 p+1 ≡− (mod p) 2 2 es par. Por tanto, multiplic´ andolas todas tendremos p−1 p+1 )! ≡ (p − 1)(p − 2) . . . (mod p) 2 2 de donde se deduce, usando el Teorema de Wilson para la u ´ltima congruencia, p − 1 2 p+1 p−1 ( )! ≡ (p − 1)(p − 2) . . . ( )! = (p − 1)! ≡ −1 (mod p). 2 2 2 (

Por tanto, x = ( p−1 on de x2 ≡ −1 (mod p). 2 )! es una soluci´

Corolario 2.20. Existen infinitos n´ umeros primos de la forma 4k + 1. Demostraci´ on: Como siempre, supongamos por reducci´on al absurdo que p1 , p2 , . . . , pr sean todos los n´ umeros primos de la forma 4k + 1. Consideramos entonces el n´ umero 2 a = (2p1 p2 . . . pr ) + 1 y sea p un divisor primo de a (que obviamente existe, por ser a > 4. Entonces, 2p1 p2 . . . pr es una soluci´on de la congruencia x2 ≡ −1 (mod p), por lo que el Teorema 2.19 implica que p = 2 o bien p es de la forma 4k + 1, es decir, es uno de los p1 , p2 , . . . , pr . Como claramente ni 2 ni p1 , p2 , . . . , pr son divisores de a, se llega la contradicci´ on buscada. Podemos mejorar el resultado anterior de la misma forma que hicimos en la Observaci´on 1.7 con el Teorema de Euclides: Corolario 2.21. Existen infinitos n´ umeros primos de la forma 8k + 5. Demostraci´ on: Supongamos que p1 , p2 , . . . , pr sean todos los n´ umeros primos de la forma 2 8k + 5, y consideramos de nuevo a = (2p1 p2 . . . pr ) + 1. Ya hemos observado en la demostraci´ on anterior que los divisores primos de a son de la forma 4k + 1. Por tanto, son de la forma 8k + 1 o bien 8k + 5. Ahora bien, como cada (p1 p2 . . . pr )2 es impar, es decir, de la forma 2k + 1, entonces a = 4(p1 p2 . . . pr )2 + 1 es de la forma 8k + 5. Por tanto, no todos los divisores primos de a son de la forma 8k + 1, por lo que a es divisible por alg´ un primo de la forma 8k + 5, es decir, por alg´ un pi , lo que es imposible.

24

3. Funciones aritm´eticas Vamos a intentar extender ahora el Teorema de Fermat (Teorema 2.13) cuando el exponente es un n´ umero arbitrario n no necesariamente primo. Mirando a la demostraci´ on, vamos a necesitar multiplicar s´ olo los restos m´odulo n que sean coprimos con n (pues si no, no podr´ıamos cancelarlos luego). Ahora bien, ¿cu´antos restos hay coprimos con n? Claramente, esto depender´ a de n, es decir, ser´a una funci´on, a la que de momento damos un nombre (nuestro objetivo en este cap´ıtulo ser´a estudiar funciones de este tipo): Definici´ on. Se llama funci´ on φ de Euler a la funci´on φ : Z≥1 → Z que asocia a cada entero n ≥ 1 el n´ umero φ(n) de enteros r ∈ {1, 2, . . . , n} tales que mcd(r, n) = 1. N´ otese que, como ni r = 0 ni r = n verifican mcd(r, n) = 1, se puede definir tambi´en φ(n) como el n´ umero de enteros r ∈ {0, 1, . . . , n − 1} tales que mcd(r, n) = 1. Con esta definici´ on, la generalizaci´on del Lema 2.4 (en realidad s´olo la implicaci´ on que nos intereresa) es la siguiente: Lema 3.1. Sea n un n´ umero entero positivo y sean r1 , r2 , . . . , rm n´ umeros enteros enteros tales que, m ≥ φ(n), para todo i = 1, 2, . . . , m se tiene mcd(ri , n) = 1 y mcd(ri , rj ) = 1 si j 6= i. Entonces, los restos de la divisi´on eucl´ıdea de r1 , r2 , . . . , rm entre n son todos distintos y coinciden, en cierto orden, con los n´ umeros entre 0 y n que son coprimos con n. Demostraci´ on: Consideramos en primer lugar los n´ umeros s1 , s2 , . . . , sr ∈ {0, 1, . . . , n − 1} tales que mcd(sj , n) 6= 1 para j = 1, 2, . . . , r. Obs´ervese que, por definici´on, r = n − φ(n). Como s1 , s2 , . . . , sr forman parte de un sistema completo de restos m´odulo n, son incongruentes dos a dos m´ odulo n. Adem´as, ning´ un ri es congruente m´odulo n con ning´ un sj , ya que eso querr´ıa decir que mcd(ri , n) = mcd(sj , n), contra nuestra hip´otesis. Por tanto, por el Lema 2.4, como m + r = m + n − φ(m) ≥ n, se tiene que los restos de la divisi´on de r1 , r2 , . . . , rm , s1 , s2 , . . . , sr entre n son todos distintos y coinciden, en cierto orden con 0, 1, . . . , n − 1. Como los restos de s1 , s2 , . . . , sr son ellos mismos, el resultado queda demostrado. Con estas herramientas, podemos proceder ya a generalizar el Teorema de Fermat: Teorema 3.2 (Euler). Dado un n´ umero entero n ≥ 2, entonces para cada n´ umero entero φ(n) a tal que mcd(a, n) = 1 se tiene a ≡ 1 (mod n). Demostraci´ on: Sean r1 , . . . , rφ(n) los enteros positivos menores que n tales que mcd(ri , n) = 1. Como mcd(a, n) = 1, la Proposici´on 1.9 implica que mcd(ari , n) = 1 para todo i = 1, 2, . . . , φ(n) y, si j 6= i, la Proposici´on 2.1(vi) implica ari 6≡ arj (mod n). Por el Lema 25

3.1, los n´ umeros ar1 , . . . , arφ(n) son congruentes, en alg´ un orden, con r1 , . . . , rφ(n) m´ odulo n. Por tanto, haciendo el producto de todos ellos tendremos aφ(n) r1 . . . rφ(n) ≡ r1 . . . rφ(n) (mod n). Como mcd(r1 . . . rφ(n) , n) = 1 (por la Proposici´on 1.9), la Proposici´on 2.1(vi) implica aφ(n) ≡ 1 (mod n). Como consecuencia, obtenemos un m´etodo pr´actico para resolver una congruencia lineal: Corolario 3.3. Si mcd(a, n) = 1, la congruencia lineal ax ≡ b (mod n) es equivalente a x ≡ aφ(n)−1 b (mod n). Demostraci´ on: Por la Proposici´ on 2.9), la congruencia tiene una u ´nica soluci´on, as´ı que φ(n)−1 basta ver que a b es una soluci´ on. Pero esto es inmediato, ya que a(aφ(n)−1 b) = aφ(n) b es, por el Teorema de Euler, congruente con b m´odulo n.

Observaci´ on 3.4. Claramente, el Teorema de Euler es una generalizaci´on del peque˜ no teorema de Fermat, ya que si p es un n´ umero primo es claro que φ(p) = p − 1 (puesto que 1, 2, . . . , p − 1 son los n´ umeros coprimos con p y menores que ´el). M´as en general, si k n = p , con p n´ umero primo, es claro que los enteros positivos menores o iguales que n que no son primos con n son los m´ ultiplos de p, es decir, 1 · p, 2 · p, . . . , pk−1 · p. Como hay pk−1 de ellos, se sigue entonces que 1 φ(pk ) = pk − pk−1 = pk−1 (p − 1) = pk (1 − ). p Calcular φ(n) para un n arbitrario no parece a priori f´acil. Sin embargo, tenemos la siguiente propiedad que resuelve el problema. Teorema 3.5. Si mcd(m, n) = 1, entonces φ(mn) = φ(m)φ(n). Demostraci´ on: Escribimos los primeros mn n´ umeros en forma de rect´angulo n × m: m−1 2m − 1 .. .

1 m+1 .. .

2 m+2 .. .

... ...

i m+i .. .

... ...

jm + 1 .. .

jm + 2 .. .

...

jm + i .. .

. . . jm + m − i .. .

(n − 1)m + 1

(n − 1)m + 2

...

(n − 1)m + i 26

...

mn − 1

m 2m .. . (j + 1)m .. . mn

es decir, escribimos cada n´ umero como jm + i, con i = 1, 2, . . . , m y j = 0, 1, . . . , n − 1. Por la Proposici´ on 1.9, un n´ umero es primo con mn si y s´olo si es primo con m y con n, luego φ(mn) ser´ a el n´ umero de elementos jm + i que son primos con m y con n. La primera observaci´on es que (por la Proposici´on 1.5(i)) mcd(jm + i, m) = mcd(i, m), luego el que un n´ umero jm + i sea primo con m depende s´olo de la columna i-´esima en que est´a. Por la propia definici´ on de φ, tendremos exactamente φ(m) valores de i tales que mcd(i, m) = 1, es decir, que los elementos de la tabla que son primos con m son los elementos de φ(m) columnas. Queda entonces ver cu´ antos elementos en cada columna de las φ(n) anteriores son adem´as primos con n. De hecho, el teorema quedar´a demostrado si vemos que cada una de esas columnas tiene exactamente φ(n) elementos primos con n. De hecho, cualquier columna de la matriz tiene exactamente φ(n) elementos primos con n. En efecto, los elementos de una columna son de la forma i, m + i, 2m + i, . . . (n − 1)m + i que, por la Proposici´on 2.5(iii), forman un sistema completo de restos m´odulo n. Por tanto, son congruentes, aunque posiblemente en otro orden, a 0, 1, 2, . . . , n−1, por lo que hay exactamente φ(n) que sean coprimos con n. Corolario 3.6. Si n = pk11 pk22 . . . pkr r es la descomposici´on de n en factores primos, entonces φ(n) = pk11 −1 pk22 −1 . . . pkr r −1 (p1 −1)(p2 −1) . . . (pr −1) = n(1− p11 )(1− p12 ) . . . (1− p1r ). Demostraci´ on: Es consecuencia inmediata de que, aplicando reiteradamente el Teorema 3.5, se tiene φ(pk11 pk22 . . . pkr r ) = φ(pk11 )φ(pk22 ) . . . φ(pkr r ), y por otra parte, como vimos en la Observaci´ on 3.4, para cada i = 1, 2, . . . , r se tiene φ(pki i ) = pki i −1 (pi − 1). Ejemplo 3.7. Aunque el teorema de Euler sea una generalizaci´on del de Fermat, normalmente con este u ´ltimo se puedun obtener resultados mejores. Por ejemplo, para estudiar cu´ando una potencia es congruente con 1 m´odulo 35, el teorema de Euler dir´ıa que, como φ(35) = φ(5 · 7) = φ(5)φ(7) = (5 − 1)(7 − 1) = 24, entonces a24 ≡ 1 (mod 35) si mcd(a, 35) = 1. Sin embargo, como mcd(a, 35) = 1 es equivalente a mcd(a, 5) = 1 = mcd(a, 7) (ver Proposici´on 1.9), por el teorema de Fermat se tiene a4 ≡ 1 (mod 5) y a6 ≡ 1 (mod 7). Elevando respectivamente al cubo y al cuadrado se tendr´a a12 ≡ 1 (mod 5) 27

y a12 ≡ 1 (mod 7), de donde se deduce a12 ≡ 1 (mod 35). Definici´ on. Una funci´ on aritm´etica es una funci´on f : Z≥1 → Z. Se dice adem´as que es multiplicativa si f (mn) = f (m)f (n) cuando mcd(m, n) = 1. Ejemplo 3.8. plicativas:

De forma trivial se pueden construir varias funciones aritm´eticas multi-

1) La funci´ on id´enticamente cero es claramente multiplicativa. 2) Si una funci´ on multiplicativa f no es id´enticamente cero, existe n ∈ Z≥1 tal que f (n) 6= 0. Como se tiene f (n) = f (n · 1) = f (n)f (1), se sigue entonces f (1) = 1. Esto sugiere otro segundo ejemplo trivial de funci´ on aritm´etica multiplicativa:  f (n) =

1 0

si n = 1 si n > 1

3) De la misma forma, la funci´on id´enticamente uno es claramente multiplicativa. 4) La funci´ on identidad id(n) = n es tambi´en claramente multiplicativa. 5) M´as en general, aunque no vayamos a usarla, para cada k ≥ 0, la funci´on f (n) = nk es multiplicativa. Todos los ejemplos anteriores son “mucho m´as multiplicativos” que la funci´on φ, ya que f (mn) = f (m)f (n) independientemente de que m y n tengan factores en com´ un o no. El siguiente resultado nos dar´ a un modo de calcular funciones multiplicativas “m´as serias” a partir de otras funciones multiplicativas, aunque sean triviales: Teorema 3.9. Si f es una funci´ on aritm´etica multiplicativa, entonces la funci´on F (n) =

X

f (d)

d|n

es tambi´en una funci´ on multiplicativa. Demostraci´ on: Sean m, n ∈ Z≥1 tales que mcd(m, n) = 1. Por el Corolario 1.17(i), cada divisor positivo de mn se puede escribir como dd0 con d|m y d0 |n. Podemos entonces escribir F (mn) =

X d|m,d0 |n

f (dd0 ) =

X

f (d)f (d0 ) =

d|m,d0 |n

X d|m

28

f (d)

 X d0 |n

 f (d0 ) = F (m)F (n).

Ejemplo 3.10. Apliquemos la construcci´on anterior a los ejemplos que tenemos de funciones multiplicativas. 1) Evidentemente, si f es id´enticamente cero, entonces F tambi´en lo es. 2) Si f vale cero para todo n, excepto para f (1) = 1, es claro que entonces F es constantemente 1. 3) Cuando f es la funci´ on constante 1, se suele escribir τ para la funci´on F correspondiente, y se tiene que τ (n) es el n´ umero de divisores positivos de n. 4) Cuando f = id, la funci´ on F se suele denotar σ, y se tiene que σ(n) es la suma de los divisores positivos de n. 5) Cuando f = φ, en principio no es f´acil deducir qui´en es F . La ventaja es que sabemos que F es multiplicativa, por lo que basta calcularla para potencias de primos. Usando que los divisores de una potencia de un primo p son las potencias menores de p, tendremos, por la Observaci´ on 3.4: F (pk ) = φ(1) + φ(p) + φ(p2 ) + . . . + φ(pk−1 ) + φ(pk ) = = 1 + (p − 1) + (p2 − p) + . . . + (pk−1 − pk−2 ) + (pk − pk−1 ) = pk . Por tanto, F es la identidad en potencias de primos, y por ser multiplicativa se tendr´ a F = id. En otras palabras, hemos demostrado que para todo n ∈ Z≥1 se verifica: X φ(d) = n d|n

Observaci´ on 3.11. El hecho de que las funciones τ y σ anteriores son multiplicativas se puede deducir directamente de su propia definici´on. En efecto, dado un n´ umero k1 k2 p1 p2 . . . pkr r , con p1 , . . . , pr primos distintos, sus divisores son (por la Proposici´on 1.16) los n´ umeros de la forma pa1 1 . . . par r con 0 ≤ ai ≤ ki , es decir, los sumandos de (1 + p1 + p21 + . . . + pk11 )(1 + p2 + p22 + . . . + pk22 ) . . . (1 + pr + p2r + . . . + pkr r ). Claramente hay (k1 + 1)(k2 + 1) . . . (kr + 1) sumandos por lo que τ (pk11 pk22 . . . pkr r ) = (k1 + 1)(k2 + 1) . . . (kr + 1) y σ(pk11 pk22 . . . pkr r ) = (1 + p1 + p21 + . . . + pk11 )(1 + p2 + p22 + . . . + pk22 ) . . . (1 + pr + p2r + . . . + pkr r ) 29

o equivalentemente σ(pk11 pk22 . . . pkr r ) =

pk11 +1 − 1 pk22 1+1 − 1 pkr +1 − 1 ... r . p1 − 1 p2 − 1 pr − 1

Con estas descripciones, si mcd(m, n) = 1, eso quiere decir que sus factorizaciones son, como en la demostraci´ on del Teorema 3.9, de la forma m = pk11 pk22 . . . pkr r y n = q1l1 q2l2 . . . qsls , con pi 6= qj para todo i = 1, 2, . . . , r y j, 1, 2, . . . , s. Por tanto, mn factoriza como mn = pk11 pk22 . . . pkr r q1l1 q2l2 . . . qsls , y se tiene τ (mn) = (k1 + 1)(k2 + 1) . . . (kr + 1)(l1 + 1)(l2 + 1) . . . (ls + 1) = τ (m)τ (n) y σ(mn) =

pkr +1 − 1 q1l1 +1 − 1 q2l2 1+1 − 1 q ls +1 − 1 pk11 +1 − 1 pk22 1+1 − 1 ... r ... r = σ(m)σ(n) p1 − 1 p2 − 1 pr − 1 q1 − 1 q2 − 1 qs − 1

En el Ejemplo 3.10 hemos visto que, aplicando la construcci´on ejemplo 2 genera el 3 y que el 3 genera la funci´on τ , mientras que el funci´on σ y est´ a generado por la funci´on φ. Cabe preguntarse, por 2 est´a generado a partir de alguna funci´on aritm´etica. El siguiente respuesta:

del Teorema 3.9, el ejemplo 4 genera la tanto, si el ejemplo resultado nos da la

Proposici´ on 3.12. Sea µ : Z≥1 → {−1, 0, 1} la funci´on definida por:   1 µ(n) = 0  (−1)r

si n = 1 si existe un primo p tal que p2 |n si n = p1 p2 . . . pr con p1 , p2 , . . . , pr primos distintos

Entonces (i) µ es multiplicativa.  P 1 si n = 1 (ii) d|n µ(d) = 0 si n > 1. Demostraci´ on: Veamos primero la parte (i). Sean por tanto m, n ∈ Z≥1 con mcd(m, n) = 1. Distinguimos tres casos: –Si m = 1 o n = 1, es evidente que µ(mn) = µ(m)µ(n), ya que µ(1) = 1. –Si existe un primo p cuyo cuadrado divide a m (resp. n), entonces su cuadrado divide tambi´en a mn, luego µ(mn) = 0, que conicide con µ(m)µ(n) ya que µ(m) (resp. µ(n)) es cero. 30

–Finalmente si ni m ni n son divisibles por cuadrados de primos, se podr´a escribir m = p1 . . . pr y q1 . . . qs , con p1 , . . . , pr , q1 , . . . , qs primos distintos. Por tanto, mn = p1 . . . pr q1 . . . qs y se tendr´ a µ(mn) = (−1)r+s = (−1)r (−1)s = µ(m)µ(n). P Para la parte (ii), definimos F (n) = a multiplicativa por (i) y d|n µ(d), que ser´ el Teorema 3.9. Por tanto, basta comprobar que F (1) = 1 (lo que es evidente, ya que F (1) = µ(1) = 1) y que si p es un n´ umero primo y k > 0 entonces F (pk ) = 0. Esto u ´ltimo k 2 k j sigue de que los divisores de p son 1, p, p , . . . , p , y que por definici´on µ(p ) = 0 si j ≥ 2, por lo que F (pk ) = µ(1) + µ(p) + µ(p2 ) + . . . + µ(pk ) = 1 + (−1)1 + 0 + . . . + 0 = 0.

Definici´ on. La funci´ on µ definida en el teorema anterior se llama funci´ on de M¨ obius. Teorema 3.13. Sea f una funci´ on aritm´etica (no necesariamente multiplicativa) y sea F P la funci´on definida por F (n) = d|n f (d). Entonces: P P (i) (F´ormula de inversi´ on de M¨ obius) f (n) = d|n µ(d)F ( nd ) = d|n µ( nd )F (d). Pn Pn  n  (ii) i=1 F (i) = k=1 k f (k). Demostraci´ on: Es claro que, en (i), las dos expresiones para f (n) son iguales, as´ı que basta demostrarlo para la primera expresi´on. Tendremos entonces: X d|n

X X X  n µ(d)F ( ) = µ(d) f (c) = µ(d)f (c). d n n d|n

c| d

d|n,c| d

La primera observaci´ on ahora es que el conjunto de pares (d, c) tales que d|n y c| nd es igual al conjunto de pares (d, c) tales que c|n y d| nc . Por tanto, la igualdad anterior prosigue como X X X X  n µ(d)F ( ) = µ(d)f (c) = f (c) µ(d) . d n n d|n

c|n,d| c

c|n

Ahora bien, por la Proposici´ on 3.12 sabemos que  X 1 si µ(d) = 0 si n d| c

por lo que

P

c|n

f (c)

P

d| n c

 µ(d) = f (n). 31

n c n c

=1 > 1,

d| c

Para demostrar (ii), tenemos que n X

F (i) =

i=1

n X X i=1

n X  X f (d) = f (d). i=1 d|i

d|i

La observaci´ on central es que basta demostrar que, para cada k = 1, . . . , n, f (k) aparece n a tantas veces como k sea un divisor k veces en la suma anterior. Y en efecto, k aparecer´   de un n´ umero i ∈ {1, 2, . . . , n}. Como k es divisor de 1 · k, 2 · k, . . . , nk k, el resultado sigue inmediatamente. Ejemplo 3.14. Aplicando el resultado anterior a los casos 3, 4, y 5 del Ejemplo 3.10, deducimos respectivamente las siguientes f´ormulas para todo n ∈ Z≥1 : Si f = 1 y F = τ : X d|n n X

n µ( )τ (d) = 1 d

τ (i) =

i=1

k=1

si f = id y F = σ: X d|n n X

n h i X n

k

,

n µ( )σ(d) = n d

σ(i) =

i=1

n X n k=1

k

k

y si f = φ y F = id: φ(n) = n

X µ(d) d|n

d

=

X d|n

n µ( )d d

n X n i= φ(k) k i=1

n X

k=1

Veamos alguna aplicaci´ on de las funciones τ y σ en teor´ıa de n´ umeros. Ejemplo 3.15. La primera observaci´on es que es equivalente que un n´ umero n sea primo a que τ (n) = 2 o σ(n) = 1 + n. Las f´ormulas anteriores nos dan en realidad una f´ormula para calcular estas funciones para un n´ umero n del que no sabemos si factoriza. En efecto, Pn−1 Pn la observaci´ on es que en general F (n) = i=1 F (i) − i=1 F (i), as´ı que, por ejemplo, tendremos la f´ ormula, para n > 1, τ (n) =

n X n k=1

k



n−1 X k=1

n−1 X  n   n − 1  n − 1 =2+ − . k k k k=2

32

Por ejemplo, para n = 7, podemos escribir τ (7) = 2 + (3 − 3) + (2 − 2) + (1 − 1) + (1 − 1) + (1 − 1) = 2, lo que concluye que 7 es primo. En realidad, el resultado no es sorprendente, ya que n > 1 es primo si y s´ olo si no es divisible por ning´ un k = 2, . . . , n − 1, es decir, que nk no es  n   n−1  entero, que es equivalente a k = k , es decir τ (n) = 2. Veamos otra simple aplicaci´ on del tipo de ideas anterior para estudiar una clase de n´ umeros que llamaron la atenci´ on a los griegos cl´asicos: Definici´ on. Se llama n´ umero perfecto a un entero positivo que sea igual a la suma de todos sus divisores menores que ´el. Usando la terminolog´ıa anterior, la suma de los divisores de n es σ(n), por lo que para obtener la suma de los divisores de n menores que n hay que quitar el propio n. Es decir, n es un n´ umero perfecto si y s´olo si σ(n) − n = n, i.e. σ(n) = 2n. El resultado m´ as importante sobre los n´ umeros perfectos es el siguiente: Teorema 3.16. Un n´ umero par n es un n´ umero perfecto si y s´olo si se puede escribir k−1 k k como n = 2 (2 − 1), siendo k ≥ 2 y 2 − 1 un n´ umero primo. Demostraci´ on: Supongamos en primer lugar que n es un n´ umero perfecto par. Lo escribimos separando los factores primos impares, es decir, de la forma n = 2k−1 m con k ≥ 2 y m impar. La condici´ on de que n sea perfecto es equivalente a σ(n) = 2n, que usando que σ es multiplicativa y la f´ ormula de la Observaci´on 3.11 quiere decir: 2k m = 2n = σ(n) = σ(2k−1 m) = σ(2k−1 )σ(m) = (2k − 1)σ(m). Como mcd(2k , 2k − 1) = 1, se sigue de que existe M ∈ Z≥1 tal que ( m = (2k − 1)M σ(m) = 2k M En particular, m y M son dos divisores distintos de m, por lo que se tendr´a σ(m) ≥ m+M , y de hecho se dar´ a la igualdad si y s´olo si m y M son los u ´nicos divisores de m, es decir, m es primo y M = 1. Ahora observamos que efectivamente se da la igualdad, ya que se tiene 2k M = σ(m) ≥ m + M = (2k − 1)M + M = 2k M. Por tanto, m = 2k − 1 es un n´ umero primo y n = 2k−1 m = 2k−1 (2k − 1). Rec´ıprocamente, si n = 2k−1 (2k − 1), con k ≥ 2 y 2k − 1 = p primo, entonces, como σ es multiplicativa y usando la f´ ormula de la Observaci´on 3.11, se tiene σ(n) = σ(2k−1 p) = σ(2k−1 )σ(p) = (2k − 1)(p + 1) = (2k − 1)2k = 2n.

33

El inter´es del resultado anterior radica en que no se conocen n´ umeros perfectos impares, y de hecho todo indica que no existen (por ejemplo, se sabe que un n´ umero perfecto impar deber´ıa tener al menos 300 cifras). Respecto a la existencia de infinitos n´ umeros perfectos pares, es tambi´en un problema abierto. Por el teorema anterior, bastar´ıa que demostrar que existen infinitos n´ umeros primos de la forma 2k −1. Es una simple observaci´ on k s k que si 2 − 1 es primo entonces k es primo (ya que, si r|k =, entonces 2 − 1|2 − 1). Definici´ on. Se llama n´ umero primo de Mersenne a un n´ umero primo de la forma Mk := k 2 − 1 con k primo (n´ otese que es equivalente a decir que, en base dos, se escribe s´olo con unos). Por tanto, existen infinitos n´ umeros perfectos pares si y s´olo si existen infinitos n´ umeros primos de Mersenne (lo que se conjetura que es cierto). Damos a continuaci´ on una tabla con los primeros k primos tales que Mk es primo, con el correspondiente n´ umero k−1 k perfecto n = 2 (2 − 1). k

Mk = 2k − 1

2

3

6

3

7

28

5

31

496

7

127

8128

13

8191

33550336

17

131071

8589869056

19

524287

137438691328

31

2147483647

2305843008139952128

61

n = 2k−1 (2k − 1)

2305843009213693951 2658455991569831744654692615953842176

N´otese que k = 11 es el primer n´ umero primo tal que Mk no es primo (M11 = 2047 = 23 · 89), y a partir de ah´ı hay muchos k con la misma propiedad. Hasta la fecha, se conocen s´ olo 43 n´ umeros primos de Mersenne (el u ´ltimo, obtenido en septiembre de 2008). El mayor n´ umero primo de Mersenne que se conoce (encontrado en agosto de este a˜ no) es para k = 43112609, y el n´ umero de cifras de Mk es 12978189 (casi trece millones). El primo M31 ya era conocido por Euler en 1772. 34

´ 4. Ordenes, ra´ıces primitivas e ´ındices El Ejemplo 3.7 muestra que, dado n ∈ Z≥1 , no siempre φ(n) es la menor potencia a la que debemos elevar un n´ umero para que nos d´e 1 m´odulo n. En este cap´ıtulo, pretendemos estudiar en qu´e condiciones φ(n) es dicho menor exponente. Las definiciones naturales de partida son las siguientes: Definici´ on. Fijado un entero positivo n, para cada a ∈ Z tal que mcd(a, n) = 1 se llama orden del elemento a m´ odulo n al menor entero positivo k tal que ak ≡ 1 (mod n). Del teorema de Euler sigue k ≤ φ(n), y diremos que a es una ra´ız primitiva (de la unidad) m´ odulo n si su orden es k = φ(n). Ejemplo 4.1. Estudiemos el caso n = 11. Si tomamos a = 2, es un simple c´alculo observar que las potencias 21 , 22 , 23 , 24 , 25 , 26 , 27 , 28 , 29 , 210 son respectivamente congruentes con 2, 4, 8, 5, 10, 9, 7, 3, 6, 1 m´ odulo 11. Por tanto, 2 tiene orden 10 m´odulo 11, y por tanto es una ra´ız primitiva. Obs´ervese que las distintas potencias nos han dado precisamente todos los restos 1, 2, 3, 4, 5, 6, 7, 8, 9, 10, aunque en orden distinto. Ya veremos m´as adelante que esto no es una coincidencia. Si en cambio tomamos a = 3, las potencias 31 , 32 , 33 , 34 , 35 son respectivamente congruentes con 3, 9, 5, 4, 1 m´odulo 11, por lo que 3 tiene orden 5 m´odulo 11 y no es una ra´ız primitiva. Tampoco es casualidad que el orden de 3 sea un divisor de φ(11) = 10. No tienen por qu´e existir siempre ra´ıces primitivas (de hecho, es lo menos frecuente). Por ejemplo, el Ejemplo 3.7 indica que 35 no tiene ra´ıces primitivas. M´as a´ un, indica que el problema es que 35 factoriza en dos n´ umeros primos entre s´ı, 7 y 5, cuyos valores de φ no son primos entre s´ı, ya que son ambos pares. El siguiente resultado indica que esta situaci´on se repetir´ a muchas veces. Lema 4.2. Si n > 2, φ(n) es par. Demostraci´ on: Si n contiene alg´ un divisor primo impar p, entonces el resultado es evidente, k ya que se podr´ a escribir n = p m con mcd(m, p) = 1 y k ≥ 1. Entonces φ(n) = φ(pk )φ(m), y φ(pk ) es par, ya que por la Observaci´on 3.4 se tiene φ(pk ) = pk−1 (p − 1) y p − 1 es par. Queda por tanto estudiar el caso en que n no tiene factores impares, es decir, es de la forma n = 2k . En tal caso, φ(2k ) = 2k−1 , que es tambi´en par por ser k > 1 (puesto que n > 2). Del lema sacamos ya muchos casos en que no hay ra´ıces primitivas. Proposici´ on 4.3. Si mcd(m, n) > 1 y m, n > 2, entonces mn no tiene ra´ıces primitivas. 35

Demostraci´ on: Sea a ∈ Z tal que mcd(a, mn) = 1, es decir, mcd(a, m) = 1 y mcd(a, n) = 1 (ver Proposici´ on 1.9). Si aplicamos el Teorema de Euler separadamente a m y n, tendremos φ(m) y φ(m) que a ≡ 1 (mod m) y aφ(n) ≡ 1 (mod n). Elevando respectivamente a φ(n) 2 2 φ(m)φ(n)

(que son enteros, ya que por el lema φ(m) y φ(n) son pares) obtenemos que a 2 es congruente con 1 m´ odulo m y n, luego tambi´en m´odulo mn. Por tanto, el orden de a . Como φ(mn) = φ(m)φ(n) al ser mcd(m, n) = 1, se m´odulo mn es a lo sumo φ(m)φ(n) 2 tiene que ning´ un a tiene orden φ(mn). Como consecuencia, los u ´nicos n´ umeros que pueden tener ra´ıces primitivas son los de k k la forma p o 2p (con p 6= 2 en el segundo caso). El caso de las potencias de 2 queda excluido (salvo para las dos primeras potencias) por el siguiente resultado. Proposici´ on 4.4. Para todo n´ umero impar a y para cada entero k ≥ 3, se tiene que 2k−2 k a ≡ 1 (mod 2 ). Por tanto, 2k no tiene ra´ıces primitivas. Demostraci´ on: Si demostramos la primera parte, como φ(2k ) = 2k−1 por la Observaci´ on 3.4, se sigue que 2k no tiene ra´ıces primitivas. Basta entonces demostrar la primera parte, k−2 es decir, 2k |a2 − 1 si k ≥ 3. Lo demostraremos por inducci´on sobre k. Si k = 3, entonces tenemos a2 − 1 = (a + 1)(a − 1). Como a − 1 y a + 1 son dos pares consecutivos, uno de ellos es necesariamente m´ ultiplo de 4, con el producto de ambos es divisible por 8. Supongamos ahora que lo tenemos demostrado para k y queremos demostrarlo para k−1 k−2 k−2 k + 1. Entonces escribimos a2 − 1 = (a2 + 1)(a2 − 1). El primer factor es par, mientras que el segundo, por hip´ otesis de inducci´on, es divisible por 2k , as´ı que el producto de ambos factores es divisible por 2k+1 , como quer´ıamos demostrar. El resultado final, que demostraremos algo m´as adelante, cuando dispongamos de las t´ecnicas necesarias, es el siguiente: Teorema 4.5. Un n´ umero natural n > 1 posee ra´ıces primitivas si y s´olo si es n = 2, n = 4, n = pk o n = 2pk donde p es un primo impar y k ≥ 1. Por las Proposiciones 4.3 y 4.4, tenemos que los u ´nicos casos posibles son los del enunciado. Por otra parte, es evidente que 3 es una ra´ız primitiva m´odulo 2 y m´odulo 4. Por tanto, basta ver que los n´ umeros de la forma pk o 2pk tienen ra´ıces primitivas. Eso lo demostraremos en el Teorema 4.12. Dado que debemos estudiar la existencia de elementos de orden φ(n), parece razonable estudiar primero propiedades de los ´ordenes de los elementos. 36

Proposici´ on 4.6. Sea a un elemento de orden k m´odulo n. Entonces: (i) ah ≡ 1 (mod n) si y s´ olo si k|h. (ii) k es un divisor de φ(n). (iii) ai ≡ aj (mod n) si y s´ olo si i ≡ j (mod k). (iv) Los elementos a, a2 , . . . , ak−1 , ak son incongruentes m´odulo n. (v) Si h > 0, entonces ah tiene orden

k mcd(h,k)

m´odulo n.

Demostraci´ on: Para la parte (i), sea h = qk + r la divisi´on eucl´ıdea de h entre k, por lo que 0 ≤ r < k. Se tendr´ a entonces ah = aqk+r = (ak )r ar ≡ 1k ar = ar (mod n). Por tanto, ah ≡ 1 (mod n) si y s´ olo si ar ≡ 1 (mod n). Como r < n y, por definici´ on de k r orden, k es el menor entero positivo tal que a ≡ 1 (mod n), se sigue que a ≡ 1 (mod n) si y s´olo si r ≤ 0, es decir, si y s´ olo si r = 0 (ya que r ≥ 0). Como esto es equivalente a decir k|h, queda demostrado (i). La parte (ii) es una consecuencia de (i), ya que, por el Teorema de Euler, se tiene ≡ 1 (mod n).

φ(n)

a

Para (iii), podemos suponer i ≤ j. Entonces ai (aj−i − 1) ≡ 0 (mod n), es decir, n|ai (aj−i − 1). Como mcd(a, n) = 1, se sigue que n|aj−i − 1, es decir, aj−i ≡ 1 (mod n). Por (i), k|j − i, o lo que es lo mismo i ≡ j (mod k). La parte (iv) es una consecuencia de (iii), ya que, si i, j ∈ {1, 2, . . . , k}, la condici´ on j a ≡ a (mod n) es equivalente a i ≡ j (mod k), que a su vez es equivalente a i = j, por ser {1, 2, . . . , k} un sistema completo de restos m´odulo k (Proposici´on 2.5(i)). i

Finalmente, para demostrar la parte (v), observamos que (ah )j ≡ 1 (mod n) si y s´ olo hj si a ≡ 1 (mod n), que por (i), es equivalente a k|hj. Por el Lema 1.1(iv), esto equivale k h k h a mcd(h,k) | mcd(h,k) j. Como, por la Proposici´on 1.5(ii), mcd(h,k) y mcd(h,k) son primos entre s´ı, el Lema de Euclides implica que la condici´on anterior es equivalente tambi´en a k h odulo n mcd(h,k) |j. En otras palabras, las potencias de a que son congruentes con 1 m´ k k son precisamente los m´ ultiplos de mcd(h,k) , lo que implica que mcd(h,k) es precisamente el h orden de a m´ odulo n. La observaci´ on principal a la hora de buscar ra´ıces primitivas es que ´estas son las soluciones de la congruencia xφ(n) ≡ 1 (mod n) que no son soluci´on soluci´on de ninguna congruencia de la forma xd ≡ 1 (mod n) con d|φ(n). En los casos que hemos visto en que n no tiene ra´ıces primitivas ocurre que existe d < φ(n) tal que la congruencia xd ≡ 1 (mod n) tiene φ(n) soluciones m´odulo n (como en el Ejemplo 3.7, en que la 37

congruencia x12 ≡ 1 (mod 35) tiene 24 soluciones). Este tipo de anomal´ıas (tener m´ as soluciones que el grado de la congruencia) no se dan en el caso en que n es un n´ umero primo (ni para congruencias de grado uno, como indica la Proposici´on 2.9), y de hecho se tiene el siguiente resultado, que ser´a clave para demostrar el Teorema 4.5: Teorema 4.7 (Lagrange). Sea p un n´ umero primo y f (x) = ad xd +ad−1 xd−1 +. . .+a1 x+a0 un polinomio con alg´ un ai no divisible por p. Entonces existen como mucho d soluciones distintas m´ odulo p de la congruencia f (x) ≡ 0 (mod p). Demostraci´ on: Lo hacemos por inducci´on sobre d. Si d = 0, entonces f (x) es una constante a0 no divisible por p, por lo que f (x) ≡ 0 (mod p) no tiene soluci´on. Supongamos entonces que tenemos el resultado demostrado para polinomios no nulos de grado menor o igual que d y demostr´emoslo para polinomios de grado d + 1. Tomemos por tanto un polinomio f (x) = ad+1 xd+1 + ad xd + . . . + a1 x + a0 con alg´ un ai no divisible por p. Distinguimos dos casos: –Si p|ad+1 , entonces alg´ un ai , con i = 0, 1, . . . , d, no es divisible por p. Adem´as, las soluciones de f (x) ≡ 0 (mod p) son las soluciones de ad xd + . . . + a1 x + a0 ≡ 0 (mod p), que, por hip´ otesis de inducci´ on, son al m´aximo d distintas m´odulo p. Por tanto, en este caso hay incluso menos de d + 1 soluciones m´odulo p. –Si en cambio p 6 |ad+1 , demostraremos por reducci´on al absurdo que f (x) ≡ 0 (mod p) no puede tener m´ as de d + 1 soluciones m´odulo p. En efecto, supongamos que tuviera d + 2 soluciones distintas, que denotamos por x ≡ x1 , x2 . . . , xd+1 , xd+2 (mod p). Consideramos el polinomio g(x) = f (x) − ad+1 (x − x1 ) . . . (x − xd+1 ), que claramente tiene grado como mucho d (ya que el coeficiente de xd+1 es cero). Adem´ as, no todos sus coeficientes son divisibles por p, ya que g(xd+2 ) = −ad+1 (xd+2 −x1 ) . . . (xd+2 − xd+1 ) 6≡ 0 (mod p) porque es el producto de n´ umeros no divisibles por p (Corolario 1.14). Por tanto, por hip´ otesis de inducci´on la congruencia g(x) ≡ 0 (mod p) tiene a lo sumo d soluciones distintas m´ odulo p, mientras que claramente x ≡ x1 , x2 . . . , xd+1 (mod p) son soluciones, lo que nos da la contradicci´on que busc´abamos.

Definici´ on. Dado un polinomio f (x) con coeficientes enteros, llamaremos ra´ıces del polinomio m´ odulo p a las soluciones de la congruencia f (x) ≡ 0 (mod p). Un ejemplo t´ıpico del resultado anterior es el polinomio xp−1 − 1, que por el peque˜ no Teorema de Fermat tiene exactamente p−1 ra´ıces m´odulo p. Entre este hecho y el teorema podemos demostrar el siguiente resultado fundamental: 38

Proposici´ on 4.8. Para cada d|p−1, la congruencia xd −1 ≡ 0 (mod p) tiene exactamente d soluciones m´ odulo p. Demostraci´ on: Si escribimos p − 1 = dk, entonces podemos escribir  xp−1 − 1 = (xd )k − 1 = (xd − 1) (xd )k−1 + (xd )k−2 + . . . + xd + 1 = (xd − 1)f (x) donde f (x) est´ a definido como el segundo factor, y en particular tiene grado dk − d = p − 1 − d. Como, por el peque˜ no teorema de Fermat cada a primo con a es ra´ız m´odulo p−1 p de x − 1, entonces tambi´en es ra´ız de xd − 1 o de f (x). Como por el Teorema de Lagrange xd − 1 tiene como mucho d ra´ıces m´odulo p y f (x) tiene como mucho p − 1 − d, se sigue que se da la igualdad en ambos casos, es decir, xd − 1 tiene exactamente d ra´ıces m´odulo p. Podemos demostrar ya el resultado principal que nos da los elementos que tenemos de cada orden, y en particular la existencia de ra´ıces primitivas m´odulo p: Teorema 4.9. Para cada d|p−1, existen exactamente φ(d) enteros incongruentes de orden d m´odulo p. Demostraci´ on: Llamamos ψ(d) al n´ umero de enteros incongruentes m´odulo p de orden d. Como el orden de un elemento m´ odulo p es un divisor de φ(p) = p − 1, se tendr´a X ψ(d) = p − 1. d|p−1

Por otra parte, como vimos en la parte 5 del Ejemplo 3.10, X φ(d) = p − 1, d|p−1

por lo que bastar´ a ver que ψ(d) ≤ φ(d) para cada d|p−1. Sea entonces d|p−1. Si ψ(d) = 0, entonces no hay nada que demostrar, as´ı que podemos suponer que existe un elemento a de orden d m´odulo p. Entonces, los elementos a, a2 , . . . , ad son todos incongruentes m´ odulo d p por la Proposici´ on 4.6(iv), luego son las d soluciones de x − 1 ≡ 0 (mod p) de la proposici´on anterior. Por tanto, los elementos de orden d son de la forma ai , y por la Proposici´on 4.6(v), un elemento ai tiene orden d si y s´olo si mcd(i, d) = 1 , es decir, hay exactamente φ(d) elementos incongruentes de orden d, lo que demuestra el resultado. Corolario 4.10. Cualquier n´ umero primo tiene ra´ıces primitivas. Demostraci´ on: Es consecuencia del teorema, tomando d = p − 1. De hecho, podemos precisar que hay exactamente φ(p − 1) elementos incongruentes entre s´ı de orden p − 1 m´odulo p, es decir, ra´ıces primitivas m´odulo p. 39

Pasar de ra´ıces primitivas m´ odulo un n´ umero primo a una potencia suya se basa en la siguiente observaci´ on: Lema 4.11. Sea r una ra´ız primitiva m´odulo un n´ umero primo p. Entonces, para cualquier k k ≥ 2, el orden de r m´ odulo p es de la forma (p − 1)pj , con j ≤ k − 1. En particular, r k−2 es una ra´ız primitiva m´ odulo pk si y s´olo si pk no divide a rp (p−1) − 1. Demostraci´ on: Sea m el orden de r m´odulo pk . Se tendr´a, en particular, am ≡ 1 (mod pk ), y en particular am ≡ 1 (mod p). Por el Lema 4.6(i), se tendr´a p − 1|m, ya que p − 1 es el orden de r m´ odulo p (por ser r ra´ız primitiva). Adem´as, por el Lema 4.6(ii) aplicado k ahora a n = p , se tendr´ a m|φ(pk ) = pk−1 (p − 1). De estas dos condiciones sobre m se deduce que es de la forma m = (p − 1)pj , con j ≤ k − 1. Para demostrar la segunda parte, es claro ahora que r no es una ra´ız primitiva m´ odulo j p si y s´olo si tiene orden m = (p−1)p , con j ≤ k −2, es decir si y s´olo si su orden m divide k−2 a (p − 1)pk−2 . Por el Lema 4.6(i), esta condici´on es equivalente a rp (p−1) ≡ 1 (mod pk ), k−2 es decir, a pk |rp (p−1) − 1. k

Este resultado nos permite finalmente terminar de demostrar el Teorema 4.5: Teorema 4.12. Si p es un n´ umero primo impar, pk y 2pk tienen ra´ıces primitivas para cada k ≥ 1. Demostraci´ on: Ya hemos visto en el Corolario 4.10 que existen ra´ıces primitivas m´ odulo p. Demostremos en primer lugar una ra´ız primitiva m´odulo p2 . Para ello, fijaremos una ra´ız primitiva r m´ odulo p, y veamos que, si r no es una ra´ız primitiva m´odulo p2 , entonces r + p lo es (en realidad valdr´ıa cualquier r + ip con mcd(i, p) = 1). En efecto, si r no es una ra´ız primitiva m´odulo p2 , por el Lema 4.11 sabemos que p2 divide a rp−1 − 1. Por tanto, m´ odulo p2 tendremos (r + p)p−1 − 1 ≡ rp−1 + (p − 1)rp−2 p − 1 ≡ (p − 1)rp−2 p (mod p2 ). Como (p − 1)rp−2 no es divisible por p, entonces (p − 1)rp−2 p no es divisible por p2 , luego (r + p)p−1 − 1 tampoco lo es, lo que implica por Lema 4.11 que r + p es una ra´ız primitiva m´odulo p2 . Construida pues una ra´ız primitiva r m´odulo p2 , veamos que, por ser p impar, r es tambi´en una ra´ız primitiva m´ odulo pk para cualquier k ≥ 2. Por el Lema 4.11, tenemos k−2 que demostrar que, si p2 no divide a rp−1 − 1, entonces pk no divide a rp (p−1) − 1. Lo demostraremos por inducci´ on sobre k, siendo el resultado trivial para k = 2. Supongamos ahora cierto el resultado para k y vamos a demostrarlo para k + 1. Por el Teorema de 40

k−2

Euler para pk−1 , sabemos que rp (p−1) − 1 es divisible por pk−1 , y nuestra hip´otesis de inducci´on indica que no es divisible por pk . En otras palabras, podemos escribir rp

k−2

(p−1)

= 1 + apk−1

con a no divisible por p. Elevando a p, tendremos r

pk−1 (p−1)

= (1 + ap

  p 2 2k−2 ) = 1 + ap + a p + ... 2

k−1 p

k

 Como p > 2 (es aqu´ı donde usamos que p sea impar), p2 es divisible por p, luego el sumando  p 2 2k−2 es divisible por p2k−1 y 2k − 1 ≥ k + 1 por ser k ≥ 2. El resto de sumandos en 2 a p los puntos suspensivos son divisibles por pi(k−1) con i ≥ 3, y como i(k − 1) ≥ 3k − 3 ≥ k + 1 son tambi´en divisible por pk+1 . Se concluye as´ı que k−1

rp

(p−1)

≡ 1 + apk (mod pk+1 )

y como apk no es divisible por pk+1 , entonces rp

k−1

(p−1)

− 1 tampoco lo es.

Finalmente, dada una ra´ız primitiva m´odulo pk , si es par le a˜ nadimos pk y seguir´ a k siendo una ra´ız primitiva m´ odulo p , pero ahora impar. Por tanto, existe r impar que es una k ra´ız primitiva m´ odulo p . Veamos que es tambi´en ra´ız primitiva m´odulo 2pk . En efecto, en primer lugar es evidente que mcd(r, 2pk ) = 1, ya que mcd(r, 2) = 1 y mcd(r, pk ) = 1. Por k otra parte φ(2pk ) = (2 − 1)pk−1 (p − 1) = pk−1 (p − 1) = φ(pk ), luego rφ(2p ) ≡ 1 (mod 2pk ) y no existe a < φ(2pk ) tal que ra ≡ 1 (mod 2pk ), por lo que r tiene orden φ(2pk ) m´odulo pk .

Ejemplo 4.13. El resultado anterior es constructivo. Por ejemplo, vamos a buscar una ra´ız primitiva m´ odulo 242 = 2 · 112 . Ya sabemos, por el Ejemplo 4.1, que 2 es una ra´ız primitiva m´ odulo 11. Nos preguntamos si ser´a tambi´en una ra´ız primitiva m´odulo 112 , para lo que usaremos el Lema 4.11. Como 210 − 1 = 1023 = 3 · 11 · 31 no es divisible por 112 , se sigue que 2 es, en efecto, una ra´ız primitiva m´odulo 112 (n´otese que, si hubi´eramos obtenido que no lo era, la demostraci´on del Teorema 4.12(ii) nos dir´ıa autom´aticamente que 2 + 11 es una ra´ız primitiva). Finalmente, por la demostraci´on de la parte (iv) del Teorema 4.12, tenemos que 2 + 112 = 123 es una ra´ız primitiva m´odulo 2 · 112 . Ahora que sabemos qu´e n´ umeros tienen ra´ız primitiva, veamos en qu´e podemos usarlo. Obs´ervese en primer lugar que, si r es una ra´ız primitiva m´odulo n, entonces por la Proposici´on 4.6(iv) los elementos r, r2 , . . . , rφ(n) son todos no congruentes dos a dos, luego, por el Lema 3.1, son congruentes m´odulo n, en alg´ un orden, a los n´ umeros 1, 2, . . . , n que 41

son primos con n. Por tanto, cualquier a ∈ Z con mcd(a, n) = 1 es congruente a un u ´nico ri con i ∈ {1, 2, . . . , φ(n)}. Recordando la definici´on de logaritmo, damos la siguiente: Definici´ on. Dado un n´ umero n con ra´ız primitiva r, se llama ´ındice de a relativo a r al menor entero positivo k tal que a ≡ rk (mod n) y escribiremos k = indr (a). Ejemplo 4.14. Si n = 11, sabemos por el Ejemplo 4.1 que r = 2 es una ra´ız primitiva m´odulo 11. Adem´ as, 1, 2, 3, 4, 5, 6, 7, 8, 9, 10 son, respectivamente, congruentes m´odulo 11 10 1 8 2 4 9 7 3 6 5 con 2 , 2 , 2 , 2 , 2 , 2 , 2 , 2 , 2 , 2 , por lo que sus ´ındices relativos a 2 son, respectivamente, 10, 1, 8, 2, 4, 9, 7, 3, 6, 5 El ´ındice funciona, en efecto, como un logaritmo en base r, y de hecho se tienen las siguientes propiedades: Proposici´ on 4.15. Si r es una ra´ız primitiva m´odulo n, se verifica: (i) rk ≡ a (mod n) si y s´ olo si k ≡ indr (a) (mod φ(n)). (ii) indr (ab) ≡ indr (a) + indr (b) (mod φ(n)). (iii) indr (ak ) ≡ kindr (a) (mod φ(n)). (iv) indr (1) ≡ 0 (mod φ(n)) (y de hecho indr (1) = φ(n)). (v) indr (r) = 1. Demostraci´ on: Veamos primero (i). Como por definici´on rindr (a) ≡ a (mod n), entonces rk ≡ rindr (a) (mod n), y por la Proposici´on 4.6(iii) se sigue k ≡ indr (a) (mod φ(n)) (recu´erdese que el hecho de que r sea ra´ız primitiva m´odulo n se traduce en que el orden de r m´odulo n es φ(n)). El resto de los apartados se demuestra inmediatamente a partir de (i). Haremos s´ olo, a modo de ejemplo, (ii). Como se tiene rindr (a)+indr (b) = rindr (a) rindr (b) ≡ ab (mod n) la parte (i) implica indr (a) + indr (b) ≡ indr (ab) (mod φ(n)). La teor´ıa de ´ındices se usa para resolver cierto tipo de congruencias no lineales, pasando de congruencias m´ odulo n a congruencias m´ odulo φ(n). Antes de ver el resultado preciso, hagamos un ejemplo. Ejemplo 4.16. Consideremos la congruencia x6 ≡ 3(mod 11). Evidentemente, cualquier soluci´on verifica mcd(x, 11) = 11. Por tanto, como 2 es una ra´ız primitiva m´odulo 11 ver Ejemplo 4.1), se podr´ a se podr´a escribir x ≡ 2y (mod 11). Adem´as, sabemos que 3 ≡ 28 (mod 11), por lo que nuestra congruencia es equivalente a 26y ≡ 28 (mod 11). 42

Por la Proposici´ on 4.6(iii), esta congruencia es equivalente a 6y ≡ 8 (mod 10). Aplicando ahora la Proposici´ on 2.9, sabemos que esta congruencia tiene dos soluciones, precisamente y ≡ 3, 8 (mod 10). De nuevo por la Proposici´on 4.6(iii), estas soluciones son equivalentes a 2y ≡ 23 , 28 (mod 11), es decir, x ≡ 8, 3 (mod 11). Teorema 4.17. Sea n un entero con una ra´ız primitiva r, y sean k ≥ 2 y d = mcd(k, φ(n)). Si a ∈ Z verifica mcd(a, n) = 1, y por tanto a ≡ rb (mod n) para alg´ un b ∈ Z≥1 , entonces son equivalentes: (i) La congruencia xk ≡ a (mod n) tiene soluci´on. (ii) d|b. (iii) a

φ(n) d

≡ 1 (mod n).

Adem´as, en tal caso, la congruencia tiene exactamente d soluciones m´odulo n Demostraci´ on: Evidentemente, las soluciones de xk ≡ a (mod n) verifican mcd(x, n) = 1, por lo que se podr´ a escribir x ≡ ry (mod n) para alg´ un y ∈ Z. En otras palabras, la congruencia tendr´ a soluci´ on si y s´ olo si tiene soluci´on la congruencia rky ≡ rb (mod n) que, por la Proposici´ on 4.6(iii), es equivalente a ky ≡ b (mod φ(n)). Por la Proposici´ on 2.9, tal congruencia tiene soluci´on si y s´olo si d|b, y adem´as en tal caso el n´ umero de soluciones m´ odulo φ(n) es d. Esto da la equivalencia de (i) y (ii), y adem´ as tomando potencias de r nos dice que el n´ umero de soluciones de nuestra congruencia m´odulo n es d. Finalmente, d|b es equivalente a φ(n)|b φ(n) on 4.6(i), es equivad , que por la Proposici´ φ(n)

lente a rb d ≡ 1 (mod n), es decir, a son equivalentes.

φ(n) d

≡ 1 (mod n), lo que demuestra que (ii) y (iii)

El resultado anterior nos permite generalizar el Teorema 2.19: Teorema 4.18. Sea p un n´ umero primo impar, y sea m ≥ 1. Entonces la congruencia 2m x ≡ −1 (mod p) tiene soluci´ on si y s´olo si p ≡ 1 (mod 2m+1 ). Demostraci´ on: Si escribimos p − 1 = 2n a con a impar, entonces es evidente que d = max{0,n−m} mcd(2m , p − 1) = 2min{n,m} y por tanto p−1 a. Por la parte (iii) del teorema d =2 (que podemos usar ya que p tiene ra´ıces primitivas), la congruencia tendr´a soluci´on si y p−1 s´olo si (−1) d ≡ 1 (mod p), es decir, si y s´olo si p−1 olo si n > m, si y s´ olo d es par, si y s´ m+1 m+1 si 2 |p − 1, si y s´ olo si p ≡ 1 (mod 2 ). Podemos por tanto generalizar el Corolario 2.20: 43

Teorema 4.19. Para cada m ≥ 1, existen infinitos n´ umero primos de la forma 2m k + 1. Demostraci´ on: Si p1 , p2 , . . . , pr fueran los u ´nicos primos de la forma 2m k +1, consideramos m−1 el n´ umero n = (2p1 p2 . . . pr )2 + 1. Claramente, n es impar, luego sus posibles divisores primos son impares. Si p es un divisor primo impar de n, entonces 2p1 p2 . . . pr es una m−1 soluci´on de la congruencia x2 ≡ −1 (mod p). Por el Teorema 4.18, p ≡ 1 (mod 2m ), es decir, p es de la forma 2m k + 1. Por tanto p tiene que ser alguno de los p1 , p2 , . . . , pr , lo que es absurdo.

Ejercicio 4.20. Demostrar que, para cada m ≥ 2, existen infinitos n´ umero primos de la m m−1 forma 2 k + 2 + 1.

44

5. Congruencias cuadr´aticas En el Teorema 4.17 hemos visto que el Teorema 4.12 parece funcionar particularmente bien para ciertas congruencias de grado 2k . En este cap´ıtulo vamos a centrarnos en las congruencias de grado dos. Obs´ervese en primer lugar que cualquier congruencia m´ odulo un entero n se puede descomponer en congruencias m´odulo las potencias de primos que dividen a n. Podemos usar por tanto que, con la excepci´on de las potencias de dos, las potencias de un primo tienen ra´ıces primitivas. De todas formas, vamos a iniciar nuestro estudio con congruencias cuadr´ aticas m´odulo un n´ umero primo. Podemos suponer que el 2 primo es impar, ya que x ≡ x (mod 2), luego las congruencias cuadr´aticas m´odulo dos son equivalentes a congruencias lineales. El primer resultado nos da la cl´asica resoluci´on de una ecuaci´on de grado dos, pero ahora en el lenguaje de congruencias. Proposici´ on 5.1. Sea p un n´ umero primo impar y sean a, b, c ∈ Z tales que mcd(a, p) = 1. 2 Entonces la congruencia ax + bx + c ≡ 0 (mod p) tiene soluci´on si y s´olo si la congruencia x2 ≡ b2 − 4ac (mod p) tiene soluci´on. Adem´as, las soluciones de ax2 + bx + c ≡ 0 (mod p) son exactamente las soluciones de 2ax ≡ −b + d (mod p) y 2ax ≡ −b − d (mod p), donde d es una soluci´ on de x2 ≡ b2 − 4ac (mod p). Demostraci´ on: Como mcd(4a, p) = 1, la congruencia ax2 + bx + c ≡ 0 (mod p) es equivalente a 4a2 x2 + 4abx + 4ac ≡ 0 (mod p), es decir, (2ax + b)2 ≡ b2 − 4ac (mod p). Por tanto, si nuestra congruencia original tiene soluci´on, entonces b2 − 4ac es un cuadrado m´odulo p. Rec´ıprocamente, si d2 ≡ b2 − 4ac (mod p), entonces la congruencia anterior es equivalente a p|(2ax + b)2 − d2 , es decir p|(2ax + b + d)(2ax + b − d), que es a su vez equivalente a 2ax ≡ −b + d (mod p) o 2ax ≡ −b − d (mod p), y ambas tienen soluci´ on u ´nica porque mcd(2a, p) = 1 (ver Proposici´on 2.9). El resultado anterior indica que para saber resolver ecuaciones cuadr´aticas m´odulo un n´ umero primo basta saber decidir cu´ando un n´ umero es un cuadrado m´odulo dicho n´ umero primo. Por supuesto, si el n´ umero es divisible por el n´ umero primo, es congruente con el cuadrado de cero, y es la u ´nica soluci´on. En caso contrario, podemos usar el Teorema 4.17 para dar una respuesta: Teorema 5.2 (Criterio de Euler). Sea p un n´ umero primo impar y sea a ∈ Z tal que p−1 2 mcd(a, p) = 1. Entonces la congruencia x ≡ a (mod p) tiene soluci´on si y s´olo si a 2 ≡ p−1 1 (mod p). Adem´ as, si la congruencia no tiene soluci´on se verifica a 2 ≡ −1 (mod p) 45

Demostraci´ on: Como φ(p) = p − 1 es par, se tiene que mcd(2, φ(p)) = 2, luego por el p−1 Teorema 4.17 la congruencia x2 ≡ a (mod p) tiene soluci´on si y s´olo si a 2 ≡ 1 (mod p). Por otra parte, por el Teorema de Euler se tiene ap−1 ≡ 1 (mod p), es decir, p|ap−1 − 1 = (a lo que implica que a

p−1 2

p−1 2

)2 − 1 = (a

≡ 1 (mod p) o a

p−1 2

p−1 2

+ 1)(a

p−1 2

− 1)

≡ −1 (mod p).

Definici´ on. Si mcd(a, p) = 1, se dice que a es un resto cuadr´ atico m´ odulo p si la congru2 encia x ≡ a (mod p) tiene soluci´ on. Se llama s´ımbolo de Legendre al n´ umero   ( 1 si es un resto cuadr´atico m´odulo p a = p −1 si a no es un resto cuadr´atico m´odulo p  Para p = 2, se tiene obviamente a2 = 1 para cualquier a impar. Si p es un n´ umero primo impar, por el Teorema 5.2, se tiene   p−1 a a 2 ≡ (mod p) p Proposici´ on 5.3. El s´ımbolo de Legendre verifica las siguientes propiedades:     (i) Si a ≡ b (mod p) entonces ap = pb .      (ii) ab = ap pb . p  2   (iii) ap = 1, y en particular p1 = 1   p−1 (iv) −1 = (−1) 2 si p es impar. p Demostraci´ on: La parte (i) es trivial. La parte (ii) (si p es impar, ya que el caso p = 2 es trivial) se sigue de que      p−1 p−1 p−1 ab a b ≡ (ab) 2 = a 2 b 2 ≡ (mod p) p p p      lo que implica la igualdad de ab y ap pb , ya que ambos valen 1 o −1. La parte (iii) es p (ii) cuando a = b, y la parte (iv) es el criterio de Euler (n´otese que es tambi´en equivalente al Teorema 2.19). Observaci´ on 5.4. La parte (ii) es mucho m´as potente de lo que aparenta. Por ejemplo,  podr´ıa parecer que no sirve para calcular 23 31 , pero basta por ejemplo escribir         3 23 −8 (−1)23 −1 2 = = = . 31 31 31 31 31 46

   2 2 3 Por (iv), tenemos −1 = = −1, mientras que por (iii) tenemos 31  31 31 , por lo que  2 23 finalmente tenemos 31 = − 31 , que es m´as f´acil de calcular. En efecto, basta utilizar el criterio de Euler, y observar que 2 por lo que

2 31



31−1 2

= 215 = (25 )3 = 323 ≡ 13 = 1 (mod 31)

= 1 y por tanto

23 31



= −1.

Teorema 5.5. Sea p un n´ umero primo impar y sea a ∈ Z tal que mcd(a, p) = 1. Para p−1 cada k = 1, 2, . . . , 2 , sea ka = qk p + rk la divisi´on de ka entre p. Separamos los restos r1 , r2 , . . . , r p−1 de la forma 2

0 } ∪ {r100 , . . . , rn00 } {r1 , . . . , r p−1 } = {r10 , . . . , rm 2

0 donde r10 , . . . , rm son los restos menores que Entonces:

p 2

y r100 , . . . , rn00 son los restos mayores que

p 2.

00 00 0 0 (i) {1, 2 . . . , p−1 2 } = {r1 , . . . , rm } ∪ {p − r1 , . . . , p − rn }   (ii) (Lema de Gauss) ap = (−1)n , donde n es como antes, es decir, el n´ umero de elep mentos de a, 2a, 3a, . . . , p−1 2 a cuyo resto al dividir por p es mayor que 2 .

Demostraci´ on: Usaremos repetidamente la Proposici´on 2.5. Como el conjunto −

p−1 p−1 p−3 ,− , . . . , −1, 0, 1, . . . , 2 2 2

est´a formado por p n´ umeros consecutivos, es un sistema completo de restos m´odulo p. Por tanto, tambi´en los n´ umeros −

p−1 p−3 p−1 a, − a, . . . , −a, 0, a, . . . , a 2 2 2

forma un sistema completo de restos m´odulo p. Como los n´ umeros a, . . . , p−1 2 a son con0 0 gruentes m´ odulo p (en alg´ un orden), a r1 , . . . , r p−1 , es decir, a r1 , . . . , rm , r100 , . . . , rn00 , se 2 tendr´a que los n´ umeros 0 0 −r10 , . . . , −rm , −r100 , . . . , −rn00 , 0, r10 , . . . , rm , r100 , . . . , rn00

forman un sistema completo de restos m´odulo p. Esto implica en particular que los n´ umeros 0 0 00 00 r1 , . . . , rm , −r1 , . . . , −rn son incongruentes m´odulo p cuando se toman dos a dos, y lo 0 , p−r100 , . . . , p−rn00 , que ser´an por tanto distintos. mismo es cierto con los n´ umeros r10 , . . . , rm Como hay m + n = p+1 umeros, y todos ellos est´an en el conjunto {1, 2 . . . , p−1 2 de estos n´ 2 }, se sigue inmediatamente la parte (i). 47

Multiplicando entonces todos estos n´ umeros, se obtiene:   p−1 0 0 00 ! = r10 . . . rm (p − r100 ) . . . (p − rn00 ) ≡ (−1)n r10 . . . rm r1 . . . rn00 (mod p). 2 y, como por otra parte, r10

0 00 . . . rm r1

. . . rn00

p−1 ≡ a(2a)(3a) . . . ( a) = 2

juntando ambas congruencias, se sigue a

p−1 2



 p−1 p−1 !a 2 (mod p), 2

≡ (−1)n (mod p), lo que prueba (ii).

Teorema 5.6. Si p es un primo impar, entonces   ( 1 si p ≡ 1 (mod 8) o p ≡ 7 (mod 8) 2 = p −1 si p ≡ 3 (mod 8) o p ≡ 5 (mod 8) Demostraci´ on: Usamos el lema de Gauss, y tenemos que ver cu´antos de los p−1 umeros 2 n´ p 2, 4, . . . , p − 1 son mayores que 2 . Lo haremos analizando separadamente todos los posibles restos de p m´ odulo 8: –Si p = 8k +1, entonces el conjunto 2, 4, . . . , p−1 = 8k tiene tenemos 4k elementos, de los que los menores de p2 = 4k+ 12 son 2, 4, . . . , 4k, es decir, 2k. Por tanto, n = 4k−2k = 2k,

que es par, luego

2 p

= 1.

–Si p = 8k + 3, entonces el conjunto 2, 4, . . . , p − 1 = 8k + 2 tiene tenemos 4k + 1 elementos, de los que los menores de p2 = 4k + 23son  2, 4, . . . , 4k, es decir, 2k. Por tanto, n = (4k + 1) − 2k = 2k + 1, que es impar, luego

2 p

= −1.

–Si p = 8k + 5, entonces el conjunto 2, 4, . . . , p − 1 = 8k + 4 tiene tenemos 4k + 2 elementos, de los que los menores de p2 = 4k + 52 son 2, 4, . . . ,4k + 2, es decir, 2k + 1. Por tanto, n = (4k + 2) − (2k + 1) = 2k + 1, que es impar, luego

2 p

= −1.

–Si p = 8k + 7, entonces el conjunto 2, 4, . . . , p − 1 = 8k + 6 tiene tenemos 4k + 3 4k + 2, es decir, 2k + 1. Por elementos, de los que los menores de p2 = 4k + 72 son 2, 4, . .. , 

tanto, n = (4k + 3) − (2k + 1) = 2k + 2, que es par, luego

2 p

= 1.

Corolario 5.7. Existen infinitos n´ umeros primos de la forma 8k + 7. Demostraci´ on: Si p1 , p2 , . . . , pr fueran todos los n´ umeros primos de la forma 8k + 7, 2 consideramos el n´ umero a = 8(p1 p2 . . . pr ) −1. Sea p un divisor primo de a (necesariamente p ser´a impar). Entonces, (4p1 p2 . . . pr )2 ≡ 2 (mod p), por lo que, por el Teorema 5.6, se tiene p ≡ 1 (mod 8) o p ≡ 7 (mod 8). Como a ≡ 7 (mod 8), no todos los divisores de a 48

pueden ser de la forma 8k + 1. Por tanto, a es divisible por alg´ un primo de la forma 8k + 7, es decir, por alg´ un pi , lo que es absurdo. Teorema 5.8. Si p es un primo impar, entonces 

−2 p

(

 =

1 si p ≡ 1 (mod 8) o p ≡ 3 (mod 8) −1 si p ≡ 5 (mod 8) o p ≡ 7 (mod 8)

Demostraci´ on: Basta combinar el Teorema 5.6 con las partes (iii) y (iv) de la Proposici´ on 5.3. Lo hacemos seg´ un el resto de p m´odulo 8:   –Si p ≡ 1 (mod 8), entonces por el Teorema 5.6, p2 = 1, mientras que por otra   parte p ≡ 1 (mod 4) luego por la Proposici´on 5.3(iv) se tiene −1 = 1. Por tanto, de la p   Proposici´on 5.3(iv) se sigue −2 = 1. p   –Si p ≡ 3 (mod 8), entonces por el Teorema 5.6, p2 = −1, mientras que por otra   parte p ≡ 3 (mod 4) luego por la Proposici´on 5.3(iv) se tiene −1 = −1. Por tanto, de p   = 1. la Proposici´ on 5.3(iv) se sigue −2 p   –Si p ≡ 5 (mod 8), entonces por el Teorema 5.6, p2 = −1, mientras que por otra   parte p ≡ 1 (mod 4) luego por la Proposici´on 5.3(iv) se tiene −1 = 1. Por tanto, de la p   = −1. Proposici´on 5.3(iv) se sigue −2 p   –Si p ≡ 7 (mod 8), entonces por el Teorema 5.6, p2 = 1, mientras que por otra   parte p ≡ 3 (mod 4) luego por la Proposici´on 5.3(iv) se tiene −1 = −1. Por tanto, de p   la Proposici´ on 5.3(iv) se sigue −2 = −1. p Corolario 5.9. Existen infinitos n´ umeros primos de la forma 8k + 3. Demostraci´ on: Como siempre, supogamos que p1 , p2 , . . . , pr son todos los n´ umeros primos 2 de la forma 8k + 3. Consideramos el n´ umero a = (p1 p2 . . . pr ) + 2. Entonces, si p es un divisor primo de a (necesariamente impar), se tendr´a que (p1 p2 . . . pr )2 ≡ −2 (mod p), y por el Teorema 5.8 se tendr´ a que p es congruente con 1 o 3 m´odulo 8. Como a es de la forma 8k + 3 (ya que el cuadrado de cada pi es de la forma 8k + 1), no todos los divisores de a son de la forma 8k + 1. Por tanto, a es divisible por alg´ un n´ umero primo de la forma 8k + 3, es decir, por alg´ un pi ; lo que es imposible. Damos a continuaci´ on un modo pr´actico de calcular los s´ımbolos de Legendre: 49

Lema 5.10. Si p es un primo impar, a ∈ Z es impar y mcd(a, p) = 1, entonces   P p−1 ka a 2 = (−1) k=1 [ p ] . p Demostraci´ on: Usando las notaciones delh Teorema 5.5, tenemos que demostrar, por la i P p−1 ka 2 parte (ii) de dicho teorema, que n y k=1 p tienen la misma paridad. Como a y p son impares, y tomando congruencias m´odulo 2, tendremos: p−1

p−1

p−1

p−1

p−1

 2  X ka

 2  X ka

2 X

2 X

2 X

k=1

p

≡p

k=1

p

=p

qk =

k=1

(ka−rk ) =

k=1

0 ka−(r10 +. . .+rm )−(r100 +. . .+rn00 ) ≡

k=1

p−1



2 X

0 k − (r10 + . . . + rm ) − (r100 + . . . + rn00 ) (mod 2).

k=1 0 Por el Teorema 5.5(i), {r10 , . . . , rm , p − r100 , . . . , p − rn00 } = {1, 2, . . . , p−1 2 }, y por tanto p−1

0 (r10 + . . . + rm ) + ((p − r100 ) + . . . + (p − rn00 )) =

2 X

k.

k=1

Sustituyendo en la congruencia anterior, obtenemos p−1

 2  X ka k=1

p

≡ ((p − r100 ) + . . . + (p − rn00 )) − (r100 + . . . + rn00 ) = pn − 2(r100 + . . . + rn00 ) ≡ n (mod 2)

que es lo que quer´ıamos demostrar. El resultado anterior nos permite demostrar el siguiente resultado fundamental, que servir´a para calcular cualquier s´ımbolo de Legendre: Teorema 5.11 (Ley de reciprocidad cuadr´atica). Si p, q son primos impares distintos, entonces    p−1 q−1 p q = (−1) 2 2 . q p Demostraci´ on: Consideramos el conjunto de pares de la forma (k, l), con k ∈ {1, 2, . . . , p−1 2 } q−1 p−1 q−1 y l ∈ {1, 2, . . . , 2 }. Evidentemente, hay 2 2 de dichos pares. Dividimos ahora el mencionado conjunto en dos subconjuntos: –En primer lugar, consideramos los pares (k, l)htales i que kq ≤ lp. Para cada l ∈ lp {1, 2, . . . , q−1 umero de pares 2 } los posibles valores de k son 1, 2, . . . , q . Por tanto, el n´ h i q−1 P 2 lp de esta forma es l=1 q . 50

–Consideramos por otra parte los pares (k, l) htales i que kq ≥ lp. Para cada k ∈ kq umero de pares {1, 2, . . . , p−1 2 } los posibles valores de l son 1, 2, . . . , p . Por tanto, el n´ i h p−1 P 2 kq de esta forma es k=1 p . Observemos que no hay pares que est´en en ambos subconjuntos, ya que por ejemplo p no divide a ning´ un kq. Por tanto, umeroh total iel n´ i de pares es igual, por una parte a p−1 h q−1 P P p−1 q−1 kq lp 2 2 k=1 l=1 2 2 , y por otra parte a p + q . Tomando potencias de −1 y usando el Lema 5.10, tenemos (−1)

p−1 q−1 2 2

= (−1)

P p−1 2 k=1

   P q−1 lp P p−1 ka P q−1 lp q 2 2 2 [ ka [ q ] = (−1) k=1 [ p ] (−1) l=1 [q] = p p ]+ l=1 . q p

Observaci´ on 5.12. La ley de reciprocidad cuadr´atica es el resultado del que m´ as demostraciones se conocen. En http://www.rzuser.uni-heidelberg.de/∼hb3/fchrono.html hay una lista de ellas, y hasta el a˜ no 2007 se llevan contabilizadas 224 distintas. Corolario 5.13. Sea p > 3 un n´ umero primo. Entonces: (   1 si p ≡ 1 (mod 12) o p ≡ 11 (mod 12) (i) p3 = −1 si p ≡ 5 (mod 12) o p ≡ 7 (mod 12) (   1 si p ≡ 1 (mod 3) (ii) −3 = p −1 si p ≡ 2 (mod 3)    p−1 Demostraci´ on: Por la ley de reciprocidad cuadr´atica, tenemos p3 = (−1) 2 p3 . Por     p−1 −1 2 . Por tanto, −3 otra parte, por la Proposici´ on 5.3(iv), tenemos p = (−1) = p     3 −1 = p3 , que claramente viene dado por la f´ormula de (ii). La parte (i) se p p demuestra como el Corolario 5.8 a partir de (i). Corolario 5.14. Existen infinitos n´ umeros primos de la forma 12k + 11. Demostraci´ on: Supongamos en primer lugar que p1 , . . . , pr sean todos los n´ umeros primos 2 de la forma 12k+11. Consideramos a = 12(p1 . . . pr ) −1. Entonces, si p es un divisor primo de a (necesariamente p > 3), se tiene que 6p1 . . . pr es una soluci´on de x2 ≡ 3 (mod p). El Corolario 5.13(i) implica entonces que p es de la forma 12k + 1 o 12k + 11. Como a es de la forma 12k + 11, necesariamente tendr´a alg´ un divisor primo de la forma 12k + 11, es decir, alg´ un pi , lo que es absurdo. 51

Ejercicio 5.15. Demostrar que existen infinitos n´ umeros primos de la forma 3k + 1. M´ as a´ un, demostrar que existen infinitos n´ umeros primos de la forma 12k + 7. Ejemplo 5.16. Veamos en un ejemplo pr´actico c´omo la Ley de Reciprocidad Cuadr´ atica  permite calcular cualquier s´ımbolo de Legendre. Supongamos que queremos calcular 17 41 .   17 41 Por la Ley de Reciprocidad Cuadr´atica, podremos escribir 41 = 17 y, como se tiene   7 41 ≡ 7 (mod 17), tambi´en tendremos 41 = 17 . Por tanto, hemos reducido calcular un 17   17 7 s´ımbolo 41 a calcular 17 , que involucra n´ umeros m´as peque˜ nos. Es claro que podemos    7 17 3 seguir el proceso, escribiendo ahora 17 = 7 = 7 , que es igual a −1 por el Corolario  5.13. Por tanto, 17 41 = −1. Aprovechamos para hacer el mismo c´alculo con un truco distinto. Podemos escribir         2   17 −24 −2 · 22 · 3 −2 2 3 = = = = 1 · 1 · (−1) = −1 41 41 41 41 41 41 donde los tres u ´ltimos s´ımbolos han sido calculados respectivamente con el Teorema 5.6, la Proposici´ on 5.3(iii) y el Corolario 5.13(i). Veamos, para terminar el cap´ıtulo, que el s´ımbolo de Legendre tambi´en mide cu´ ando un n´ umero es un resto cuadr´ atico m´odulo la potencia de un primo: Teorema 5.17. Si p es un primo impar, k ≥  1,  y mcd(a, p) = 1, entonces la congruencia x2 ≡ a (mod pk ) tiene soluci´ on si y s´olo si ap = 1. Adem´as, en tal caso, el n´ umero de soluciones m´ odulo pk es dos. 2 k 2 Demostraci´ on: Obviamente,   si x ≡ a (mod p ) tiene soluci´on, tambi´en la tiene  x ≡ a (mod p), por lo que ap = 1. Hay que demostrar entonces el rec´ıproco, que si ap = 1

entonces x2 ≡ a (mod pk ) tiene soluci´on. Lo haremos por inducci´on sobre k, siendo trivial el caso k = 1. Supongamos entonces que x2 ≡ a (mod pk ) tiene soluci´on x0 y encontremos una soluci´on de x2 ≡ a (mod pk+1 ). Como las soluciones de esta segunda congruencia son soluciones de la primera, en realidad buscamos enteros de la forma x0 + ypk que sean soluci´on de la segunda congruencia. x20

Que x0 sea soluci´ on de la primera congruencia quiere decir que se puede escribir k = a + bp , para alg´ un b ∈ Z. Por tanto, tendremos (x0 + ypk )2 = x20 + 2x0 ypk + y 2 p2k = a + (2x0 y + b)pk + p2k

de donde se deduce que (x0 + ypk )2 ≡ a (mod pk+1 ) si y s´olo si p|2x0 y + b. Como mcd(2, p) = 1, la congruencia 2x0 y +b ≡ 0 (mod p) tiene soluci´on, y tomando una soluci´ on k 2 k+1 y se tiene que x0 + yp es una soluci´on de x ≡ a (mod p ). 52

El hecho de que haya exactamente dos soluciones m´odulo pk es consecuencia del Teorema 4.17, ya que pk tiene ra´ıces primitivas. Puede hacerse tambi´en directamente, ya que, si x0 es una soluci´ on, entonces cualquier otra soluci´on x verifica x2 ≡ x20 (mod pk ), es decir, pk |x2 −x20 = (x+x0 )(x−x0 ). No puede ocurrir simult´aneamente p|x+x0 y p|x−x0 , ya que entonces p|(x + x0 ) − (x − x0 ) = 2x0 y, como p es un primo impar, p|x0 , lo que contradice que x20 ≡ 1 (mod pk ). Por tanto mcd(pk , x + x0 ) = 1 o bien mcd(pk , x − x0 ) = 1, por lo que pk |x − x0 o bien pk |x + x0 . Por tanto, x ≡ x0 (mod pk ) o bien x ≡ −x0 (mod pk ), y ´estas son las u ´nicas soluciones.

Ejemplo 5.18. Veamos c´ omo se puede seguir la demostraci´on anterior para resolver  3 congruencias cuadr´ aticas. Por ejemplo, como 13 = 1 (por el Corolario 5.13(i)), entonces 2 2 la ecuaci´on x ≡ 3 (mod 13 ) tendr´a dos soluciones m´odulo 132 . En primer lugar, debemos resolver la ecuaci´ on x2 ≡ 3 (mod 13), y se ve a ojo que x ≡ ±4 (mod 13) son las soluciones. Trabajamos por elemplo con la soluci´on x0 = 4, y tenemos que buscar ahora los valores de y para los que 4 + 13y es soluci´on de x2 ≡ 3 (mod 132 ). Operando (mejor as´ı que intentar aprenderse de memoria la f´ormula) queremos 16 + 104y + 132 y 2 ≡ 3 (mod 132 ), es decir, 13(1 + 8y) ≡ 0 (mod 132 ), que es equivalente a 1 + 8y ≡ 0 (mod 13). Congruencias lineales de este tipo ya aprendimos a resolverlas en el Cap´ıtulo 2, y se comprueba f´acilmente que y ≡ 8 (mod 13) es la soluci´ on. Por tanto, x ≡ 4 + 13 · 8 = 108 (mod 132 ) es una soluci´on. Para calcular la segunda soluci´on no hace falta empezar con x0 = −4, ya que necesariamente x ≡ −108 ≡ 61 (mod 132 ) es la otra soluci´on. Nos queda ver el caso de restos cuadr´aticos m´odulo una potencia de 2, que estudiamos a continuaci´ on: Teorema 5.19. Sea a un n´ umero impar. Entonces: (i) La congruencia x2 ≡ a (mod 2) tiene siempre soluci´on. (ii) La congruencia x2 ≡ a (mod 4) tiene soluci´on si y s´olo si a ≡ 1 (mod 4). (iii) Si k ≥ 3, la congruencia x2 ≡ a (mod 2k ) tiene soluci´on si y s´olo si a ≡ 1 (mod 8). Adem´ as, en este caso, la congruencia tiene exactamente dos soluciones m´odulo 2k−1 Demostraci´ on: La parte (i) es obvia, y la parte (ii) sigue del hecho de que el cuadrado de cualquier n´ umero impar es de la forma 4k + 1. Para parte (iii) observamos que, como el cuadrado de un n´ umero impar es siempre de la forma 8k + 1, entonces si la congruencia 2 k x ≡ a (mod 2 ) tiene soluci´ on, entonces a ≡ 1 (mod 8). Demostraremos el rec´ıproco por inducci´ on sobre k, siendo el caso k = 3 trivial (las soluciones m´odulo 4 son 1 y 3). Supongamos entonces que x2 ≡ a (mod 2k ) tiene soluci´on, es decir, que existe x0 ∈ Z tal 53

que x20 = a + 2k b para alg´ un b ∈ Z. Veamos que existe una soluci´on de x2 ≡ a (mod 2k+1 ) de la forma x = x0 + 2k−1 y. En efecto, como (x0 + 2k−1 y)2 = x20 + 2k x0 y + 22k−2 y 2 = a + 2k (b + x0 y) + 22k−2 y 2 y x0 es impar, si tomamos y de la misma paridad que b tendremos que x0 + 2k−1 y es soluci´on de x2 ≡ a (mod 2k+1 ). Adem´as, si x0 es una soluci´ on de x2 ≡ a (mod 2k ), entonces para cualquier otra soluci´on x se verifica 2k |x2 − x20 = (x + x0 )(x − x0 ) y, como el m´ aximo com´ un divisor de x + x0 y x − x0 es divisible por 2 pero no por 4, se sigue que x ≡ −x0 (mod 2k−1 ) o x ≡ x0 (mod 2k−1 ).

54

6. Ecuaciones diof´anticas Se llama ecuaci´ on diof´ antica a una ecuaci´on cuyas variables son n´ umeros enteros. La n n m´as conocida es la llamada ecuaci´ on de Fermat, que es x + y = z n . Fermat asegur´ o que, si n ≥ 3, las u ´nicas posibles soluciones son aqu´ellas en que alguno de los x, y, z es ´ cero. Este resultado, conocido como Ultimo Teorema de Fermat, ha sido demostrado s´ olo recientemente por Wiles. En esta secci´on estudiaremos los casos m´as sencillos de esta ecuaci´on, as´ı como otras ecuaciones diof´anticas similares. ´ Empezamos con el caso n = 2, que no entra dentro del Ultimo Teorema de Fermat, ya que en este caso s´ı que existe soluci´on, y de hecho infinitas, como demostraremos enseguida, viendo qu´e estructura tienen. Definici´ on. Se llama terna pitag´ orica al conjunto de tres enteros x, y, z tales que x2 +y 2 = z 2 . El nombre viene dado porque, si x, y, z > 0 forman una terna pitag´orica, por el teorema de Pit´ agoras existe un tri´angulo rect´angulo que tiene a x e y como longitudes de sus catetos y a z como longitud de la hipotenusa. Un tri´angulo as´ı (es decir, rect´angulo con las longitudes de los lados enteras) se llama tri´ angulo pitag´ orico. Lema 6.1. Sea x, y, z una terna pitag´orica y d = mcd(x, y, z). Entonces: (i) d = mcd(x, y) = mcd(x, z) = mcd(y, z). (ii) Si escribimos x = dx0 , y = dy 0 , z = dz 0 , entonces x0 , y 0 , z 0 es una terna pitag´orica. Demostraci´ on: De (i) demostraremos s´olo d = mcd(x, y), siendo id´enticas las demostraciones de las otras igualdades. Sea d = mcd(x, y). Es evidente que mcd(x, y, z) divide a d, por lo que basta demostrar que d divide a mcd(x, y, z). Para ello, observamos que d|x y d|y implica d2 |x2 y d2 |y 2 . Por tanto, d2 |x2 + y 2 = z 2 , de donde se deduce d|z (por el Teorema 1.18). De aqu´ı se concluye d| mcd(x, y, z). La parte (ii) es inmediata.

Definici´ on. Una terna pitag´ orica se dice que es primitiva si mcd(x, y, z) = 1, o equivalentemente cualquiera de los mcd(x, y), mcd(x, z), mcd(y, z) es 1. Lema 6.2. Sea x, y, z una terna pitag´orica primitiva. Entonces entre x, y, z hay exactamente un n´ umero par, que es o bien x o bien y. Demostraci´ on: Es evidente que entre x, y, z hay exactamente un n´ umero par. Si fuera z, 2 2 2 entonces xx, y ser´ıan impares, luego x , y ≡ 1 (mod 8), luego z = x2 + y 2 ≡ 2 (mod 8), lo que es absurdo, porque z 2 debe ser divisible por 4. 55

Teorema 6.3. Sea x, y, z una terna pitag´orica primitiva con x, y, z > 0 y x par. Entonces existen s, t ∈ Z tales que 0 < t < s, mcd(s, t) = 1, s 6≡ t (mod 2) y  x =2st   y =s2 − t2   z =s2 + t2 Demostraci´ on: Observamos en primer lugar que mcd(z +y, z −y) = 2. En efecto, cualquier divisor com´ un de z + y y z − y lo es tambi´en de su suma y su diferencia, es decir, de 2z y 2y Como mcd(y, z) = 1, se sigue el u ´nico posible divisor com´ un de z + y y z − y es 2, que efectivamente es un divisor, ya que y, z son impares. Podemos escribir entonces z + y = 2a y z − y = 2b, con a, b enteros tales que mcd(a, b) = 1. Por tanto ( y =a−b z =a+b p

Obs´ervese que, como y < x2 + y 2 = z, entonces b > 0 y, como y > 0, tambi´en a > b. Por otra parte, como x es par, podemos escribir x = 2c. De la igualdad x2 = z 2 − y 2 = (z + y)(z − y) se deduce entonces c2 = ab, luego, por el Corolario 1.17(ii), existen enteros positivos s, t tales que a = s2 y b = t2 (y por tanto x = 2st, y = s2 −t2 y z = s2 +t2 ). Como mcd(a, b) = 1, necesariamente mcd(s, t) = 1 y, como a > b, ser´a s > t. Finalmente,como ni y ni z son pares, necesariamente s y t tienen distinta paridad. Teorema 6.4. La ecuaci´ on x4 + y 4 = z 2 no tiene soluci´on entera positiva. Demostraci´ on: Sea S el conjunto de los enteros positivos z para los que existen x, y > 0 2 tales que z = x4 +y 4 . Supongamos, por reducci´on al absurdo, que la ecuaci´on x4 +y 4 = z 2 tiene soluci´ on entera positiva. Esto quiere decir que el conjunto S es no vac´ıo, y por el principio del buen orden existir´ a un elemento m´ınimo de S. Sea pues una soluci´on x, y, z con S m´ınimo en S. Los enteros x2 , y 2 , z forman entonces una terna pitag´orica, que veremos a continuaci´ on que es primitiva. En efecto, si mcd(x2 , y 2 ) 6= 1, entonces mcd(x, y) = d > 1. Si escribimos x = dx0 , y = dy 0 , entonces d4 |x4 + y 4 = z 2 , luego d2 |z y podremos escribir z = d2 z 0 , con z 0 ∈ Z≥1 . Entonces, haciendo esas sustituciones en la igualdad x4 + y 4 = z 2 queda d4 x04 + d4 y 04 = d4 z 02 , de donde se deduce x04 + y 04 = z 02 . Por tanto, z 0 ∈ S, lo que es imposible porque z 0 < dz 0 = z y z era m´ınimo en S. Sin p´erdida de generalidad, podemos suponer que x2 es par e y 2 , z impares. Por el Teoorema 6.3, se tendr´ a que existen s, t enteros positivos primos entre s´ı y de distinta paridad tales que  2 x =2st   y 2 =s2 − t2   z =s2 + t2 56

En particular, t2 +y 2 = s2 , luego t, y, s es tambi´en una terna pitag´orica primitiva y adem´ as necesariamente s es impar y t es par. Usando de nuevo el Teorema 6.3, concluimos que podemos encontrar u, v primos entre s´ı y de distinta paridad tales que  t =2uv   y =u2 − v 2   s =u2 + v 2 Como s es impar, mcd(2t, s) = 1, luego de la igualdad x2 = (2t)s y del Corolario 1.17(ii) deducimos que existen t0 , s0 ∈ Z≥1 tales que ( 2t =(2t0 )2 s =s02 Finalmente, haciendo la sustituci´ on t = 2t02 en la igualdad t = 2uv llegamos a t02 = uv, y usando de nuevo el Corolario 1.17(ii) y mcd(u, v) = 1, obtenemos que existen u0 , v 0 ∈ Z≥1 tales que ( u =u02 v =v 02 Por tanto, la igualdad s = u2 + v 2 se convierte en s02 = u04 + v 04 , luego s0 ∈ S. La desigualdad s0 ≤ s02 = s ≤ s2 < s2 + t2 = z produce entonces una contradicci´ on con el hecho de que z era m´ınimo en S. ´ Corolario 6.5. El Ultimo Teorema de Fermat es cierto para n = 4. Demostraci´ on: En efecto, si la ecuaci´on x4 + y 4 = z 4 tuviera una soluci´on entera x = a, y = b, z = c con abc 6= 0, entonces x = |a|, y = |b|, z = c2 ser´ıa una soluci´on entera positiva de x4 + y 4 = z 2 , lo que es imposible por el Teorema 6.4. ´ Observaci´ on 6.6. El corolario anterior reduce el Ultimo Teorema de Fermat al caso en ´ que el exponente n es un n´ umero primo. En efecto, supongamos demostrado el Ultimo Teorema de Fermat para exponentes primos. Entonces, para cada n > 2, o bien n tiene un divisor primo impar o es una potencia de 2. En el primer caso, si n = pk, con p > 2 primo, entonces la ecuaci´ on de Fermat se puede escribir como (xk )p + (y k )p = (z k )p , que no tiene soluci´on no trivial, porque por hip´ otesis xp + y p = z p no la tiene. Si, en cambio, n es una potencia de 2, como n > 2, necesariamente se puede escribir n = 4k (adem´as, k ser´ıa una potencia de 2, aunque esto no nos interesa). En este caso, se puede escribir la ecuaci´ on k 4 k 4 k 4 k k k como (x ) + (y ) = (z ) , pero el Corolario 6.5 implica que entonces x y z = 0, es decir, xyz = 0. Veamos otra ecuaci´ on diof´ antica m´as restrictiva que la de Fermat de grado cuatro: 57

Teorema 6.7. La ecuaci´ on x4 + y 2 = z 4 no tiene soluci´on entera positiva. Demostraci´ on: Como en el Teorema 6.4, suponemos por reducci´on al absurdo que exista soluci´on y tomamos aqu´ella con z m´ınimo, lo que implicar´a en particular que x2 , y, z 2 es una terna pitag´ orica primitiva. Tenemos que distinguir ahora dos casos seg´ un la paridad de x e y. Si x es impar e y es par se tendr´a, por el Teorema 6.3, que existen s, t ∈ Z tales que  2 x =s2 − t2   y =2st   2 z =s2 + t2 y, multiplicando la primera y u ´ltima igualdad se obtiene x2 z 2 = (s2 − t2 )(s2 + t2 ) = s4 − t4 , y en particular t4 + (xz)2 = s4 . Como s2 < s2 + t2 = z 2 , se concluye que s < z, lo que contradice la minimalidad de z. Nos queda entonces el caso en que x2 es par e y impar. Por el Teorema 6.3, se tendr´ a que existen s, t enteros positivos primos entre s´ı y de distinta paridad tales que ( 2 x =2st z 2 =s2 + t2 donde no decimos si y = s2 − t2 o y = t2 − s2 ; por tanto, s y t juegan un papel sim´etrico, y podemos suponer que s es par y t impar. Como la igualdad s2 + t2 = z 2 nos dice que s, t, z es una terna pitag´ orica (primitiva porque mcd(s, t) = 1), por el Teorema 6.3, podemos encontrar u, v primos entre s´ı y de distinta paridad tales que  s =2uv   t =u2 − v 2   z =u2 + v 2 Como t es impar, mcd(2s, t) = 1, luego de la igualdad x2 = (2s)t y del Corolario 1.17(ii) deducimos que existen s0 , t0 ∈ Z≥1 tales que ( 2s =(2s0 )2 t =t02 Finalmente, haciendo la sustituci´ on s = 2s02 en la igualdad s = 2uv llegamos a s02 = uv, y usando de nuevo el Corolario 1.17(ii) y mcd(u, v) = 1, obtenemos que existen u0 , v 0 ∈ Z≥1 tales que ( u =u02 v =v 02 58

Por tanto, la igualdad t = u2 − v 2 se convierte en v 04 + t02 = u04 , y claramente u0 ≤ u02 = u ≤ u2 < u2 + v 2 = z lo que contradice la minimalidad de z. Corolario 6.8. El ´ area de un tri´ angulo pitag´orico no puede ser un cuadrado perfecto. Demostraci´ on: Si x, y, z son las longitudes de los lados de un tri´angulo pitag´orico, el ´ area 1 2 del mismo es 2 xy, luego si es un cuadrado perfecto existir´a u ∈ Z≥1 tal que xy = 2u . Por tanto tendremos (x + y)2 = x2 + 2xy + y 2 = z 2 + 4u2 (x − y)2 = x2 − 2xy + y 2 = z 2 − 4u2 y multiplicando llegamos a (x2 − y 2 )2 = z 4 − (2u)4 que contradice el Teorema 6.7. Observaci´ on 6.9. No podemos seguir poniendo m´as restricciones a la ecuaci´on de Fermat de grado cuatro sin llegar ya a soluciones no triviales. Obs´ervese que en los Teoremas 6.4 y 6.7 hemos cambiado un exponente 4 por un 2. Si cambi´aramos dos exponentes 4 por un 2, es decir, si consider´ aramos las ecuaciones x2 + y 4 = z 2 y x2 + y 2 = z 4 , ya tendr´ıamos infinitas soluciones. Basta mirar en la expresi´on de las ternas pitag´oricas primitivas dada en el Teorema 6.3 para ver que se obtienen infinitas soluciones de x2 + y 4 = z 2 si existen infinitas soluciones de  x =2st   y 2 =s2 − t2   z =s2 + t2 lo que es cierto porque existen infinitas soluciones de y 2 = s2 − t2 . De la misma forma, existir´an infinitas soluciones de x2 + y 2 = z 4 si existen infinitas soluciones de  x =2st   y =s2 − t2   2 z =s2 + t2 lo que vuelve a ser cierto, porque basta encontrar infinitas soluciones de z 2 = s2 + t2 . Las soluciones de la ecuaci´ on x2 + y 2 = z 2 se pueden interpretar desde otro punto de vista: ¿cu´ ando un cuadrado perfecto z 2 se puede escribir como suma de dos cuadrados 59

x2 + y 2 ? O bien, ¿cu´ ando un cuadrado perfecto x2 se puede escribir como diferencia de dos cuadrados z 2 − y 2 ? A continuaci´ on queremos estudiar un problema m´as general: ¿cu´ ando un n´ umero se puede escribir como suma o diferencia de dos cuadrados?. Empezamos con el caso de la diferencia de cuadrados, que resulta sorprendentemente f´acil: Teorema 6.10. Un n´ umero n ∈ Z se puede escribir como n = x2 − y 2 si y s´ olo si n 6≡ 2 (mod 4). Demostraci´ on: Supongamos primero que n = x2 − y 2 . Como x2 , y 2 ≡ 0, 1 (mod 4), se sigue que n = x2 − y 2 ≡ 0, 1, 3 (mod 4). Rec´ıprocamente, supongamos que n 6≡ 2 (mod 4). Distinguimos dos casos: n−1 2 2 -Si n es impar, entonces escribimos n = ( n+1 2 ) −( 2 ) . n−4 2 2 -Si n es par, entonces es m´ ultiplo de 4, y podemos escribir n = ( n+4 4 ) −( 4 ) .

El caso de la suma de dos cuadrados es m´as complicado. Empezamos con una observaci´on que ser´ a muy u ´til: Lema 6.11. El producto de dos sumas de dos cuadrados es tambi´en una suma de dos cuadrados. Demostraci´ on: En efecto, basta observar que (x2 + y 2 )(z 2 + w2 ) = (xz + yw)2 + (xw − yz)2 .

A la vista del resultado anterior, parece l´ogico estudiar primero qu´e n´ umeros primos son sumas de dos cuadrados. En este caso, tenemos el siguiente primer resultado: Proposici´ on 6.12. Si p es un n´ umero primo y x, y ∈ Z, entonces: √ (i) p = x2 + y 2 si y s´ olo si p|x2 + x2 y 0 < |x|, |y| < p. (ii) Si p = x2 + y 2 = z 2 + w2 , entonces o bien |x| = |z|, |y| = |w| o bien |x| = |w|, |y| = |z|. Demostraci´ on: Para demostrar (i), supongamos primero p = x2 + y 2 , de donde se deduce inmediatamente p|x2 + x2 . Adem´ as, no puede ser x = 0, ya que entonces p = y 2 , y no podr´ıa ser un n´ umero primo. De la misma forma, y 6= 0, por lo que |x|, |y| > 0. De aqu´ı √ √ se deduce tambi´en x2 < x2 + y 2 = p, luego |x| < p y, an´alogamente, |y| < p. √ Rec´ıprocamente, supongamos p|x2 +x2 y 0 < |x|, |y| < p. En particular, x2 +y 2 = np √ √ para alg´ un n > 0. Por otra parte, np = x2 + y 2 < ( p)2 + ( p)2 = p + p = 2p, luego n < 2, es decir, n = 1 y x2 + y 2 = p. Para demostrar (ii), es claro que basta suponer x, y, z, w > 0. Tenemos entonces (xz −yw)(xw−yz) = x2 zw−xyz 2 −xyw2 +y 2 zw = (x2 +y 2 )zw−xy(z 2 +w2 ) = p(zw−xy). 60

√ Como p es primo, entonces p|xz − yw o p|xw − yz. Por otra parte, por (i), x, y, z, w < p, luego |xz − yw|, |xw − yz| < p, por lo que decir que xz − yw o xw − yz son divisibles por p es lo mismo que decir que xw − yz o xw − yz son cero y, en cualquiera de los caso, se tendr´a tambi´en zw − xy = 0. Supongamos por ejemplo xw − yz = 0. Entonces zw2 = w(zw) = w(xy) = y(xw) = y(yz) = y 2 z de donde se sigue y 2 = w2 y por tanto x2 = w2 , es decir, y = w y x = z. Cuando xw − yz = 0, el mismo argumento demuestro x = w, y = z. A la vista de la parte (i) del resultado anterior, empezamos viendo cu´ando una suma de dos cuadrados es divisible por un n´ umero primo. Lema 6.13. Sea p un n´ umero primo y sean x, y n´ umeros enteros no divisibles por p. Sea 2 a ∈ Z tal que ax ≡ y (mod p). Entonces p|x + y 2 si y s´olo si a2 ≡ −1 (mod p). En particular, un n´ umero primo p ≡ 3 (mod 4) no puede dividir a la suma de dos cuadrados no divisibles por p. Demostraci´ on: Para la primera parte, basta observar que x2 ≡ −y 2 ≡ −a2 x2 (mod p). Como mcd(x, p) = 1, por la Proposici´on 2.1(vi) la congruencia anterior es equivalente a a2 ≡ −1 (mod p). Para la segunda parte, supongamos por reducci´on al absurdo que p ≡ 3 (mod 4) y p|x + y 2 con x, y no divisibles por p. Como mcd(x, p) = 1, por la Proposici´on 2.9, existir´ a 2 a ∈ Z tal que ax ≡ y (mod p). El Lema 6.13 implica entonces a ≡ −1 (mod p), y por el Teorema 2.19 no puede ser p ≡ 3 (mod 4). 2

El resultado anterior y la Proposici´on 6.12(i) sugieren que necesitaremos el siguiente: Lema 6.14 (Thue). Sea p un n´ umero primo y a ∈ Z con mcd(a, p) = 1. Entonces existen √ x, y ∈ Z tales que ax ≡ y (mod p) y 0 < |x|, |y| < p. Demostraci´ on: Consideramos el conjunto de expresiones de la forma ax − y con 0 ≤ x, y ≤ √ √ p. Como tenemos ([ p] + 1)2 > p posibilidades para los pares x, y, necesariamente tendremos dos pares distintos x1 , y1 y x2 , y2 en esas condiciones tales que ax1 − y1 ≡ ax2 − y2 (mod p). Por tanto, x = x1 − x2 , y = y1 − y2 es la soluci´on buscada. Teorema 6.15. Un n´ umero primo p se puede expresar como suma de dos cuadrados si y s´olo si p = 2 o p ≡ 1 (mod 4). Demostraci´ on: Suponemos en primer lugar que podemos escribir p = x2 + y 2 . Por la Proposici´on 6.12(i), 0 < |x|, |y| < p, luego x, y no son divisibles por p. El Lema 6.13 implica entonces que p no es congruente con 3 m´odulo 4, luego p = 2 o p ≡ 1 (mod 4). 61

Rec´ıprocamente, si p = 2 o p ≡ 1 (mod 4), el Teorema 2.19 implica que existe a ∈ Z tal que p|a2 + 1. Evidentemente, mcd(a, p) = 1, por lo que por el Lema de Thue existen √ x, y ∈ Z tales que ax ≡ y (mod p) y 0 < |x|, |y| < p. La condici´on ax ≡ y (mod p) √ implica, por el Lema 6.13, que p|x2 + y 2 , y entonces la condici´on 0 < |x|, |y| < p implica, por el Lema 6.13, p = x2 + y 2 , como quer´ıamos. Podemos caracterizar finalmente los enteros que son sumas de dos cuadrados: Teorema 6.16. Un n´ umero positivo es suma de dos cuadrados si y s´olo si, en su descomposici´on en factores primos, todos los factores primos de la forma 4k + 3 aparecen con exponente par. Demostraci´ on: Sea n = x2 + y 2 y llamemos d = mcd(x, y). Podemos entonces escribir x = dx0 y y = dy 0 con mcd(x0 , y 0 ) = 1. Entonces n = d2 (x02 + y 02 ). Cada factor primo p de x02 + y 02 , no puede dividir ni a x0 ni a y 0 (si dividiera a uno, dividir´ıa a ambos), luego por la Lema 6.13, no es p ≡ 3 (mod 4). Por tanto, los factores primos de n de la forma 4k + 3 est´an todos en d2 , luego aparecen con exponente par. Rec´ıprocamente, si n tiene todos sus factores primos de la forma 4k + 3 con exponente par, se podr´ a escribir n = d2 p1 . . . pr , donde p1 , . . . , pr son primos distintos que no son de la forma 4k +3. Por el Teorema 6.15, cada pi es suma de dos cuadrados, y por el Lema 6.11 su producto tambi´en lo es, es decir, p1 . . . pr = x2 + y 2 . De aqu´ı se sigue n = (dx)2 + (dy)2 .

Observaci´ on 6.17. N´ otese que, si n no es primo, no se tiene la unicidad de la descomposici´on en suma de dos cuadrados que obtuvimos en la Proposici´on 6.12(ii). En efecto, consideremos n = 65 = 5 · 13. Tenemos entonces 5 = 22 + 12 y 13 = 32 + 22 . Aplicando el Lema 6.11 con x = 2, y = 1, z = 3, w = 2, obtenemos 65 = 82 + 12 , mientras que, si aplicamos el Lema 6.11 con x = 2, y = 1, z = 2, w = 3, obtendremos 65 = 72 + 42 . Ya que hay muchos n´ umeros positivos que no se pueden escribir como suma de dos cuadrados, cabe plantearse ahora al pregunta: ¿podr´a escribirse todo n´ umero positivo como suma de tres cuadrados? Inmediatamente encontramos una respuesta negativa: Teorema 6.18. Ning´ un n´ umero de la forma 4n (8k + 7) se puede escribir como suma de tres cuadrados. Demostraci´ on: Lo demostramos por inducci´on sobre n. Si n = 0, tenemos que ver que no se puede escribir 8k +7 = x2 +y 2 +z 2 . Esto es evidente, ya que x2 , y 2 , z 2 ≡ 0, 1, 4 (mod 8). 62

Supongamos ahora demostrado el resultado para n y veamos que es cierto para n + 1. Supongamos, por reducci´ on al absurdo, que podemos escribir 4n+1 (8k + 7) = x2 + y 2 + z 2 . Como 4n+1 ≡ 0 (mod 4) y x2 , y 2 , z 2 ≡ 0, 1 (mod 4), se sigue que necesariamente a2 , b2 , c2 ≡ 0 (mod 4), es decir, x, y, z son pares. Escribimos entonces x = 2x0 , y = 2y 0 y z = 2z 0 , por lo que se tendr´ a 4n (8k + 7) = x02 + y 02 + z 02 , lo que contradice la hip´otesis de inducci´on. Observaci´ on 6.19. En realidad, se puede demostrar que el teorema anterior caracteriza a los n´ umeros que no son sumas de tres cuadrados. Sin embargo, la demostraci´on no es f´ acil (por lo que no la haremos). Esto se debe a que, contrariamente a lo que ocurre para la suma de dos cuadrados (ver el Lema 6.11) el hecho de ser suma de tres cuadrados no se respeta por productos. Por ejemplo, 14 = 32 + 22 + 12 y 18 = 42 + 12 + 12 , mientras que 14 · 18 = 4(8 · 7 + 7) no es suma de tres cuadrados por el Teorema 6.18. Ya que con tres cuadrados no se puede, podemos seguir pregunt´andonos ahora: ¿Y ser´ a cierto que todo n´ umero entero positivo se puede escribir como suma de cuatro cuadrados? La respuesta ahora ser´ a positiva, y se basa en que, en este caso ya es cierto el an´alogo del Lema 6.11, que es una simple cuenta que dejamos como ejercicio: Ejercicio 6.20. Comprobar que se verifica la igualdad (a2 + b2 + c2 + d2 )(x2 + y 2 + z 2 + w2 ) = r2 + s2 + t2 + u2 donde    r = ax + by + cz + dw    s = bx − ay + dz − cw  t = cx − dy − az + bw     u = dx + cy − bz − aw A la vista de este resultado, bastar´a ver que todo n´ umero primo es suma de cuatro cuadrados. Como en el caso de dos cuadrados, empezamos de momento con propiedades sobre los divisores de distintas sumas de cuadrados: Lema 6.21. Si p ≡ 3 (mod 4) es un n´ umero primo, entonces existen x, y ∈ Z tales que 2 2 p|x + y + 1. Demostraci´ on: Por (iii) y (iv) de la Proposici´on 5.3, tenemos respectivamente    las  partes  p−1 1 −1 =1y = = −1. Podremos encontrar entonces a ∈ {2, 3, . . . , p − 1} tal p p     p que a−1 = 1 y ap = −1 y, en particular, p      −a −1 a = = (−1)(−1) = 1. p p p 63

Por tanto, existen a − 1 y −a son restos cuadr´aticos m´odulo p, es decir, existen x, y ∈ Z tales que x2 ≡ a − 1 (mod p) y y 2 ≡ −a (mod p). Sumando, tendremos x2 + y 2 ≡ (a − 1) + (−a) = −1 (mod p), es decir, p|x2 + y 2 + 1. Lema 6.22. Sea m un divisor impar de una suma x2 + y 2 + z 2 + w2 . Entonces existen a, b, c, d ∈ Z tales que  x ≡ a (mod m)      y ≡ b (mod m)  z ≡ c (mod m)     w ≡ d (mod m) y a2 + b2 + c2 + d2 = mm0 , con m0 < m. Demostraci´ on: Sea r el resto de la divisi´on de x entre m. Si r < m 2 tomamos a = r, y si m r > 2 tomamos a = r − m (hace falta que m sea impar para que no se pueda dar el caso m r= m 2 . Entonces x ≡ a (mod m) y |a| < 2 . Construimos de la misma forma b, c, d, es decir, congruentes respectivamente con y, z, w m´odulo m y de valor absoluto menor que m 2 . Por tanto, tendremos a2 + b2 + c2 + d2 ≡ x2 + y 2 + z 2 + w2 ≡ 0 (mod m) luego existir´ a m0 ∈ Z tal que a2 + b2 + c2 + d2 = mm0 . Adem´as, 0

2

2

2

2

mm = a + b + c + d <

 m 2 2

+

 m 2 2

+

 m 2 2

+

 m 2 2

= m2

por lo que m0 < m. Finalmente, podemos demostrar ya el resultado que busc´abamos: Teorema 6.23. Todo n´ umero natural se puede escribir como suma de cuatro cuadrados (no necesariamente distintos de cero). Demostraci´ on: Por el Ejercicio 6.20, basta demostrar que cada n´ umero primo p es suma de cuatro cuadrados. Si p = 2 o p ≡ 1 (mod 4) ya sabemos que p se escribe como suma de incluso s´olo dos cuadrados, as´ı que basta demostrar el caso p ≡ 3 (mod 4). Por el Lema 6.21, sabemos que p divide a la suma de tres cuadrados de enteros no todos nulos, luego en particular un m´ ultiplo de p se puede poner como la suma de cuatro cuadrados de enteros no todos nulos. Sea kp el m´ ultiplo m´as peque˜ no de p que se puede escribir como suma de 64

cuatro cuadrados de enteros positivos no todos nulos. Por el Lema 6.22 tomando m = p, se tiene k < p, y lo que queremos ver es k = 1. Veamos primero que k es impar. Escribimos kp = x2 + y 2 + z 2 + w2 . Si k fuera par, entonces entre x, y, z, w hay una cantidad par de n´ umeros pares y una cantidad par de impares. En cualquier caso, reorden´andolos, podemos suponer que x e y tienen la misma paridad y que z y w tienen la misma paridad. Entonces tendremos k p= 2



x+y 2

2

 +

x−y 2

2

 +

z+w 2

2

 +

z−w 2

2 ,

lo que contradice la minimalidad de k, lo que implica que k es impar. Supongamos, por reducci´ on al absurdo, que k > 1. Aplicando ahora el Lema 6.22 tomando m = k, existir´ an a, b, c, d ∈ Z tales que  x ≡ a (mod k)      y ≡ b (mod k)  z ≡ c (mod k)     w ≡ d (mod k) y a2 + b2 + c2 + d2 = nk con n < k. Si fuera n = 0, entonces a = b = c = d = 0, luego k dividir´ıa a x, y, z, w, as´ı que k 2 dividir´ıa a x2 + y 2 + z 2 + w2 = kp, es decir, k dividir´ıa a p, lo que contradice el hecho de que 1 < k < p. Por tanto, 0 < n < k. Usando la igualdad del Ejercicio 6.20 tendremos k 2 np = (a2 + b2 + c2 + d2 )(x2 + y 2 + z 2 + w2 ) = r2 + s2 + t2 + u2 donde  r = ax + by + cz + dw      s = bx − ay + dz − cw  t = cx − dy − az + bw     u = dx + cy − bz − aw

≡ x2 + y 2 + z 2 + w2 ≡ 0 (mod k) ≡ ba − ab + dc − cd = 0 (mod k) ≡ ca − db − ac + bd = 0 (mod k) ≡ da + cb − bc − ad = 0 (mod k)

es decir, r, s, t, u son divisibles por k. Se tiene entonces np =

 r 2 k

+

 s 2 k

 2   t u 2 + + , k k

lo que de nuevo contradice la minimalidad de k, ya que n < k. Por tanto, k = 1, como quer´ıamos. 65

Observaci´ on 6.24. Para sumas de potencias superiores, se tiene el llamado problema de Waring. En concreto, fijado n ∈ Z≥1 , ¿cu´al es el m´ınimo n´ umero g(n) tal que cada n´ umero entero positivo es suma de g(n) potencias n-´esimas de n´ umeros enteros? En este contexto, el Teorema 6.18 indica que g(2) ≥ 4 y el Teorema 6.23 indica que g(2) ≤ 4, por lo que g(2) = 4. Para n ≥ 3 el problema es mucho m´as complicado, ya que no ocurre que el producto sumas de g potencias n-´esimas sea una suma de g potencias n-´esimas (como ocurre para n = 2 con g = 2 y g = 4). Hay adem´as una situaci´on nueva. Por ejemplo, para n = 3 no es dif´ıcil ver que 23 = 23 + 23 + 13 + 13 + 13 + 13 + 13 + 13 + 13 es la menor descomposici´ on en suma de cubos de 23, por lo que g(3) ≥ 9. Por otra parte, ha logrado demostrarse (por L. Dickson) que cualquier entero positivo distinto de 23 y 239 puede escribirse como suma de como mucho ocho cubos, y de la misma forma se sabe (por un resultado debido a Linnik) que salvo una cantidad finita de casos, todo entero positivo es suma de a lo sumo de siete cubos. Esto da lugar a otro problema, llamado el gran problema de Waring (al anterior se le suele llamar por contraposici´on peque˜ no problema de Waring): ¿cu´al es el m´ınimo n´ umero G(n) tal que cada n´ umero entero positivo, salvo una cantidad finita de casos, es suma de G(n) potencias n-´esimas de n´ umeros enteros? Mientras que para n = 2 sigue siendo g(2) = G(2) = 4, lo anterior para n = 3 se traduce en g(3) = 9, G(3) ≤ 7. Es un problema abierto cu´al es el valor de G(3) (se sospecha G(3) = 4, pero s´olo se sabe 4 ≤ G(3) ≤ 7) y en general de G(n) para n ≥ 3. Respecto a g(n), se ha demostrado recientemente que g(4) = 19, g(5) = 37 y g(n) = [( 23 )n ] + 2n − 2 si n ≥ 6 salvo quiz´a una cantidad finita de valores de n. Los Teoremas 6.10 y 6.16 pueden interpretarse como haber estudiado cu´ando las ecuaciones de la forma x2 − y 2 = n o x2 + y 2 = n tienen soluci´on entera. El caso m´as sencillo ha sido el de diferencia de dos cuadrados. Podemos complicar la ecuaci´on un poco m´ as y poner coeficientes a la x y a la y, en cuyo caso las cosas se complican much´ısimo. El primer caso, aparentemente sencillo, ser´ıa la ecuaci´ on x2 − dy 2 = 1 con d entero positivo que no sea un cuadrado perfecto. Esta ecuaci´on (llamada ecuaci´ on de Pell, aunque en realidad se deber´ıa atribuir a Fermat), no es f´acil en absoluto, aunque al menos se puede decir que sus soluciones (caso de existir) tienen una buena estructura: Teorema 6.25. Sea d un entero positivo que no sea un cuadrado perfecto y consideremos la ecuaci´on x2 − dy 2 = 1. (i) Si x0 , y 0 y x00 , y 00 son soluciones enteras de la ecuaci´on, entonces x0 x00 +dy 0 y 00 , x0 y 00 +x00 y 0 es tambi´en soluci´ on de la ecuaci´on. √ (ii) Si existe una soluci´ on entera positiva x1 , y1 tal que x1 + y1 d es m´ınimo entre todas las soluciones enteras positivas de la ecuaci´on, entonces las soluciones enteras positivas √ √ de la ecuaci´ on son los pares de la forma xn , yn donde xn + yn d = (x1 + y1 d)n . 66

Demostraci´ on: Obs´ervese que √ √ √ (x0 + y 0 d)(x00 + y 00 d) = (x0 x00 + dy 0 y 00 ) + (x0 y 00 + x00 y 0 ) d √ √ √ (x0 − y 0 d)(x00 − y 00 d) = (x0 x00 + dy 0 y 00 ) − (x0 y 00 + x00 y 0 ) d √ √ Como las soluciones de la ecuaci´ on son los enteros que verifican (x + y d)(x − y d) = 1, multiplicando las dos igualdades anteriores se obtiene (i). De la misma forma se obtiene que los pares xn , yn de (ii) son soluci´on. Lo que falta ver es que son las u ´nicas soluciones. Supongamos, por reducci´ on al absurdo, que tenemos otra soluci´on entera positiva √ x , y que no es ninguna de las xn , yn . Por la minimalidad de x1 , y1 , se tendr´a x1 + y1 d < √ √ x0 +y 0 d. Adem´ as como la sucesi´ on (x1 +y1 d)n es mon´otona creciente con l´ımite infinito se tendr´a que existe n ∈ Z≥1 tal que 0

0

√ √ √ (x1 + y1 d)n < x0 + y 0 d < (x1 + y1 d)n+1 . √ √ Multiplicando por (x1 − y1 d)n (que es positivo porque su producto con (x1 + y1 d)n es 1) se tiene √ √ √ 1 < (x0 + y 0 d)(x1 − y1 d)n < x1 + y1 d. Ahora bien, por (i), si escribimos √ √ √ √ √ (x0 + y 0 d)(x1 − y1 d)n = (x0 + y 0 d)(xn − yn d) = x00 + y 00 d se tiene que x00 , y 00 es una soluci´ on de la ecuaci´on. Para encontrar el absurdo que buscamos √ basta ver que esta u ´ltima soluci´ on es positiva. Ahora bien, como x00 + y 00 d > 1 y √ √ √ (x00 + y 00 d)(x00 − y 00 d) = 1, entonces 0 < x00 − y 00 d < 1. Por tanto, √ √ 2x00 = (x00 + y 00 d) + (x00 − y 00 d) > 1 + 0 > 0 √ √ √ 2y 00 d = (x00 + y 00 d) − (x00 − y 00 d) > 1 − 1 > 0 lo que termina la demostraci´ on. Esta demostraci´ on indica lo que es una de las t´ecnicas fundamentales de la Teor´ıa Algebraica de N´ umeros: ampliar el conjunto de los enteros a expresiones, por ejemplo, de √ la forma a + b d. El problema principal de esta t´ecnica es que, salvo para valores muy concretos de d, no se verifica la factorizaci´on u ´nica que tenemos para los enteros (y cuando se verifica no es de forma tan f´ acil). Para terminar de estudiar la ecuaci´on de Pell (y en particular la existencia de la soluci´ on x1 , y1 que necesita el teorema anterior) desarrollaremos en la siguiente secci´ on toda una t´ecnica nueva, la de las fracciones continuas. 67

7. Fracciones continuas Cuando pasamos de los n´ umeros enteros a los racionales, la escritura que solemos hacer de ellos, aparte de como fracciones, es a partir de su notaci´on decimal. Esta escritura decimal presenta dos inconvenientes. Uno es que depende de una elecci´on de escritura en base 10, que a priori es arbitraria. El segundo inconveniente es que las expresiones con decimales suelen ser infinitas, aunque al menos son peri´odicas y se pueden representar de forma finita. Ejemplo 7.1. Veamos una tercera representaci´on de un n´ umero racional a partir del a algoritmo de Euclides. Si tenemos el n´ umero b , escribimos a = ba0 + r1 b = r1 a1 + r2 r1 = r2 a2 + r3 .. .

0 < r1 < b 0 < r2 < r1 0 < r3 < r2

rn−2 = rn−1 an−1 + rn rn−1 = rn an

0 < rn < rn−1

donde rn = mcd(a, b). Adem´ as, a1 , . . . , ar > 0 porque son cocientes de n´ umeros positivos uno mayor que el otro. Podemos entonces escribir: r1 a = a0 + b b b r2 = a1 + r1 r1 r1 r3 = a2 + r2 r2 .. . rn rn−2 = an−1 + rn−1 rn−1 rn−1 = an rn (es decir, a0 , a1 , . . . , ar son las partes enteras de a r1 1 = a0 + = a0 + b b a1 +

r2 r1

= a0 +

rn−1 a b b , r1 , . . . , rn )

1 a1 +

1 r a2 + r3 2

68

y, poniendo todo junto,

= . . . = a0 +

1 a1 +

1 a2 +

1

..

.

1 an−1 + 1 an

Ejemplo 7.2. Podr´ıamos intentar ahora repetir un proceso parecido para cualquier n´ umero: dado un n´ umero real α 6= 0, vamos escribiendo α0 := α = [α0 ] + (α0 − [α0 ]) = a0 + (α0 − a0 ) α1 :=

1 = [α1 ] + (α1 − [α1 ]) = a1 + (α1 − a1 ) α0 − a0 .. .

es decir, definimos por recurrencia, a partir de α0 = α, los n´ umeros ak y αk mediante: ak = [αk ] αk+1 =

1 . αk − ak

Tendremos entonces α = a0 +

1 a1 +

.

1 a2 +

1

..

.

1 ak−1 +

ak +

1 1 αk+1

Obs´ervese que, como ak ≤ αk < ak + 1, necesariamente 0 ≤ αk − ak < 1, por lo que, si αk 6= ak (es decir, si αk no es entero), se tendr´a αk+1 > 1, por lo que ak ≥ 1 si k ≥ 1 (aunque a0 puede ser cero o incluso negativo). Si α es racional, el Ejemplo 7.1 indica que el proceso termina (lo que ocurre cuando llegamos a alg´ un αk entero y, por tanto ak = αk ), mientras que si α es irracional, entonces cada αk es tambi´en irracional, y por tanto el √ proceso no termina nunca. Por ejemplo, si tomamos α 2, tendremos entonces √ √ α0 = 2 = 10 414213562 . . . = 1 + 00 414213562 . . . = 1 + ( 2 − 1) √ √ 1 = 2 + 1 = 20 414213562 . . . = 2 + 00 414213562 . . . = 2 + ( 2 − 1) 2−1 √ y, a partir de aqu´ı, α1 = α2 = . . . = 2 + 1, y por tanto a1 = a2 = . . . = 1, mientras a0 = 1. Parecer´ıa entonces natural escribir α1 = √



2=1+

1 2+

1 2+

2+

1 1 2+...

Definici´ on. Se llama fracci´ on continua a una expresi´on del tipo [a0 ; a1 , a2 , . . .] := a0 +

69

1 a1 +

1 a2 +

a3 +

1 1 a4 +...

donde los n´ umeros a1 , a2 , . . . (que pueden ser una cantidad finita o infinita) son estrictamente positivos. Se llama fracci´ on continua asociada a un n´ umero real α a la fracci´ on continua [a0 ; a1 , a2 , . . .] definida como en el Ejemplo 7.2. Se dice que una fracci´on continua es simple si a0 , a1 , . . . son n´ umeros enteros (y por tanto a1 , a2 , . . . ≥ 1). Usaremos tambi´en la notaci´ on [a0 ; a1 , . . . , ar , ar+1 , . . . , ar+s ] para indicar la fracci´on continua infinita peri´odica [a0 ; a1 , . . . , ar , ar+1 , . . . , ar+s , ar+1 , . . . , ar+s , ar+1 , . . . , ar+s , . . .]. Por lo que hemos visto, un n´ umero racional se expresa siempre como una fracci´ on continua simple finita, y es obvio que las fracciones continuas simples finitas representan n´ umeros racionales. Por otra parte, el m´etodo anterior sugiere que los n´ umeros irracionales tienen asociada una fracci´ on continua infinita. Para dar sentido a dicha expresi´on infinita, habr´a que verla como l´ımite de sus subfracciones infinitas. M´as precisamente: Definici´ on. Se llama convergente k-´esimo de una fracci´on continua simple [a0 ; a1 , a2 , . . .] (finita o infinita) a Ck = [a0 ; a1 , a2 , . . . , ak ]. Teorema 7.3. Sea [a0 ; a1 , a2 , . . .] una fracci´on continua (no necesariamente simple o finita). Definimos recursivamente dos sucesiones de n´ umeros pk y qk con k ≥ −2 mediante   q =1 p−2 =0    −2  q−1 =0 p−1 =1     qk =ak qk−1 + qk−2 pk =ak pk−1 + pk−2 Entonces: (i) Para cada k ≥ 0 se tiene Ck =

pk qk .

(ii) Para cada k ≥ −2 se tiene pk+1 qk − pk qk+1 = (−1)k . Demostraci´ on: Demostraremos (i) por inducci´on sobre k. Para k = 0, se tiene p0 = a0 , q0 = 1, luego pq00 = a0 = [a0 ] = C0 . Supongamos ahora que sabemos que la f´ormula es cierta para convergentes k-´esimos y veamos que es cierta para los (k + 1)-´esimos. Para ello consideramos la fracci´on continua [a00 ; a01 , a02 , . . . , a0k ] := [a0 ; a1 , a2 , . . . , ak +

1 ak+1

],

que claramente coincide con Ck+1 . Por otra parte, es claro que si i ≤ k −1 entonces p0i = pi y qi0 = qi , mientras que para i = k se tiene p0k = a0k p0k−1 + p0k−2 = (ak + 0 0 qk0 = a0k qk−1 + qk−2 = (ak +

1 ak+1 1 ak+1

)pk−1 + pk−2 = ak pk−1 + pk−2 +

pk−1 pk−1 = pk + ak+1 ak+1

)qk−1 + qk−2 = ak qk−1 + qk−2 +

qk−1 qk−1 = qk + ak+1 ak+1

70

luego, por hip´ otesis de inducci´ on, Ck+1 =

Ck0

pk + p0 = k0 = qk qk +

pk−1 ak+1 qk−1 ak+1

=

ak+1 pk + pk−1 pk+1 = ak+1 qk + qk−1 qk+1

Demostramos (ii) tambi´en por inducci´ on sobre k, siendo trivial el caso k = −2. Supuesta cierta la f´ ormula para k, se tiene pk+2 qk+1 − pk+1 qk+2 = (ak+2 pk+1 + pk )qk+1 − pk+1 (ak+2 qk+1 + qk ) = pk qk+1 − pk+1 qk = = −(pk+1 qk − pk qk+1 ) = −(−1)k = (−1)(k+1) .

Ejemplo 7.4. La parte (ii) del teorema anterior implica, si la fracci´on continua es simple, que pk , qk son enteros primos entre s´ı (usando por ejemplo el Teorema 1.8). De hecho, puede usarse esta parte para obtener expl´ıcitamente una combinaci´on lineal de dos enteros que d´e su m´aximo com´ un divisor. Por ejemplo, consideremos los enteros a = 34, b = 14. Si aplicamos el algoritmo de Euclides podremos escribir 34 = 2 · 14 + 6 14 = 2 · 6 + 2 6=3·2+0 de donde sacamos

34 = [2; 2, 3]. 14

Las sucesiones pk , qk valen entonces p0 = 2, p1 = 5, p2 = 17 q0 = 1, q1 = 2, q2 = 7 34 lo que nos da 14 = C2 = parte nos da la relaci´ on

17 7 .

Esto indica, por una parte, que mcd(34, 14) = 2, y por otra 2 · 17 − 5 · 7 = −1,

que, cambiando de signo y multiplicada por el m´aximo com´ un divisor nos da 5 · 14 − 2 · 34 = 2 71

Lema 7.5. Dada una fracci´ on continua, se tiene, para todo k ≥ 0,: (i) Ck+1 − Ck = (ii) Ck+2 − Ck =

(−1)k qk+1 qk . (−1)k 1 qk+1 ( qk



1 qk+2 ).

(iii) Si la fracci´ on es simple, 0 < q1 < q2 < . . .. Demostraci´ on: Por el Teorema 7.3, tenemos Ck+1 − Ck =

pk+1 pk pk+1 qk − pk qk+1 (−1)k − = = , qk+1 qk qk+1 qk qk+1 qk

lo que demuestra (i). Utilizando dos veces (i) tendremos (−1)k+1 (−1)k (−1)k + = Ck+2 − Ck = (Ck+2 − Ck+1 ) + (Ck+1 − Ck ) = qk+2 qk+1 qk+1 qk qk+1



1 1 − qk qk+2

 ,

lo que demuestra (ii). Para la parte (iii), se tiene q1 = a1 > 0, luego basta demostrar qk−1 < qk para todo k ≥ 2. Lo hacemos por inducci´ on sobre k. Si k = 2, tenemos q2 = a2 q1 + q0 = a2 q1 + 1 > a2 q1 . Como a2 ≥ 1 (ya que a2 > 0 y a2 es entero por ser la fracci´on simple), se sigue q2 > q1 . El paso de inducci´ on sigue los mismos pasos. Si suponemos que se verifica q1 < q2 < . . . < qk−1 < qk , entonces en particular qk−1 , qk > 0. Como de nuevo se tiene ak+1 ≥ 1 por ser la fracci´ on simple, se sigue: qk+1 = ak+1 qk + qk−1 ≥ qk + qk−1 > qk . Teorema 7.6. Dada una fracci´ on continua simple, se tiene, para todo k ≥ 0, C0 < C2 < C4 < . . . < C2k < C2k+1 < . . . < C3 < C1 . Adem´as, si la fracci´ on continua es infinita, existe el l´ımite de la sucesi´on {Ck }, que es un n´ umero irracional α que verifica C2k < α < C2k+1 para todo k. k

1 1 as qk+1 > 0 Demostraci´ on: Por el Lema 7.5, se tiene Ck+2 − Ck = (−1) qk+1 ( qk − qk+2 ) y adem´ y qk < qk+2 , luego Ck+2 − Ck es positivo si k es par y negativo si k es impar, por lo que los convergentes pares son crecientes y los impares son decrecientes. Adem´as Ck+1 − Ck = (−1)k qk+1 qk , que es positivo si k es par y negativo si k es impar. Por tanto, un convergente impar es siempre mayor que su convergente anterior par.

Tenemos entonces la sucesi´ on creciente C0 < C2 < C4 < . . ., que est´a acotada, luego tiene un l´ımite α, y la sucesi´ on decreciente C1 > C3 > C5 > . . ., que tambi´en est´a acotada, 0 luego tiene l´ımite α ≥ α. Adem´ as, para todo k se tiene α0 − α < C2k+1 − C2k = 72

(−1)2k , q2k+1 q2k

que es un valor que tiende a cero (por ser qk una sucesi´on creciente de n´ umeros enteros), luego α = α0 . Veamos finalmente que α es irracional. Supongamos, por reducci´on al absurdo, que fuera α = ab , con a, b enteros (y b > 0). Entonces, como para cada k se tiene que α est´ a entre Ck y Ck+1 (qui´en sea el mayor de los dos depende de la paridad de k), se tiene a 1 0 < | − Ck | < |Ck+1 − Ck | = . b qk+1 qk Multiplicando por bqk , se obtiene b 0 < |aqk − bpk | < , qk+1 lo que es absurdo, porque aqk − bpk es un entero y si k es suficientemente grande se tendr´ıa b qk+1 < 1 (ya que qk+1 tiende a infinito). Definici´ on. Se llama valos de una fracci´ on continua simple infinita al l´ımite α dado por el Teorema 7.6. Ejemplo 7.7. Veamos ahora un ejemplo de c´alculo del valor de una fracci´on continua infinita. Tomemos, por ejemplo, [1; ¯1]. Las sucesiones pk , qk verifican p−1 = 1, p0 = 1, pk = pk−1 + pk−2 q0 = 1, q1 = 1, qk = qk−1 + qk−2 , por lo que pk = uk+2 y qk = uk+1 (donde un es el n-´esimo n´ umero de Fibonacci; ver uk+2 Ejercicio 0.3). Se tiene entonces Ck = uk+1 . Usando la f´ormula del Ejercicio 0.3 para el n-´esimo n´ umero de √ Fibonacci, es un simple ejercicio de An´alisis Matem´atico demostrar uk+2 1 que lim uk+1 = 2 + 25 . Sin embargo, para fracciones continuas peri´odicas, hay un truco m´as f´acil para calcular su valor. Como el Teorema 7.6 nos asegura que [1; ¯1] tiene un valor α, se tendr´ a la relaci´ on: 1 1 α=1+ =1+ , 1 α 1 + 1+ 1 1+

1 1+...

por lo que debiera ser α2 − α − 1 = 0, es decir, α = la soluci´on positiva).

1 2



+

5 2

(obviamente, debemos tomar

Tenemos entonces un modo de, a partir de un n´ umero irracional, conseguir una fracci´ on continua simple infinita; y tenemos tambi´en un modo de, a partir de una fracci´on continua simple infinita, darle un valor irracional. Una pregunta natural es si ambos procesos son inversos el uno del otro. Por ejemplo, se deja como ejercicio al lector comprobar que √ 1 5 la fracci´on continua asociada a 2 + 2 (ver el Ejemplo 7.7) es precisamente [1; ¯1]; de la √ misma forma, el valor de la fracci´ on continua [1; ¯2] (ver el Ejemplo 7.2) es 2. El resultado (afirmativo) general es el siguiente: 73

Teorema 7.8. Cada n´ umero irracional α es el valor de una y s´olo una fracci´on continua infinita simple, que es precisamente la fracci´on continua asociada a α. Demostraci´ on: Supongamos primero que un n´ umero irracional α es el valor de las fracciones 0 0 0 continuas simples [a0 ; a1 , a2 , . . .] y [a0 ; a1 , a2 , . . .]. Entonces, por el Teorema 7.6, tendremos C0 < α < C1 , es decir a0 < α < a0 + a11 . Como a1 ≥ 1, se tiene entonces a0 = [α], y de la misma forma a00 = [α], por lo que a0 = a00 . Como se tiene a0 +

1 1 = [a0 ; a1 , a2 , . . .] = [a00 ; a01 , a02 , . . .] = a00 + 0 0 0 [a1 ; a2 , a3 , . . .] [a1 ; a2 , a3 , . . .]

se deduce [a1 ; a2 , a3 , . . .] = [a01 ; a02 , a03 , . . .], y por un argumento de recurrencia se sigue a0k = ak para todo k. Rec´ıprocamente, dado un n´ umero irracional α, sea [a0 ; a1 , a2 , . . .] su fracci´on continua asociada (tomaremos la notaci´ on del Ejemplo 7.2). Es claro que, para todo k, se tiene α = [a0 ; a1 , a2 , . . . , ak , αk+1 ]. Entonces el convergente (k + 1)-´esimo de esta fracci´on continua finita es α, mientras que el convergente k-´esimo es claramente el convergente k-´esimo Ck de [a0 ; a1 , a2 , . . .]. Por el Lema 7.5 aplicado a [a0 ; a1 , a2 , . . . , ak , αk+1 ], se tendr´a entonces α − Ck =

(−1)k , (αk+1 qk + qk−1 )qk

donde hemos usado el Teorema 7.3 para calcular el denominador del convergente (k + 1)´esimo. Usando ahora las desigualdades αk+1 > ak+1 y qk+1 > qk se tiene |α − Ck | =

1 1 1 1 < = < 2. (αk+1 qk + qk−1 )qk (ak+1 qk + qk−1 )qk qk+1 qk qk

Como qk tiende a infinito, se tiene que α = lim Ck .

Ejercicio 7.9. Con la ayuda de una calculadora, comprobar la validez de la f´ormula (al menos en sus primeros t´erminos): e = [2; 1, 2, 1, 1, 4, 1, 1, 6, 1, 1, 8, . . .] y utilizar la misma para calcular las diez primeras cifras decimales de e.

Observaci´ on 7.10. La demostraci´on tanto del Lema 7.5 como del Teorema 7.8 nos da el grado de aproximaci´ on de un convergente a un n´ umero irracional α, precisamente pk 1 1 |α − qk | < qk+1 qk < q2 . Por otra parte, si una considera una soluci´on positiva x = p, y = q k

74

de la ecuaci´ on de Pell x2 − dy 2 = 1, se tiene p2 = 1 + q 2 d > q 2 d, de donde se deduce √ p > q d. Por tanto, √ √ √ p √ 1 1 (p − q d)(p + q d) 1 1 1 √ √ < √ < 2. | − d| = (p − q d) = = q q q 2q (p + q d) q(p + q d) 2q 2 d √ Cabe entonces preguntarse, ya que pq aproxima d como un convergente (incluso en principio a´ un m´ as), si eso no implicar´a que pq es realmente un convergente. Si miramos por √ ejemplo el caso d = 2 del Ejemplo 7.2, en que demostramos 2 = [1; ¯2] tenemos los siguientes valores: p0 = 1, p1 = 3, p2 = 7, p3 = 17, . . . q0 = 1, q1 = 2, q2 = 5, q3 = 12, . . . Por tanto (p0 , q0 ) y (p2 , q2 ) son soluciones de la ecuaci´on x2 −2y 2 = 1, mientras que (p1 , q1 ) y (p3 , q3 ) no son soluciones. De hecho, veremos que las soluciones son convergentes, pero no todos los convergentes son soluciones. Para ver que un n´ umero racional que aproxima mucho a un irracional es un convergente necesitaremos el siguiente lema previo. Lema 7.11. Sea α un n´ umero irracional, sea pqkk el convergente k-´esimo de su fracci´ on continua y sea q un entero positivo tal que q < qk+1 . Entonces, para todo entero p se tiene |pk − qk α| ≤ |p − qα|. Demostraci´ on: Observamos en primer lugar que existen x, y ∈ Z tales que (

pk x + pk+1 y = p qk x + qk+1 y = q.

En efecto, el determinante de la matriz de coeficientes es, por el Teorema 7.3(ii), pk qk

pk+1 = pk qk+1 − pk+1 qk = −(−1)k = ±1 qk+1

luego existe una u ´nica soluci´ on del sistema anterior, que es adem´as entera. Observamos tambi´en que x 6= 0, ya que si fuera x = 0, entonces 0 = pq − qp = p(yqk+1 ) − q(ypk+1 ) = y(pqk+1 − qpk+1 ) y como y 6= 0 (ya que q 6= 0), se sigue pqk+1 = qpk+1 . Ahora bien, mcd(pk+1 , qk+1 ) = 1, luego qk+1 |q, lo que es absurdo porque q < qk+1 . 75

Comparemos ahora los signos de x e y. Si y < 0, entonces qk x = q − qk+1 y > 0 de donde se sigue x > 0. Si en cambio y > 0, entonces y ≥ 1 y se tiene qk x = q − qk+1 y < qk+1 − qk+1 y = qk+1 (1 − y) ≤ 0 y por tanto x < 0. Usando lo anterior, tenemos |p − qα| = |(pk x + pk+1 y) − (qk x + qk+1 y)α| = |x(pk − qk α) + y(pk+1 + qk+1 α)|. Si supi´eramos que |x(pk − qk α) + y(pk+1 + qk+1 α)| ≥ |x(pk − qk α)|, se concluir´ıa inmediatamente, ya que |x(pk − qk α)| = |x||pk − qk α| ≥ |pk − qk α| (puesto que x 6= 0 y por tanto |x| ≥ 1). La desigualdad |x(pk − qk α) + y(pk+1 + qk+1 α)| ≥ |x(pk −qk α)| es inmediata si y = 0, mientras que si y 6= 0 hay que demostrar que x(pk −qk α) e y(pk+1 − qk+1 α) tienen el mismo signo. Y esto es as´ı ya que, por una parte, hemos visto que x e y tienen signos opuestos cuando y 6= 0; y, por otra parte, como α est´a siempre k+1 en medio de dos convergentes consecutivos, los signos de pqkk − α y pqk+1 − α tambi´en son opuestos, es decir, pk − qk α y pk+1 − qk+1 α tienen signos opuestos. Por tanto, x(pk − qk α) e y(pk+1 − qk+1 α) tienen el mismo signo, como quer´ıamos. Como primer corolario, podemos ver que el convergente k-´esimo es la mejor aproximaci´on de un n´ umero irracional mediante un n´ umero racional de denominador como mucho qk : Corolario 7.12. Sea α un n´ umero irracional y sea pqkk el convergente k-´esimo de su fracci´ on pk p continua. Entonces, si q ≤ qk , se tiene | qk − α| ≤ | q − α| para cualquier entero p. Demostraci´ on: Como qk < qk+1 , se tiene, por el Lema 7.11, |pk − qk α| ≤ |p − qα| y por tanto pk |pk − qk α| |p − qα| |p − qα| p | − α| = ≤ ≤ = | − α|. qk qk qk q q

Teorema 7.13. Sea α un n´ umero irracional y sea α − pq < 2q12 , entonces pq es un convergente de α. 76

p q

un n´ umero racional tal que q > 0. Si

Demostraci´ on: Como qk es una sucesi´on creciente de n´ umeros enteros, existir´a k tal que p pk qk ≤ q < qk+1 . Veamos entonces que q = qk . Si no fuera as´ı, entonces pqk − qpk 6= 0, luego |pqk − qpk | ≥ 1. Tendremos entonces, por una parte: p pk |pqk − qpk | 1 − = ≥ q qk qqk qqk y por otra parte, usando el Lema 7.11: p pk p |pk − qk α| p p p k − ≤ − α + α − = − α + − α + |p − qα| = ≤ q q qk q qk q qk qk p p q − α < 1 + q 1 = 1 + 1 . = − α + 2q 2 q qk q qk 2q 2 2q 2 2qqk Poniendo juntas las dos desigualdades se obtiene 1 1 1 < 2+ , qqk 2q 2qqk lo que implica elecci´on de k.

1 2qqk

<

1 2q 2 ,

que a su vez implica q < qk , lo que es una contradicci´on con la

Corolario 7.14. Si p, q es una soluci´on positiva de x2 − dy 2 = 1, entonces √ convergente de d. Demostraci´ on: Por la Observaci´ on 7.10, | pq − √ 7.13, que pq es un convergente de d.



d| <

1 2q 2 ,

p q

es un

lo que implica, por el Teorema

√ Necesitamos ver ahora qu´e convergentes de α = d son soluciones de la ecuaci´ on de Pell. Por la construcci´ on por recurrencia del Ejemplo 7.2, est´a claro que cada αk se puede √ escribir en funci´ on de d. El siguiente resultado nos dar´a una f´ormula precisa, en que lo √ fundamental es que el coeficiente de d en el numerador es 1: √ Lema 7.15. Sea α = d y, para cada k ≥ 0, sea αk definido como en el Ejemplo 7.2. Entonces, definiendo por recurrencia s0 = 0, t0 = 1, sk+1 = ak tk − sk tk+1 =

d − s2k+1 d − (ak tk − sk )2 = tk tk 77

se puede escribir αk =

√ sk + d , tk

y adem´as sk y tk son enteros y tk 6= 0.

Demostraci´ on: Lo demostramos por inducci´on sobre k, siendo claro el caso k = 0. Si √ sk + d ahora suponemos αk = tk con sk y tk enteros y tk 6= 0, entonces tenemos αk+1 =

1 = αk − ak

1

√ sk + d tk

= − ak

√ tk tk ( d − sk + ak tk ) √ √ = = √ = sk + d − ak tk ( d + sk − ak tk )( d − sk + ak tk ) √ √ √ tk (ak tk − sk + d) ak tk − sk + d sk+1 + d = = d−(a t −s )2 = . k k k d − (ak tk − sk )2 tk+1 tk

Es claro que sk+1 es entero por serlo ak , sk , tk . Por otra parte, si escribimos tk+1 =

d − s2k+1 d − (ak tk − sk )2 d − s2k = = − a2k tk + 2ak sk = tk−1 − a2k tk + 2ak sk tk tk tk

se obtiene que tk+1 es entero (en realidad, como estamos usando que tk−1 es entero, habr´ıa que demostrar tambi´en el caso k = 1 de la inducci´on, pero al ser t0 = 1 es evidente que t1 es entero). Proposici´ on 7.16. Con las notaciones del Lema 7.15, se tiene: √ pk (i) Si qk es el convergente k-´esimo de d, entonces p2k − dqk2 = (−1)k+1 tk+1 . (ii) tk > 0 para todo k ≥ 0. (iii) Existen enteros t, s tales que tk = t y sk = s para infinitos valores de k. √ Demostraci´ on: Escribiendo d = [a0 ; a1 , . . . , ak , αk+1 ], sabemos que coincide con su convergente (k + 1)-´esimo, y por el Teorema 7.3 tendremos √

αk+1 pk + pk−1 d= = αk+1 qk + qk−1

√ sk+1 + d pk tk+1 √ sk+1 + d qk tk+1

+ pk−1 + qk−1

√ sk+1 pk + tk+1 pk−1 + pk d √ = sk+1 qk + tk+1 qk−1 + qk d

Por tanto √ √ √ √ (sk+1 pk +tk+1 pk−1 )+pk d = (sk+1 qk +tk+1 qk−1 +qk d) d = dqk +(sk+1 qk +tk+1 qk−1 ) d lo que implica (

pk =sk+1 qk + tk+1 qk−1 dqk =sk+1 pk + tk+1 pk−1 78

Usando estas igualdades junto con el Teorema 7.3(ii) tendremos p2k − dqk2 = pk (sk+1 qk + tk+1 qk−1 ) − qk (sk+1 pk + tk+1 pk−1 ) = = tk+1 (pk qk−1 − pk−1 qk ) = (−1)k−1 tk+1 lo que prueba (i). Demostremos (ii) por inducci´ on sobre k. Es evidente para k = 0, ya que, por definici´ on, t0 = 1. Supongamos entonces que sabemos tk > 0 y demostremos tk+1 > 0. Por (i), tendremos 2 p2k+1 − dqk+1 tk+1 =− . tk p2k − dqk2 El signo del numerador es igual al signo de 2 p2k+1 − dqk+1 2 = Ck+1 −d 2 qk+1

√ √ que es el signo de Ck+1 − d. An´ alogamente, el signo del denominador es el signo Ck − d. √ √ Como por el Teorema 7.6 los signos de Ck+1 − d y Ck − d son distintos, se sigue que tk+1 en tk+1 es positivo. Esto tk tiene signo positivo, y como tk es positivo entonces tambi´ demuestra (ii). Para demostrar (iii), basta recordar la igualdad tk tk+1 + s2k = d y, al ser tk , tk+1 > 0 √ por (ii), se tendr´ a 1 ≤ tk ≤ d y |sk | < d. Por tanto, hay una cantidad finita de posibles valores para los pares (tk , sk ), por lo que necesariamente alguno se debe repetir infinitas veces.

Observaci´ on 7.17. N´ otese que la parte (i) de la Proposici´on 7.16 indica que la ecuaci´ on 2 2 de Pell x − dy = 1 tiene soluci´ on (ya sabemos por el Teorema 7.13 que necesariamente pk olo si existe alg´ un k ∈ Z para el que tk+1 = 1 (recu´erdese debe ser un convergente qk ) si y s´ √ que tk+1 > 0 por la parte (ii)). Esto es equivalente a decir αk+1 = sk+1 + d, es decir, √ 1 1√ ak+1 = [αk+1 ] = sk+1 + [ d] y αk+2 = αk+1 −a = √d−[ = α1 . Por tanto, los k+1 d] datos de la fracci´ on continua se deben repetir despu´es de k + 1 pasos, por lo que debe ser √ d = [a0 ; a1 , a2 , . . . , ak+1 ]. De momento, la parte (iii) de la Proposici´on 7.16 garantiza s´olo que existen k, l tales que tk+l = tk y sk+l = sk , y por tanto αk+l = αk . De aqu´ı se deduce que ak0 +l = ak0 para todo k 0 ≥ k, y lo que faltar´ıa ver es que se puede tomar k = 1. √ Lema 7.18. Para cada k ≥ 1, definimos βk mediante la recurrencia β1 = a0 + d y βk+1 = ak + β1k . Entonces: 79

(i) βk > 1 (y por tanto [βk+1 ] = ak ). (ii) βk =

√ tk d−sk

=

√ sk + d tk−1 .

Demostraci´ on: Para demostrar (i), usamos inducci´on sobre k, siendo evidente el caso k = 1. Si suponemos ahora βk > 1, entonces β1k > 0, y como ak ≥ 1, de la f´ormula de recurrencia βk+1 = ak + β1k se sigue βk+1 > 1. Para demostrar (ii), observamos primero que, por el Lema 7.15, tenemos: √ √ √ √ tk (sk + d) tk tk ( d + sk ) tk (sk + d) sk + d √ √ = = √ = = d − s2k tk tk−1 tk−1 d − sk ( d − sk )( d + sk ) por lo que basta ver que βk coincide con uno cualquiera de los dos valores. Lo demostramos por inducci´ on sobre. Si k = 1, recordando del Lema 7.15 las igualdades s0 = 0, t0 = 1 y √ √ s1 + d s1 = a0 t0 − s0 = a0 , tenemos inmediatamente β1 = a0 + d = t0 . Supongamos ahora tk demostrado βk = √d−s y demostremos la f´ormula para k + 1. Para ello, aplicamos de k nuevo el Lema 7.15 y tendremos: √ √ √ 1 d − sk (ak tk − sk ) + d sk+1 + d βk+1 = ak + = ak + = = . βk tk tk tk

Teorema 7.19. Si un entero positivo d no es un cuadrado perfecto, entonces su fracci´ on √ continua tiene la forma d = [a0 ; a1 , a2 , a3 . . . a3 , a2 , a1 , 2a0 ]. Demostraci´ on: Como ya vimos en la Observaci´on 7.17, existir´an k, l tales que tk+l = tk y sk+l = sk . Veamos en primer lugar que podemos tomar k = 1. En efecto, si k > 1, por la parte (ii) del Lema 7.18 tendremos βk+l = βk , mientras que la parte (i) implica entonces ak+l−1 = [βk+l ] = [βk ] = ak−1 . Como tambi´en tenemos 1 1 = αk+l = αk = αk+l−1 − ak+l−1 αk−1 − ak−1 se concluye αk+l−1 = αk−1 , es decir, tk+l−1 = tk−1 y sk+l−1 = sk−1 . Reiterando el proceso, llegaremos a tl+1 = t1 y sl+1 = s1 . Esto quiere decir que αl+1 = α1 , lo que implica que tendremos √ d = [a0 ; a1 , a2 , . . . al ]. Por otra parte, podemos escribir: β1 = βl+1 = al +

1 1 = al + βl al−1 +

= . . . = al +

1 βl−1

1 al−1 +

1 al−2 +

1

..

.

1 a2 +

80

1 a1 + 1 β1

de donde se deduce β1 = [al ; al−1 , al−2 . . . , a1 ], o equivalentemente β1 = [al ; al−1 , al−2 . . . , a1 , al ]. Por otra parte tenemos tambi´en √ β1 = a0 + d = a0 + [a0 ; a1 , a2 , . . . al ] = [2a0 ; a1 , a2 , . . . al ]. Comparando ambas expresiones se obtiene el resultado. En realidad, no nos va a interesar el aspecto sim´etrico que tiene el periodo, sino simplemente saber que empieza en a1 y qu´e longitud tiene. √ Definici´ on. Llamaremos longitud del periodo de d al m´ınimo entero positivo r para el √ que podemos escribir d = [a0 ; a1 , . . . , ar ]. √ Teorema 7.20. Si el periodo de d tiene longitud r, entonces tk = 1 si y s´olo si r|k. √ Demostraci´ on: Como d = [a0 ; a1 , . . . , ar ], se sigue αir+1 = α1 para todo i, es decir, 1 1 =√ . αir − air d − a0 De aqu´ı sigue, junto con el Lema 7.15, √

d − a0 = αir − air

√ sir + d = − air tir

luego quitando denominadores queda √ (tir − 1) d = sir − air tir + a0 tir , de donde deducimos tir = 1 para todo i, es decir, que tk = 1 si r|k. Supongamos ahora tk = 1. Por tanto, por el Lema 7.15 se tendr´a αk = sk + tomando parte entera se sigue √ ak = [αk ] = sk + [ d] = sk + a0 .



d, y

Por tanto, αk+1 =

1 1 1 √ = =√ = α1 αk − ak (sk + d) − (sk + a0 ) d − a0

y por tanto el bloque a1 , . . . , ak se repite peri´odicamente en la fracci´on continua de decir, r|k. El resultado final es entonces el siguiente: 81



d, es

√ Teorema 7.21. Si el periodo de d tiene longitud r, entonces las soluciones positivas de la ecuaci´on de Pell x2 − dy 2 = 1 son: (i) Si r es par, x = pir−1 , y = qir−1 . (ii) Si r es impar, x = p2ir−1 , y = q2ir−1 . Demostraci´ on: Por el Corolario 7.14, las posibles soluciones de la ecuaci´on de Pell son √ de la forma x = pk , y = qk donde pqkk es un convergente de d. Por la Proposici´on 7.16, x2 − dy 2 = (−1)k+1 tk+1 , por lo que debe ser (−1)k+1 tk+1 = 1. Como adem´as tk+1 es positivo, x = pk , y = qk es una soluci´on de la ecuaci´on si y s´olo si k impar y tk+1 = 1. Por el Teorema 7.20, esto es equivalente a que k sea impar y r|k + 1. Distinguimos ahora dos casos: –Si r es par, entonces la condici´on r|k + 1 ya implica que k es impar, luego x = pk , y = qk es una soluci´ on de la ecuaci´on si y s´olo si se puede escribir k = ir − 1. –Si r es impar, la condici´ on r|k + 1 equivale a que se pueda escribir k = i0 r − 1, y que k sea impar equivale ahora a que i0 es par, luego se puede escribir k = 2ir − 1.

82

Related Documents


More Documents from "Jose Martin Chuctaya Macedo"

Estadistica Con Spss 24
January 2021 1
March 2021 0
March 2021 0